Вы находитесь на странице: 1из 68

MAY 1990

'\-\\

The student ol math and science


e
E.
LIJ
J
CD I
I

Nrl I rondl (;dl]erl ol Art, llrd ih iIlto, Colllctrrrr


/ rrr i\ Ir r r rrri I lr-i l, ilr i .l Iri,r, r rr r a, -\L1,4

The Blank Signttture (1965) by Ren6 Magritte

fhe Belgian artrst Ren6 Magritte t1898-19(r7l \,\ras a master of thc visual pun and optical paradox. His fe1low Surrealist
I Salvador Dalr called hirn "one of thc most an.rbiguous pirinters of our time."
Surreahsrn was a moverrent ln arr antl literature that ilourishecl in thc 1920s and 1930s. There was no single Surrealist
style, as can be seen by looking at thc u-ork oi sr,rch artists i1s Max Ernst, foan Mrr6, ancl Paul Klee. Magritte's approach was
to present rninutely detailed, realistrc clepictions oi an impossible or irrational world. By provoking a sympathetic response,
the painting induces thc vieu,cr to acknorr-lec1ge the inherent "sense" of the irration:rl and logically inexplicable.
Perhaps you've l-rac1 a similar fccling rr.hen conirontecl rr,ith the sqr-r;rre root oi -1 or non-Euclideirn geometr).. The t,er)'
Magritte plays with the notions oi "s,.rbie ct" ar-rcl "background" in this painting is not unlikc thc rvay scientists and ilathe-
maticians play with assumptiolls ancl r.ariabLcs, proclucLr-rg r-a1id concepts that ilr- rn the face oi colnnlon s!-nsc.
Even after you've figured out the "trLCk" in anv partrcr-rlar painting br'\{agr-rtte, rt r'emains LlnsettLlng ancl uncannr- ,:nl r,
you like that sort of thing, weirdl,v chamring.
O UA TU MAY 1990

FEATURES
4 Where Biology Meets Physics
]low do we ll'eathe?
by K. Y. Bogdanov

I Where Middling Values Reign


Consideralion$ ol coltlinttity
by S L Tabachnrkov

lB Where Scales Are Weighed


Iempel'alure, [Eal, altd lltemomelers
The sllldslll lllagarIs,i0l lnrlh and $cisl]cB
by A Kikoyin

Cover aft by Sergey Ivanov


24 Where Biology Meets Mathematics
The surface pierced by the furious
bull's horns is a hyperbolic par- The Ueometl'y 0lrupulatiolt gettstics
aboloid. It's interesting that this by I M Yaglom
curved shape can be constructed {rom
two interlaced sets of straight lines:
any two lines belonging to one set are
askew, while any two lines from dif- DEPARTMENTS
ferent sets always intersect. Also,
each point of the surface belongs to
just one line in each set.
3 PuhlisItel''s Paue 43 0uanlum Smiles
Disorder in the court l
Here's a more precise description. 1 3 Bt'ainleasers
A hor,se is a horse
Imagine two points moving at the
same speed along two skewed lines. 14 Gefling to lfiuw...
The cltentical elements
4l At lhe Blailh0al'd
The line joining them sweeps out a Constrttctions tt*itlt compass
hyperbolic paraboloid. In algebraic 22 ]low h
You tiuure? ttlone
terms, the equation for this surface
with respect to the appropriate space 3 0 In Youl' ]lead 5 0 Malfiematical Sul'lrises
Friezinl4 our ltrav into
coordinates is z: xy. Ballpark estimates sun1n7 e I
' Our buli in his rage is unable to tell 2 l(aleidrscoIe
green from red, iust as persons suffer-
3
What a commotion!
52 In the Lah
ihg from color blindness have trouble Walker in a winter
With these two colors. Strangely 34 Conlesl wonderland
enough, this medical condition1ras When days are months
At sixes and sevens
54 tla[[eninUs
something in common with our re- Summer study in New York
markable sur{ace-see "The Geome-
try of Population Genetics," which
3 0 Lookinu Baclr and Tartu ... The Amefican
Regions Mathematics League
Tlte secre t ctf the
starts on page24. Venerable Cooyter Bulletin Board
From the prehistory, of 5
g Solulions
radict
84 [[eckmate!
S),mmetry on the Chessboard

0uanlum/lttlay I000
Have you written an article that you
thinkbelongs in Quantum? Doyou
have an unusual topic that students
would find fun and challenging? Do
you know of anyone who would make
agrcAt Quantum author? Write to us
and we'1l send you the editorial gurde-
OUANTUM
THE STUDENT MAGAZINE OF N/ATH AND SCIENCE
lines {or prospective Quantum con- A publication of the National Science Teachers Associatiln (NSTA)
tributors. Scientists and teachers iri & Quanrum Bureau of the USSR Academy of Sciences
any country are invited to submit ma- in conjunctionwith
terial, but it must be written in collo- American Association of Physics Teachers (AAPT)
the
quial English and at a level appropri- & the National Council of Teachers of Mathematbs (NCTM)
ate {or Quonrum's predominantly high
school readership.
Publisher
Bill G. Aldridge, Executive Director, NSTA
Send your inquiries to:
USSR editor in chief Managing editor
Managing Editor Yuri Ossipyan Timothy Weber
Quantum Vice President, USSR Academy of Sciences Production editor
17 42 Connecticut Avenue NW Elisabeth Tobia
US editor in chief for physics
Washingtoq DC 20009-1 171 I ntemational consultant
Sheldon Lee Glashow
Edward Lozansky
Nobel Laureate, Harvard University
Adverising manager
Be a lacton in the
US editor in chief for mathematics Paul Kuntzler
William P. Thurston Director of NSTA publications

OUANTUM Fields Medalist, Princeton University Phyttis Marcuccio

Equalioll!
US advisory board
JackM. Wilson, Executive Officer, AAPT

W
James D. Gates, Executive Director, NCTM
Lida IL Barrett, Dean, College of Arts and Sciences, Mississippi State University, MS
George Berzsenyi, Professor of Mathematics, Rose-Hulman Institute of Technology, IN
Sciencc and
Arthur Eisenkraft, Science Department Chair, Fox t ane High School, NY
Math Events: Judy Franz, Professor of Physics. West Virginia University, WV
Donald F. Holcomb, Professor of Physics, Comell University, NY
Connecting
. ai;' .::;';"'i 'l
-.. '] .,.i
Margar:et J. Kenney, Associate Professor of Mathematics, Boston College , MA
and f, ji . -,::.,iJ:,...r, Larry D. Kirkpatrick, Professor of Physics, Montana State University, MT
Competing ''€,,,'..t=a=.a Robert Resniclg Professor of Physics, Rensselaer Polytechnic Institute, I{Y
,_.> ,:..-;: ). ,.:
-...'^ra.- MarkE. Saul, Computer ConsultanVCoordinator, Bronxville School, NY
Barbara I. Stott, Mathematics Teacher, Riverdale High School, LA
When you are trying to build student
interest and enthusiasm in math and USSR advbory board
science, few resources can match the Sergey Krotov, Chairman, Quantum Bureau
excitement generated by science clubs Victor Borovishki, Deputy Editor in Chief, Kvant magazrne
and competitions. But how do you get Alexander Buzdin, Professor of Physics, Moscow State University
your high-school students involved? And Alexey Sosinsky, Professor of Mathematics, Moscow Electronic Machine Design Institute
how do you keep them involved? With
plans for successful fairs, details on 25 Quantum (ISSN 1048-8820) contains authorized EnglishJanguage translations from
national and international contests, and Kvant, aphysics and mathematics magazine published by the Academy of Sciences of
commentary by 89 prize-winning the USSR and the Academy of Pedagogical Sciences of the USSR. Copyright @ 1990
scientists, this new publication prepares National Science Teachers Association. Subscription price for 1990-91 (four issues) is
you and your students for $9.95 in the US and $17.95 for all other countries. Bulk subscriptions: 20-49 copies,
connecting and competing in the 1990s. $9.00 each; 50-99 copies, $8.00 each;
#PB-47,1990, 196 pp. $7.00 100+ copies, $7.50 each. Correspon-
All orders of $25 or lass must be prepaid. dence about subscriptions, advertising,
Orders over $95 must inilude a purchase order. and editorial matters should be addressed
All orders must include a postage and handling
to Quantum, 17 42 Cornecticut Avenue
National
fee of $9. No credits or refunds for returns.
Send order to, Publications Sales, NSTA 1 749 NW, Washington, DC 20009-7171.
ConnecticutAve. NW, Washington, D.C. 90009.

OUAilTUlt]l/1l4AY 1 SSO
PUBLISHER S PAGE

0uanlum in oulel' $paGB


and IhB innel' $pace ol al't

PACE TRAVEL: PERHAPS the world. It would cover many space USSR. The technical aspect of the
nothing else syrnbolizes quite so science topics of interest to students illustration is always coupled with an
well the scientific aspirations and in all these countries. This special appeal to the aesthetic side of our
achievements of our age. Count- issue of Quantum would also contain nature. Often the art draws upon a
less generations o{ stargazerswould schedules of activities for the Intema- fable, legend, or myth; sometimes it's
envy us as we explore the outer re- tional Space Year, including student related to a famous painting or sculp-
gions, whether "in person" or by means exchanges, competitions, and other ture. (The picture of the three cows in
of sophisticated bundles of instrumen- kinds of events that are likely to oc- the Yaglom article, for instance, was
tation. Isay "we," but how many of us cur. I encourage all of you to partici- inspired in part by a well-knov,n sculp-
will ever get the chance to see an pate in the International Space Year ture by Kanova, "The Three Graces,"
Earthrise on the Moon, or conduct activities. We'Il try to keep you in- in the Hermitage Museum in Lenin-
experiments in orbit, or travel to an- formed through Quantum magazine, grad.) Sometimes historical figures
other planet? Wel1, I'11 have to be especially the ]anuary 1992 issue. and events are cleverly woven into the
satisfied that my photograph has flovrn artwork.
in space-copies of the premier issue I'n rrre ro MAKE afewcotnments Whatever approach the Kvant afi-
of Quantum have been caried aboard about the illustrations provided to us ist takes, the illustrations end up being
Soviet and American spacecraft. It's tor Quantum by our colleagues in the more meaningful. They convey far
my belie{ that more than a few of you Soviet Union. Ever since I first more than the technical idea, impor-
will have the opportunity to explore encountered Kvant magazine,I have tant as that is-they're also pleasing
the outer reaches of space, and not just been fascinated by the differences iust to look at. I hope you enjoy the art
in picture form! between Kvant artwork and the kind in Quantum. Because of dif{erences
In February a conference was held of technical illustration we usually in printing technology, the art you see
in Deauville ,Fratrce, on the Interna- encounter here in the United States. is even more resplendent than it origi-
tional Space Year, which is set for Our illustrations are technically cor- nally appeared in Kvant. Perhaps,
1992. An educational component was rect and simple, with nothing super- with the changes that are so rapidly
co-chaired by Victor Borovishki, the fluous. I recently had the opporturuty ransfonrring theUSS& this will change
deputy editor in chief of Kvant maga- to explore this topic with Kvant's afi as welI. But one thrng I hope does not
zine in the USSR, and myself . Some director, Sergeylvanov, who was among change is the artistic tradition repre-
40 countries were represented. One o{ the group of Kvant editors who visited sented by Sergey lvanov and his fel1ow
the major recommendations to come the Qu antum off ices in Washington artists. It seems to me dhlr is one of
out of this conference is to designate to participate in our advisory board many areas where we can leam some-
thelanuary 1992 issue oI Quanntm an meetilg. thing irom the Soviet Union.
official publication of the Intemational A talented artist himself, Sergey G. Aldridge
Space Year. Authors would be drawn creates some of the artwork that ap-
-Bill
from as many countries as possible, pears in Kvant and Quanrum. {In fact,
and the articles wouldbe written by Sergey drew the cover of this issue
some of the best scientists in those specifically for Quantum.) What I
countries. The rnagazine would be discovered in talking with Sergey is
translated into as many languages as that the illustrations rn Kvant are ot'
possible and distributed throughout ten prepared by leading artists in the

OlJAlllTUll,l
li,.{

I ' ' : :: " ::


': ' \

-\ -\

-x+".
-da
--\-.<

J
-)a
on whether the lungs are filledwith a
solution because elasticity doesn't
depend on that. Niirgard's data could
be explained when it was found that
the inside of each alveolus is coated
with a thin layer of liquid. This cir-
cumstance fundamentally alters the
Howdutttlehl'eathe? mechanicai properties of the alveolus.
In particular, the pressure needed to
in{late an alveolus is greater than that
needed to inJlate a hollow ball made of
"ln pulmonary tissue. Here's why.
and out" is the silly answer, but the It's known that the surface of a
straight answer may surprise you liquid behaves like a piece of stretched
film-that is, it possesses surface ten-
sion. In order to estimate the role of
surface tension in the mechanics of an
by K. Y. Bogdanov 'lt is now impossible to alveolus, let's consider a spherical film
clarify biolog tcal questions if made of a liquid. The simplest ex-
you don't know phystcs." ample of such a film is a soap bubble.
Mayer, 1842 Because of the liquid's surface ten-
-Julius
sion, the air pressure inside the bubble
HAT DOES AN ELECTRO- vessels. An adult has 700 million of is always greatil than the outside pres-
cardiogram tell us? How do these interconnected, air-filled alve- sure. The amount of excess pressure
bats catch flies in the dark? oli 1fig. 1). At any given moment the inside a sphere of radius R is deter-
How do carrier pigeons find blood vesseis surrounding the alveoli mined by Laplace's relation Lp =ZolR,
their way home? The answers to contain approximately 70 ml of blood. where o is the coefficient of surface
these and many other questions are Carbon dioxide diffuses from the blood tension for the film. The greater this
given by biophysics, the science that into the alveoli, while oxygen diffuses excess pressure/ the smaller the radius
studies physical phenomena in living in the opposite direction. The huge of the sphere. In a soap bubble film,
organisryrs. The biophysicist works combined surface area of the alveoli both the inside and outside surfaces
on the boundary between physics and makes it possible to saturate the blood are in contact with air, so the coeffi-
biology, using the latest achievements with oxygen and cleanse it of excess cient o for the film is twice the coeffi-
of physics in biologrcal investigations. carbon dioxide.
Armed with our knowledge of high
school physics, let's try to answer a Bl'ealhing altd $oil [ulhles
seemingly simple biologrcal question: Is it easy for us to take a breath?
How do we breathe? How much does the air pressure in-
side the lungs (in the alveoli) exceed
0ur l'esphalory syststn the pressure outside (in the pleural
When our organism breathes nor- cavity) during the process of taking a
mally, it consumes about 0.5 kg of breath?
oxygen aday andexhales almost the If each alveolus is assumed to be a
same amount of carbon dioxide. Oxygen hollow ball made of an elastic mem-
enters and carbon dioxide exits through brane, the pressure needed to keep it
the lungs. in-{1ated for a glven extemal pressure is
The inner space of the lungs com- determined completely by the ball's
municates with the atmosphere via diameter and the membrane's elastic-
the respiratory tract. The respiratory ity. Is this correct?
tract consists of the nasal ca\,,lty (where In 1929 the Swiss scientist Karl von
air is warmed and moistened), phar- Niirgard discovered that the pressure
ynx, laqmx, trachea, and the two pnn- needed to inflate the lungs can be
p cipal bronchi (which supply air to the substantially reduced if the lungs are Figure 1

S right and left lungs). filled with a physiological solution. In Tlte tenninal branches of a lttng tte the
no way did this accord with the idea ttlveoli. The ditu'neter of ttn alveolus is, on
$ Eachbronchus is dividedinto smaller
overLtge. 0.1 mm. The tralls ttre 0.4 ptnt
S bronchi (bronchioli) and ends in mi- that alveoli are hollow elastic balls: if
thicl<. Tlte totttl sttrf ace ttrett of alveoli in
A croscopic bubbles, or "alveoli," en- in taking a J:reath we overcome only an aduhhuntanbeingis about 100 m2-
5 cased in a thick network of blood elastic forces, our efforts don't depend jttst abottt ltalf a tennis ccttu't!

OUAlllTl,llil/IIATURI
cient of surface tension for the liquid. different sizes. First let's assume that \
In alveoli air-liquid contact takes place the air caviti.es of the two alveoli are
P >D p
only on one side, the inside. not connected with each other (fig.
Let's estimate Ap. For intracellular 3al. Thc air pressure p, rn the left
fluid, o:5 . 10'z Nim. We'llalso take alveolus is pleater than the pressure pr
this value for the fluid that coats the in the right alvcolus lsincc R < R,l.
inner surface of the alveolus. Assum- Now, as soon as we opulr an inragilary
ing that R = 50 pm = 5 . 10-s m, we get valve connecthg the air cavities oi the
Lp:2.103N/m2. two alveoli, the air from the left alveo-
Figure 2 shows how the volume of lus begins to flow into the right alveo- J
the lungs depends on the pressure lus until the pressure in both alveoli
inside them (more exactly, how much becornes equal (fig.3b). So when two
this pressure exceeds the extemal pres- alveoli of dijferent sizes are corurected,
sure). It's clear from the graphs that, if the larger alveolus will inflate and the
not all, then at least a considerable smaller one will cleflate. t)
part of the pressure that expands the It's obvious that such an interac-
Iungs during a breath is spent to over- tion between the neighboring alveoli Figure 3
come the forces of surface tension. would cause all the lung's small alve- A schematic deltiction of two neighboilng
But when the lungs are filled with the oli to deflate and the large alveoli to spherictil alveoli v,ith diff erent urdii when
(tt) tlte di cavities are isolated t:utd (b) the
physiological solution, additional burst. As a result, the lung wouldn't ttit' ctlt ities ate corute ctetl.
pressure is needed only to overcome be able to {unction.
the elastic properties of the pulmo- So how do we breathe? tension dependent on surface area? As
nary tissue. It's obvious that the dif- Lookrng at the lnteraction between a rule, the impurrties reducing o are
ference between the two curves in neighboring alveoli, we assumed that cluite srmilar to orclinary soap in their
figure 2 represents the contribution o{ different alveoli have the sirme surfacc chernical structure. When they dis-
surface tension to the lung's elastic- tensron, which cloesn't depend on the soh.e, they form a thin fihn on the
ity. hr ordinary breathing, the volume state (inflated or cleflated) of the alve- \\'ater s suriace. If the irnpurity con-
of a human being's h.rngs is about 50% oli. The surface tension of pure 1ic1- centration is high, so that the film can
of their maximum volume. It follows uicls is indeed independent of the size covcr the whole slrrlace in a continu-
from figure 2 that the contribution of oi the suriace. But rhc cociiicrcnt oi ous layer, thcn o for such a liclurd is
surface tension is more than 30%. sr.rrface tension of liquids contarning equal to a certain value for the given
N different impurities depends on the impurity. But r,vhen the concentra-
size of the interface bcnr.ccn the 1ic1- tion of the irrpurity is insufiicicnt tor
s\
^\'-
uid and gas. the {ihn to cover the whole suriace,
Again natLu e lia. lorcscen cver\'- the value of the surf;rce tension u-i1l be
\" thing. The licluicl u,etting the insidc somewhere bcnr.cen the corrcspond-
,$
surfacr of rhe.r]r coli contarns, as atr ing values oi cr ior \\'ater and ior the
L'
cS
( impuriil', :r substance that controls impure liquid An incleirse in the
c
k\ thc r alr.re oi the surircc tcnsion in liquid's sllrtacc then leac'1s to a de-
$ sr-ich a \{ay that o is minimal at the crease ir-i the sr-rriace concentration of
g
begindng of :rn inhalation {minimum t1t. rrrtl'r.r rrr .'il]11 cau.cs o to increase,
surface) and rnzrximal at the end 1.r ut:rt. r:- r-.t1r.tc ncarer to o tor water.
\J
.$
\ {rlaximurn surfaceJ. So despite the If t:e Liqr-rrcl s surface area decreases, o
o.Y
fact that the radius of an alveolus Ls rrrlL also decrease.
s very small at the beguning oi ar-r urha- \ou iook at figurc 4. It shows how
lation, the cotttribr.ttion ot 5t1ri.1.L tL:l- surface tension depends on the size of
Figure 2 sion is small. This makes it prp551l.l3 the arr-liquid interface. Do you no-
The rclation between lungvolume and to blow up a deflated alr'eolus L.r'r-ne.rns rice that for thc sanre surfrce arefl oI
pr e s sw e of the air ( br oken line ) or li qui d of relatrvely 1o.,v pre ssule . -\t the contact, the value of o at inhalation is
(solid line) inside.
same tirne, the increase Ln . r\ 1th the always geater than at exhalatron? This
radius of ln alr c,,11.- pt-r',n1. Lr\-crilt- is because the irnpurl.ty that reduces
Bulil's ttotall $o $ilnple flation at the hergi-rr ot a1r rntake. In suriace tension rs in the interior of the
Our use of surface tension to ex- addition, thrs clepen'lelce oi o on the liquid and not jr-rst on the str.rface.
plain the lung's mechanical proper- size of the alr-eolr-r. rcsulate s relations When the surface area increases, the
ties leads to a "paradox" when we befil.een nei.qhlrc,i.urg a1r-eoh, prevent- irnpr-rrity concentration on the sur-
study the interaction between neigh- ing the aFpcarilnce r,,i the rlechanism face decreases. Some molecr-rles oi tl-re
boring alveoli. Figure 3 shows (in shorrn rn trgtrre l. impuntl'r-ush to the silrface t,ut e .lur-
outline) two neighboring alveoli of \\:hr .lo rr-r"rpnrrties rnake surface hbntttn rsrl'I e:tab-hrlt..l t :: -: -. :-. :,, ti c-

OUAlllIUllil/llllAY 1 SgO
ously. So when we begin to inhalg for applies, for instance, to cold-blooded crease the depth of your breathing
instance, the rapid increase in the liq- animals-frogs, lizards, snakes, croco- (that is, the volume of inhaled air).
uid's surface area is accompanied by a diles, and so on. Since these animals Obviously, i{ the volume of dead space
sharp increase in o because the impu- don't need to spend energy to heat is made equal to the maximum pos-
rities dissolved in the liquid have no their own organisms, their need for sible inhalation volume {about 4,500
time to emerge on the surface. Equi- oxygen is, on average, ten times less cm3), you'Il start to suffocate-no fresh
librium between impurity molecules than that of warm-blooded animals. air enters the alveoli at alll The pres-
sets in only at the end of the inhala- So the surface area across which gas is ence of dead space in the respiratory
tion. In much the same way/ the erid exchanged between blood and air is system of mammals is a "mistake" of
of the exhalation corresponds to an less than in warm-blooded animals. nature.
equilibrium state. This explains the The relative reduction of the lung's And nature made a second "mis-
dependence of o on surface area shown surface area in cold-blooded animals take," I would say, in its design of
in {igure 4. is due to the fact that the diameter of mammalian lungs. This deficiency
their alveoli is approximately ten times has to do with the factthat ar moving
greater than the diameter of the alve- in the lungs changes duection as inha-
t$ oli of warm-blooded animals. The lation rs followedby exhalanon. Almost
a comparatively larger radius of the al- half the time the lungs are practically
veoli makes it possible to inflate them idle--during exhalation fresh air doesn't
'n\
.$
easily even if there is no SAS on their enter the alveoli.
inner surface. (In fact, Lp - llR.) But nature again attained perfec-
v
Another group of animals that have tion in the bird family. Besides ordi-
D'
S no SAS rn their lungs consists of warrn- nary lungs, birds have an additional
h
blooded creatures that live a rather system consisting of five or more pairs
fast-paced, active life: birds. Mam- of air bags connected to the lungs (fig.
mals and birds of eclual weight have 5). The cavities of these bags are
almost the same energy expenditures, widely distributed tfuoughout the body.
, €l""qn
a ofl .tuufaes azu,"{ and a bird's oxygen needs are great as
l-igure 4 well. But a bird's lungs have the CONTINUED ON PAGE 42
The dependence of o onthe swface arca of
liquid-atu contact (1) for liquid isolated
unique ability to saturate the blood
fuom the alveoli of a healthy percon and with oxygen when it is flying at great
(2) for liquid taken from the alveoli of altitudes {about 6,000 m), where the
newboms unable to breathe independently. concentration of oxygen is half that at
sea level. Mammals (including hu-
So we have substances in our lungs mans) at such a height start to'feel a
that reduce surface tension and make deficiency of oxygen, sharply limit
our breathing easier. But where do their activity, and sometimes even
they come from? It turns out they're become dazed. How can the lungs of a
in
slmthesized by special ceils located bird, without any SAS, allow it to
the alveoli walls. These "surface- breathg saturating its blood with oxygen?
active substances" (SAS) are produced And why can't mammals do this?
throughout a human being's li{e, from Let's engage in a little self-criti-
birth to death. Curve 2 in frgure 4 cism. What's wrong with our lungs?
corresponds to pulmonary liquid with For one thing, not all the inhaled air
a reduced SAS content. You can see takes part in the exchange of gases
that theminimumvalue of ois eight with the l:lood. The air inside the
times the norm. Some babies are bom trachea and bronchi at the end of an
without the cells that generate SAS. inhalation can't give oxygen to the
hr these rare cases/ the newl:oms can- blood and take carbon dioxide from it
not breathe on their own. Unfortu- because there are practically no blood
nately, infants still die all over the vessels in these parts of the lung. So
world, never taking their first breath, the portion of the lung occupied by the
because of a deficiencv or absence of trachea and bronchi is called "dead
SAS in their alveoli. space." As a rule, the dead space in a
human being's lungs is about 150 cms.
An uceilion lo lhs ruls You can artificially increase the vol- Figure 5
The respiator.v system of tt bid: (1) hmgs;
Many animals thatbreatheby means ume of dead space bybreathing through
P4) an'bttgs. Auotus show the ntoventent
of lungs do not suffer at all from the a long pipe. If you try this, you'll ctf atr when the btud (a) brettthes in and (b)
absence of SAS in their alveoli. This probably notice that you have to in- breatltes out,

OUAIIIIUll,l/TEAIUBI
Considel'alions ol coilinuily
lf you continue reading long enough, you'll learn how to get
your wobbly kitchen stool to sit still
bvs L Tabachnikov
OST OF THE PROBLEMS IN
your math textbook ask you to
solve equations, or compute a
certain specificvalue/ or con-
struct a geometric figure with given
properties, and so on. This article is
devoted to problems of another sort:
you're merely asked to prove that the
recluired root/ or number, or figure, or
whatever, actually exists. The method
we'll use to solve them is known as
"considerations o{ continuity."

Plane sels
We'll start rl,ith the iollorving prob-
lem: Given a certaur set bor-rnded b,v a
closed curve in the plzrne, pror,e that
there exists a vertical hne that drvrdes
it into two parts of equal area.
Let's take a vertical line to the ieft
of the grven set (iig. 1al and start mov-
ing it to the right. The line eventually
touches the bor-rndary of the set (fig.
1b), then slides over the set (fig. 1c-1f)
until it moves past it to the right (frg.
1S). As the line movecl, horv did the
area of the part oi the set to the leit oi
the line {shown rn recll r.arr'l Obvi-
ously this area changed ccuttinttctttsls'
from zero 1fig. 1a) to thc total area S of
the set (iig. 1g1. So at sorle tn-ne this
area was eqr-ra1 to ex.rcth-ha1i of the
total are:r. At that:i--cc1sc mornent the
line rlrr-r.1..1 r..., --: -nr ,r\ro p,rlrt of
e.lu:il ar.a -i l.
\\ l-1...: .r,r i'c,'.r tiruil< of this solr.rtion?
It Jr'-s:r i aIi .1 method ior construct-
1r: r:. :u.lLlrrccl line-it only cstab-
o
.:1.. :L. J,-i)fc]]c?. 1Actua1ly, it wotildn't
:::.,--<e sense to look for a recipe for con-
.t:r--ctrng this line-the given set is
E It sholrrs that there exists a
.ilL.,itrarJ-.)
a
c
llrle oi drbitrary dfuection dividlng the
i set into two parts of equal errea, Also,

OUAlllIUllll/llllAY 1 SOO
Figure 1 a
p0040 b fg
the solution implies that there is a rem goes like this: II f(x) is a continu- simultaneously divides each of the two sets
unique line in any given direction that ous function on the closed interval into two parts of equal area. {This problem is
divides the set into parts of equal area. often called the plane version of the ham.-
la,bl and c is some number between and-cheese-sandwich problem: Can one always
We'lI use this fact later. the numbers f (a) andf (b), then there
cut such a sandwich with one slash o{ the
Let's look at the solution from a diJ- exists a point xo on the intervalla,b) knife lnto two parts so that each contains as
ferent ang1e. Choose a horizontal nu- such that//xoi = c. much ham and as much cheese as the other?)
merical axis in the plane (fig. 2a). The For example, in our problem the (b) What happens to this problem i{ one o{

position of an arbitrary vertical line I is function /(x) assumes the value S/2 the sets contracts to a point?
(c) Suppose each of the sets in case (a) ls a
determinedby the number x (thepoint contained in the interval between the
parallelogram. Construct the required 1ine.
where the line intersects the numeri- valuesf (a)=0 andf(b)= S. 3. Given a convex set/ prove that there
cal axis). Consider the area of the part The intermediate value theorem is exists a line that simultaneously divides the
of our set to the left of line I as a almost self-evident. Like many obvi- area of the set and the length of its boundary
finctionf(x). The graph of this con- ous statements, though, it's not too curve ilto equal parts.
tinuous function is shown in figure easy to prove. We'd have to go into an
4. (a) Glven two convex sets, one con-
tained within the other, prove that there
2b. Frnding a vertical line dividing our in-depth treatment of the notions of exist two parallel chords of the bigger set,
set into two parts of equal area is then "continuous functions" afid "real tangent to the inner one, such that S,= S, (fig.
the same as finding a point c on the numbers." This would distract us 3).
numerical axis such that f(c) = Sl2. from the main topic of this article/ (b) What happens to thls problem if the
so
inner set contracts to a point?
Now let's consider the horizontal let's just leave it at that-the ilterme-
5. Prove that any convex set contains
line y: S/2 rn figure 2b. The left part of diate value theorem is simply obvious equal and parallel chords dividing its area
the graph of f (x)lies below this line to us. into three equal parts.
and the right part lies above it, since And another thing: we didn't prove 6. Prove that a square can be circum-
f(a) :o < S/2, while f (b) : S > S/2. So that the area to the left of our vertical scribed about any convex set.
there exists a point c where the hori- line continuously depends on its posi-
zontd line y= Sl2afithegraphof f(x) tion (that is, that f1x) is a conti nuous Chail's altd sqtlares
intersect. It's precisely at this point c function). This is almost self-evident Imagine that the floor in your krtchen
thatwehavef (c): S12. too. Throughout the rest of this ar- is not too even (which is often the case
The property of continuous func- ticle, we'll dispense with proving the if it's covered by linoleum). A kitchen
tions used in this proof is known as continuity of functions arising in our stool will usually touch the {1oor with
the "intermediate value theorem"-a solutions. only three of its legs, while the fourth
continuous function assumes all the will be slightly up in the air. Is it
intermediate values between any two Problems always possible to move the stool so
of its values. 1. (a) Given a convex setr and apoint iying that all four legs touch the floor or, no
outside it, prove that there exists a straight
A more for:rnal version of this theo-
line passing through this point that divides
the set into two parts of equal area.
{b) What happens to this problem if the
point extends to infinity?
(c) Solve case (al when the point is inside
the set.
(d) Is it true that there is only one solution
in cases (a) and (c)?
2. (a) Given two convex sets {which may
intersectJ, prove that there exists a line that

1A
"convex set" is a set that contains
all line segments ioining any two of its
points. For example, a disk (the part of
the plane bounded by a circle) is convex,
while its boundary (the circumference
Figure 2 itself) is not. Figure 3

OUAlIIUIl4/TtAIllRt
"h
h7
Figure 4 a b tv

matter how you move it, will it al- A and C continue to barely touch the Problems
ways wobble? Of course/ we assume floor, while the distance from floor 7. Your math teacher asked the class to
prove that any convex set can be divided by a
that when the stool is on all {our legs, level (that is, the depth) of leg B re- pair of perpendicular lines into four parts of
it doesn't have to be absolutely level. mains equal to that of D. AJter we've equal area. One of your classmates proposed
(Before you begin the mathemati- rotated through an angle of 90o, legs A thefollowingsolution: "Choosing an arbi-
cal solution of this problem, try to find and C will occupy the positions of legs trary direction, we can draw a unique iine 7
the experimental solution right in your D and B in figure 4c, while legs B ard D that divides the area of our set in hal{ ({ig. 5a).
Each hal{ can be divided by lines perpendicu-
own kitchen. AJter you get a positive will take up the positions of legs A and 1ar to 1 (fig. 5b). Let's begin to change the
answer to the question, come back to C in figrre 4c. While the stooi is berng directron of 7, repeating the previous con-
the mathematics of the problem.) turned, the distance of legs B andD figuration. After the direction changes by
We'lI assume that the kitchen floor, from the floor changes continuously. 180', points A and B will change places. So at
though not a p1ane, doesn't differ too At the outset (fig ab) this distance was some moment these points will coincide. At
that tlme figure 5b will look like figure 5c
much from a plane surface. This as- positive, since both legs B and D were
and we will have obtained the requlred pair
sumption allows us to ignore floors of above floor level; at the final moment/ of perpendicular lines."
"pathologrcal" shape-for example, the shown in figure 4c, it's negative, since Find the error in this argument.
(a)
floor of a cave covered with stalag- legs B andD arebelowfloorlevel. So Find a correct solution.
(b)

mites (upside-down calcium icicles). there must have been a moment when B. Prove that any convex set with a center
of symmetry possesses an lnscribed square.
So let's suppose that legs A, B, C oI that distance was equal to zerc. At (Actually, a square can be inscribed in any
our stool are on the floor, while the that instant the stool was standing on set. But for theproof of this statement, given
{ourth leg D hangs in the air (fig. 4a). all four legs. by the outstanding Soviet mathematician
Without lifting legs A and C off the Our solution has not only theoreti- L.G. Shnirelman (1905-1938), simpie con-
floor, rotate the stool about Lrne AC so cal but practical value. By rotating a siderations of continuity are no longer suffi-
cient.)
that legs B and D are both off the floor stool about its center by less than 90o,
at the same distance from it (fig. ab- we can always find its stable position. Figure 6
the distance from the floor is counted You can check this experimentally.
in the direction of the legs). In this One more thing. Our solution is
position (two legs on the floor, the based on the fact that the extremities
other two at the same distance from of the four legs of a stool form the
the floor) the stool can be moved. vertices o{ a square. Were they located
Imagine for a minute that the floor at the vertices of a rectangle, or some
ab
is made of soft clay into which the legs other quadrilateral, our argument would 9. Gir.en t\vo convex sets, one cont:rined
of the stool can easily penetrate. Let's nothave worked. I don't know if the insidc t1-re other, prove the follou,rng state-
1-ncnts:
lower the stool until legs B and D just statement of the problem is true for
ial The rc are t\v(-) parallel chorcls oi the
touch the floor, while legs A and C any quadrilaterals other than the square. bigger set that arc tangent to the lnner set
have been pushed into the clay below (In any case, it's clear that the quadri ancl arc oi ecpral length (fig. 6a).
floor level to the same depth (fig. 4c). Lateralmust be inscribed in a circle.) If (bl Thcre is a point X of the bigqer set irorr

Now, going back to the setup in you're able to settle this question, rvhich nvo equal tangcnts to the inner set
can be c1r:ru,n lirg (lbl.
figure 4b, let's rotate the stool counter- please let us know.
Hlrt: Among the chords of the bipg;cr sct
clockwlse about its center so that legs

Figure 5 b

10 OUAI\IIUll4/ll]!AY 1 SSO
that are tangent to the inner one there is a If the number I is of the form Tf n,
longest chord. From its ends, draw tangents f(x)=ax3+bx2+cx+d where n is a positive integer, then the
to the inner set. graph of the function necessarily has a
1 0. Given three nested convex sets {fig.
reduces to the case considered above horizontal chord of length l; i{ 7 is not
7), prove thai there exists a point X of the
by dividing f(x)by a. of that form, there exists a continuous
biggest set from whlch tangents to the irtrler-
most set can be drawn so that the parts of Our proof is a good illustration of function with the same values at the
these tangents contained in the second larg- the strength and weakness of continu- endpoints whose graph has no hori-
est set are of equal length. ity considerations. We obtained the zontal chords of length l.
proof of a difficult fact-the existence
o{ a root-almost "f or free, " but it re-
mains unclear how to find this root for
a specific polynomial. (Actualiy, for-
mulas for the roots of cubic pollmomi-
als exist, but that's an entirely differ-
ent topic.)
Here's another example. Suppose Figure B
f(x) rs a continuous periodic function
of period 7. Prove that its graph has a Problems
horizontal chord of length Tl2. 11. Prove that any odd-degree polyno-
Figure 7 The existence of a horizontal chord mial has a root.
12. Yesterday at midnight it was colder
of length 1 is equivalent to the relation
than it was at midnight the day before yester-
f(") = f(" + 1) lfrg.8). So we have to prove day and today. Prove that at some time today
Rool$ altd chords that there is an x such that f(x + Tl2) = the temperature was the same as yesterday
You know, of course, that a quad- f(x) .In other words, we must show at the same time.
ratic trinomial might not have a real that the function S&) = f(x + Tl2)-f(x)
13. Suppose f(x) is a continuous function
on some closed interval and that all its
root. Such is the case, for instance, has a root-thatis, g(x) = 0 for aceraun
values lie in that same interval. Prove that
with the polynomial xz + x + 1. But x. there exists an xstchthatf (x) = x.
what's the situation with cubic poly- Let's take an arbitrary number a. II 14. The polynomtal f(x) = a* + bx + c rs
nomials? We'll now prove that any Sk) = 0, we're all done. So we'll such that the equation /(x) = x has no real
third-degree polynomial has at least assume that g(a) is nonzero. To be roots. Prove that the eryationf(f(x)) = x also
defirute, let's suppose Sb) .0. Let b :
has no roots.
one real rcot.
15. Suppose l(xl is a continuous periodic
First let's consider a cubic pol1no- a +T12. We can easily computeg(b): frlnction (on the numerical line) of period T.
mial with leading coefficient 1: Prove that its graph has horizontal chords o{
s(b) = s(a +T12) lengh ! where (a) 1 = Tl3, lb) 1 = Tfn, lcl I = lpl
f(r) =r' + bx2+ cx + d. : f( a * T 12 + T l2) - f( a + T l2)
.i) T, (d) 7 is any real number.

:f(a*T)-f(a+Tl2) 16. Suppose l(x/ is a continuous function


on a closed interval of length Twhose values
We can rewrite it in thefollowrng forrn: :f(a)-f (a +rl2) at the endpoints are the same. Prove that the
:-sb). sraphol f(x) has a horizontal chord oi length
f (x)=*lr +bf x+ cl*+ dl*). I where (a) 1 = Tl2, lb) I = Tl3, lc) 1 = Tln. Nso,
Therefore, gb) ,0. construct such a function whose graph doesn't
have horlzontai chords o{ length 2fl3.
If the value of lxl is very large, the The {unction g(x/ is continuous,
17. The "mean value of the function /(x)"
summands b f x, cf *, d/t' become very since it's the difference of continuous on the closed interval la,blis, by de{inition,
small. In that case the number in functions. So once again we can use the number
parentheses is very close to 1 and is the rntermediate value theorem, which
certainly positive. So for iarge lxl the implies that there is a number x be- )-e
D-a - ft*M*.
sigrr of l/x) is detenninedby the sign of tween d and b such that g(x) :0.
Prove that iJ the mean value of a continuous
the number t'. Likewise /(x) is less The statement about horizontal function on a closed interval is 0, then the
than 0 for negative xwith large abso- chords of graphs of continuous peri- il
function assumes the value 0 the iaterual.
lute values, and f (x) is greater than 0 odic functions can be considerably 18. Consider {unction o{ the ftorrt f(x) =
a

for large positive x. strengthened: The graph of such a a,sinx + arsin2x + ... + a,,sinnx + b,cosx +
brcosZx+... + bocoslcr, where ar, ..., anartdb,
Because a cubic polynomial, like function has a hoilzontal chord of ar- ..., bu are real numbers. Prove that the
any polynomial, is a continuous func- bitrary length, equationl(x/ 0has aroot.
=
tion, we can apply the intermediate But for graphs of {unctions defined
value theorem. The theorem implies on a closed interval, the situation is Conclusion
that there exists an x such that f(x) = O, quite different. Suppose a continuous We've worked out several problems
so the cubic polynomial has a root. function is defined on a closed inter- whose solutions are based on consid-
The case of the general cubic poly- va1 of length T and assumes eclual erations of continuity. In each of
nomial values at the endpoints of the interval. them the relevant magnitude depended

OUAlllTl|llll/IIAIURI ll
on one parameter. For example, the
area o{ the part of a set to one side of a
line passing through a fixed point
depended on the line's angle of incli-
nation. Considerations of continuity
also work when there's more than one
parameter. Now isn't the time to go
OUANTUM lllalG$ a [srlsctgill!
into a detailed account of this theme-
Use the response card below to order Quantum for your child, $andchild,
I'11 just leave you with a few theorems
nephew; niece, mother,lather, friend ... Four colorful, challenging/ enter-
to mull over.
taining issues for only $9.951
l. Given three bodies arbitrarily lo-
cated in space, there exists a plane that
divides the volume of each into two Factor x into fhe Quanlum equation,
equal parts. (This is the space version where x is any potentialQuantum reader you know!
of the ham-and-cheese-sandwich prob-
1em, the three bodies being the slice of
bread and the pieces of ham and cheese.2)
2. There are two diametrically op-
posed points on the planet right now
Don't Miss Out on This Inspiring Guide
where both pressure and temperature to the World of Fractals!
coincide. Springer-Verlag, in colperatiln with the National
3. A cube may be circumscribed Council of Teachers of Mathematics, presents . . .

about any convex body. (This theo-


rem generalizes problem 6 to space.)
4. Imagine a sphere covered with
hair. Can it be combed smoothly-
Fractals for the Classroom
By the award-winning authors:
that is, so that each hair is tangent to Heinz-Otto Peitgen, Hartmut Jiirgens, and
the sphere and the directions of nearby Dietmar Saupe.
hairs do not differ too much? The Advised by leaders from the teaching community:
answer is no: there will always be a Evan Maletsky, Terry Perciante, and
hair that will stick out perpendicu- Lee E. Yunker.
larly to the sphere. This statement is
knor,m as the "porcupine theorem" or Based on several lectures and lecture series given to
the "sphere-combrng theorem. " various communities of teachers and students, Fractals
5. This is for those of you who for the Classroom is written especially for teachers
know complex numbers. We've shown and is intended for the high school and college level. As
that any odd-degree polynomial has a such, it will be a valuable teaching aid for classroom use
as well as for independent study.
real root. What about complex roots?
l990/approx. 450 pp., many illus./Hardcover $29.OO (tent.)
It tums out any polyrromial {except a
rsBN 0-387-9704r-X
constant zero-degree polynomial) has
at least one complex root. This state- Forthcoming!
ment is so important it's often called Fractals for the Classroom will be supplemented by two special
the fundamental theorem of algebra. volumes by the same authors:
And one of the approaches to its proof Fractals for the Classroom
is based on-you guessed it-consid- Strategic Lessons on Fractals
9-
(in eaoperation with E. Maletsky, T Perciante and E. Yunker) ea
erations of continuity. CI
L.

Fractals for the Classroom


H
Strategic Computer Experiments on Fractal*
o
H
2An open-faced sandwich!
Illustrations from The Science of Fractal Images edited by H.-O. Peitrm cnd D. Saup€. .D
-Ed. a
a
.D

a Springer-Verlag c
Want to show your students how
chemistry affects their lives?
q) New York Berlin Heidelberg Vienna London Paris Tokyo Hong Kong
I75 Fifth Avenue, New York, NY 10010 (212) 160-1500
a
a-
OPPORruNITIES IN CHEMISTRY: o
Ecl National Council of Teachers of Mathematics
TODAY AND TOMORROW qP 1906 Association Drive , Reston, VA2209l (703) 620-9840 o
the resource book by Pimentel & Coonrod
$8.95 fom OpsinOhem, 754 Coventry Rd. To order cal[: I-8O0-SPRINGER (In N], call (201) 348-4033) -,?
Kensington CA 94707 (415) 52$7543 or simplv rvrite to one of,the abovc addressesl

12 OUAl\lTUllil/ll]lAY 1 SSO
BRAINTEASERS

Jusl lol' thg lun ol it


Problems offered for your enioyment
by E. Chernyshov, N, Antonovich, A, Savin,
B. Proizvolov, l. Slobodetsky, and L, Aslamazov

B6
fr-oil a goatt a head of cabbage, two wolves, and a dog be
"*
transported across a river if it's known that the wolf is
"culinarily partialto" goat and dog, the dog is "on bad
terms with" the goat,and the goat is "not indifferent to"
cabbage? There are only three seats in your boat, so you can
only take two passengers-animal or vegetable-at a time.

B7 a shooting match. The first


t-hirty people took part in
participant scored 80 points, the second scored 60 points,
the third scored the arithmetic mean of the number of
points scored by the first two, and each subsequent com-
petitor scored the arithmetic mean of the number of points
scored by the previous ones. How many points drd the last
competitor score?

SOLUT/O/VS ON PAGE 61 BB
W["e" we multiply multidigit numbers, we have to do
some addition as'well-the final step is to sum up the
subtotals. Here's aptzzle that exposes the steps and tags
the various digits as odd ("0") or even ("E"):

EEO
xOO
EOEO
+ EOO
ooooo
Find numbers that satisfy this scheme.

o
6
B9
N
6 Ascuba diver loses his bearings deep in the ocean. How can
Z

ffi
c he tell which way to go to get to the surface?
o
c=
U
810
a
Witti candle burn in a spaceship, where everything is
weightless?

OUA[lIUflil/BRAIlllIIASERS
l3
GETTING TO KNOW

The chemical elemenls


Just like the oldiimer who's "been around,"
many an element has a story to tell ...

by Sheldon Lee Glashow

Phosphonous
The first known discoverer of a inner so-called 4/-shell. Their exis-
chemical element is the German al- tence was first hinted at by the work of
chemist Hennig Brand. In 1669 he the Finnish chemist Gadolin in the
prepared the "light-be arirtg" element very same year that Lavoisier died on
by distilling a mixture of solid and the guillotine. Rare earths are not so
Liquid excrement while trying to find rare and are used in the phosphors of
a liquid that would turn silver into television tubes. A similar family, the
gold. Instead, he found a pearly-white actinides (Z =90-103), corresponds to
waxy stuff that glowed in the dark and the completion of the inner 5f-she11.
could light a pipe. Ironically, Brand's
home town of Hamburg was virtually Gallium
destroyed by phosphorous bombs during When Mendeleyev proposed his table,
World War tr. he left three vacant spaces for ele-
ments not yet discovered and pre-
tltlendehuium [ = 101) dicted their chemical and physical
Mendelevium is an artificial ele- properties. Gallium was found in 1875,
ment {irst made in CaLifomia in 1955 scandium in I879, and ger rnanium in
and named after the pioneer of the 1886. Their names suggest where
periodic table. (A.N. Znoiko in the they were found, and they had just the
USSR suggested the name {or element expected properties. Ga is a rare by-
number 97, whose properties he had product of alumrnum mamrfacture and
predicted, but it was named berke- an essential component of up-to-date
lium after the city of its discovery.) semiconducting devices. Soviet and
Today, there are 21 such arti{icial ele- American scientists now collaborate
ments. The first one, technetium in an experiment using 40 tons of Ga
lZ = 431, was discovered in Italy in iborrowed from the Soviet strategic
1935,while the last three(Z:107-lo9l stockpile!) to study solar neutrinos.
were made in Germany in the 1 980s.
Plutonium is needed to make bombs The plalinum lnstals
that must never be used. The platinum metals form a rec-
tangle in the table: ruthenium, rho-
Ihe l'al'e earth$ fl = 57-71 ) dium, palladium, and below them
The periodic behavior of the ele- osmium, iridium, platinum. They are
ments reflects the sequential filling chemically simrlar and found together.
up of electron shells. These 15 ele- Platinum metallurgy was developed
ments are chemically similar because by American Indians in pre-Colum-
they have two valence electrons but bian times. Along with coffee, to-
varying numbers o{ electrons in an bacco, maize, and potatoes, Pt is a

14 otlAturIt.l/l,lAY lsso
New World gift. Four related metals monia and got different answers. He
were found rn South American Pt ores and William Ramsay solved the puzzle ]lydt'rUen
by two British chemists and friends in in 1894 by showing that l'/" of air is a In 1745 the Russian poet-scientist
the year 1803. Ruthenium was iso- new element they called argon. They M.V. Lomonosov wrote, "On solu-
lated and studiedby the Russian chem- went on to find several other noble tion of any nonprecious metal in acid,
ist Karl Karlovich Klaus. Curiously, gases----chemically inert new elements there emerges an inflammable vapor
all six metals of the Pt family are that did not fit rnto the original table. that is nothing else than phlogiston."
laboriously separated from the same Mendeleyev reluctantly added an ex- It was hydrogen, whose atom is the
rocks only to bi recombined with one tra column. simplest of all. Much Iater, Niels Bohr
another to make special a11oys {or computed its spectrum with his quan-
things like dental fillings, pen points, Bhenium tum rules. The structure of all the
and precision instruments. Rhenium is the last long-lived ele- atoms and the meaning of the periodii
ment to be found. It was discovered by table was to be explained by quantum
Al'Uon a collaboration of three German chem- mechanics.
Lord Rayleigh measured the den- ists rn 1925, two of whom subsequently
sity of nitrogen from air and from am- (consequently? ) married one another. Selenium
Selenium is one of nine elements
named after heavenly bodies (the Sun,
the Moon, five planets, and the aster-
oids Ceres and Pallas). A tiny bit of Se
is essential to our diet, but too much is
v/// toxic. In South Dakota, cows grazing
on plants that concentrate Se deveiop
"alkali disease." Se powder smells
like rotting horseradish.

Chlu'ine

H Lavoisier taught us that burning is


oxidation. He gave oxygen a name
meaning "acid maker" because he
thought it was in all acids. Humphry
\\ Davyproved him wrongby showing
that "muriatic acid" is O-free HCI and
tu that Cl is an element. Fluorine, chlo-
rine, bromine, and iodine are halo-
5 gens-reactive nonmetals whose at-
oms have almost complete valence
shells.
Y
Rubidium
,-) Each element emits light of defi-
) nite colors when heated. These ap-
pear as bright lines when the light
passes through a prism. Many ele-
\) ments were discovered this way and
given names from Greek or Latin for
the colors produced: rubidium (ruby-
I red), thallium (bud-green), and cesium
n (sky-blue). Helium's lines were first
seen in sunlight, which is why it was
named for the sun. O
Sheldon Lee Glashow is the Htggins hofasor
of Physics and Mellon Professor of the
su Sciences at Harvard University.
,r**- -7. -L-"-"--'[)-' - -

Dntitrv Mendele,vev 1)eers ollt ttt the world tltottgh the btns of his peilctdic tttble.
OtlAlllTUlt4/GIIIIlilS IO I(lllOt|ll... l5
Tempel'alure, heal, and lhel'momeler$
What does it mean to "measure temperature"?
Why are there so many thermometer scales?
And how is heat related to temperature?

by A Kikoyin

HE SHORT LIST OF PHYSICAL tures of one degree each, just as a man mass of some body we have to com-
notions we become acquainted of 25 is not the same as 25 one-year- pare it to the unit mass. This is
with before we begin to read, let old babiesl because a body's length and mass are
alone studyphysics, includes tem- So the main thing about tempera- eclual to the sums o{ the lengths and
perature. In inlancy we leam that the ture is that it's not an additive quan- masses of its parts. But temperature
words hot, warm, and cold that de- tity, and this peculiar property will can't be measured this way.
scribe our sensations mean different affect how we measure it. To measure How is temperature measured?
values o{ temperature; that in sum- the length of a body we have to com-
mer the temperature is high, in winter pare it to another body that's consid- Alilolhistmy
it's 1ow. You probably can't recall ered the unit length; to determine the Around 1592 Galileo devised the
when you becarne aware that a healthy first instrument for rneasuring tem-
person's temperature is "98-point-6" perature {although the word "ther-
and that a doctor should be called if mometer" didn't appear in print until
Youte
your temperature is much higlrer. 1624). Galileo's method {or measur-
known that "a11your li{e." ing temperature, shown schematically
Since temperature is so familiar to in figure 1, is essentially the same as
us, we're not usually aware of the that used nowadays. A sma11 glass
special properties that distinguish it bulb (a) is fused to the top of a long,
from otherfamfiar notions like lengttl narrow tube (b) that is open at the
mass, or volume. But the difference is bottom. The bulb is warmed by some-
crucial. body's hands and the lower end of the
If we take ten rods, each one meter tube is dipped in a container of water
long and put them end to end so as to (c). As thebulb cools to the tempera-
form one rod, we get a rod ten meters ture of the surrounding air, the leve1 of
long. Simiiariy, i{ we take ten bodies, the water in the tube rises above that
each with a mass of 1 kg, and lump of the water in the container.
them together, we get a body with a Galileo's thermometer is obviously
mass of 10 kg; and so on. But if we based on the fact that the volume of
combine ten bodies, each at a tem- the gas in the bulb depends on its
perature of 20"C, we end up with a temperature, so that one can judge a
body whose temperature is 20o, not change in temperature by the change
200"C, because the temperatures of in the gas's volume. But the instru-
bodies don't add up when we put them ment.has no scale, so we can't assign a
together, unlike their lengths, volumes, numerical value to the temperature.
masses/ and so on. A temperature of It's not really a thermometer, it's a
100 degrees is not the sum of tempera- Figure "therrrroscop e" shows temperature
-it
l8 OUAl\lTUfil/lllAY 1 SSO
but it doesn't measure it. It took
almost 150 years to come up with a
scale.
The important point for now is
this. Galileo's therrrrometer is based
on an idea we still find useful-that
you don't flreasure temperature di-
rectly, you measure a quantity that is
dependent on temperature. The cluan-
tity
used in Gahleo's thermometer
a gas. In a modern
was the volume of
mercury thermometer, the quantity
* that depends on temperature, and whose
change indicates a change in tempera-
ture, is againvolume-but this time
the volume of mercury, not a gas. For
this purpose we could also use other
physical cluantities-for instance, the
pressure o{ a gas (at constant volume),
the length of a solid rod, or the electri-
cal resistivity of a metal.

E A physhal law lhal calt'l [e discouel'ed


tnlithottla thsrlnolnslgl'
The first crude thermometers and
even thermoscopes led to the discov-
ery of one of the most important physi
cal laws, the law of therrnai equilib-
rium. The date of its discovery, which
no scientist has claimed, is unknown,
perhaps because it seemed self-evi-
dent. The law asserts that any isolated
system of bodies eventually arrives at
a state in which all its components
have the sarne tellrperature. This state
is called the state of thermal equilib-
rium.
Obviously, the law of thermal equi-
librium could be discovered only after
the invention of the thermometer.
On the other hand, temperature meas-
urement itself by means of a ther-
mometer is based on this law. After
all, a thermometer is a body at some
temperature that indicates its own
,l *t. i temperature. So in ordel to assess a
\ body's temperature with a thenlome-
ter, this body must be in thermal equi
e librium with the thermometer-their
temperatures must be the same if the
.-9
-
q. therrrometer is to indicate the body's
F
p . /\ temperature in addition to its own.
a
o
o
That's whyyou always have to wait
l
for some time before reading a tem-
c
i I t >n
-ll
9av peratLue-you're waiting for thermal
equilibrium to be established.

OUAIIIIll[,l/ItAIUft[ 1I
Lomonosov, who wrote: "It is well in temperature if temperature were
Some more [islory known that heat is generated by mo- the concentration of heat in a body.
To recap: the thermoscope appeared tion; hands are warmed up by their The true nature of temperature-
at the end of the 16th century and mutual friction; wood can be ignited that quantity whose meaning remarned
became the thennometer in the middle by rubbing sparks appear from strokes obscure for such a long time-became
of the 18th. But what does the ther- of {lint on steel; iron becomes hot clear only a{ter the kinetic theory of
mometer indicate-what is tempera- from hammering it with strong rapid matter was worked out. To under-
ture? It took scientists another hundred strokes." This led to the conclusion stand how, we have to get a handle on
years to find the corect answer to this' that heat is not a substance but the the basic ideas of this theory.
question. motion of the "imperceptible particles"
Temperature is a quantity that char- (as they called them then) that consti- Muhculan rham and ils laws
acterizes the thermal state of a tute all bodies. The kinetic theory o{ matter as-
body. We say that cold andhot sumes that any physical body
bodies have different tempera- consists of sma11 particles-mole-
twes. So the nature of tempera- cules-that are in constant cha-
ture comes down to this: how is Galileo's motto: otic motion. Complex forces of
a cold body different from a hot attraction and repulsion act be-
body?
"To measure that which is tween molecules. But for gases
The first answer to this ques- under ordinary conditions, these
tion was grven by Galileo him-
lneasunatle, forces are sma1l, and we may
self. He started from the easily and to stniue to make even visualize a gas in which
observed fact that when a cold any interaction between mole-
body is placed close to a hot ong lneasunalle cules is absent. Such gas, owrng
the hot body cools while the to its purely theoretical nature,
cold one becomes warrner. Gali- tftat which is not yet so." is called an " ideal gas."
leo concluded that something An ideal gas consists of a huge
travels from the cold body to number of molecules movrng cha-
the hot body (though we might otically every which way in a
just as well suppose that something Experiments camed out in the 18th container at speeds of hundreds of
travels in the opposite direction). He century led most scientists to recon- meters per second, colliding with each
assumed that it is a specific substance, sider the caloric fluid theory. In 1760 other and the walls of the container.
and in the 17th and 1Sth centuries the Scottish physicist and physician The system as a whole, however, is
most scientists accepted that point of |oseph Black showed that when the su.bject to very strict rules. Since the
view. The substance was called the same amount of heat is added to ec1ual interaction between the molecules of
" caloric fluid." masses of different substances, differ- an ideal gas can be neglected, the rules
According to caloric theory, a hot ent changes in temperature result. If can be stated in a rather simple form.
body differs from a cold one in that it temperature were the concentration In particular, using the laws of me-
contains more caloric fluid. When of caloric fluid in a body, the acquisi- chanics we can find the pressure-
thermal equilibrium is established, tion of the same amount of heat by that is, the force with which the gas
this caloric fluid has passed from the equal masses shouldproduce the same acts on a unit area of the container's
hot body to the cold one. So in this change in temperature. So Black's walls.
view, a body is a mixure of two sub- results were incompatible with the One can shorv by certain calcula-
stances: the material of the body itself theory of caloric fluid. Black discov- tions that in a vessel of volume V
(for example / watert copper/ rron, glass) ered that different substances have containing N moiecules of g:rs, the
and the caloric fluid in it. This is what we now call different "specific pressure on the walls equals
where the name for the unit of tem- heats."
perature/ a " degree," comes from- In 1764 Black observed that tem- 2,V=
the same unit was used to indicate perature remains constant while ice
't)=-'-+L.
.lv (1)

concentrations of aqueous solutions. melts, even though melting requires where f lwhlch equals m? I2l is the
This concept of ternperature was that a large amount of heat be added to mean kinetic energy of chaotic mo-
generally accepted for some 200 years. the ice. From the time of Black's ex- tion assigned to one molecule of gas.
And that's what they talked about- periment this quantity of heat has Equation ( I ) indicates that gas pres-
"degrees of warmth." been called the "latent melting heat." sure is equal to 213 of the mean kinetic
At the same time there was another Likewise, a certain amount of heat is energy of the chaotic motion of mole-
theory, based on the fact that a body removed when water freezes, which cules per unit volume (since N/V is
can be warmed up by mechanical also takes place at a fixed temperature. just the number of molecules per unit
motion. One of the founders of this Obviously, heat could not be absorbed volume).
theory was the Russian scientist or released by a body without a change Real gases require more sophisti-

l8 OUAlllIUlI4/ll,lAY I SSO
cated calculations, but under appro- Various methods were used to con- 0 and that of melting ice as 100. Eight
priate conditions equation (1) can still struct thermometer scales. Every years later, however, this scale was in-
be used. It gives an approximate rela- manufacflrrer or designer of thermorrre- verted and has stayed that way for
tion between p and E that is accurate ters worked out a different one. By the more than 200 years.
enough for small N/V and E. The end of the lBth century there were Even before Celsius, in 1724Fahr-
formula can be used for pressures of about twenty di{ferent thermometer enheit manuf actured thermometers
about I atmosphere and lower. scales. Three of them (too many, in in which two reference points, melt-
At first glance all this has nothing fact) have survived to the present day. ing ice and boiling water, were taken
to do with temperature, which doesn't The three survivors are variations as 32 and 212, rcspectively, and the
even appear in equation (1). To see on a single theme. Earlier in the interval between them was divided
what's really going on, let's tum again century the German glassblower and into 180 regular lntervals (degrees).
to temperature measurement. amateur physicist Daniel Gabriel Like Celsius, the contemporary French
Fahrenheit and the Swedish astrono- scientist Ren6-Antoine Ferchault de
Scales: least allsr lamine mer Anders Celsius calne up with a R6aumur took the temperature of melt-
The first thermometers used for principle for constructing thermome- ing ice as 0, but according to R6aumuls
practical purposes were licluid ther- ter scales that has silce been generally scale water boils at 80.
mometers made by a group of scien- accepted. It's based on the use of two So we see there was a lot of confu-
tists in Florence. Later they were reference points-thermal states that sion when it came to selecting tem-
manufactured in other countries. can be characteized as constant. The perature scales. The number of inter-
Different licluids were used, but the points they chose for this purpose are vals between the two fixed points was
most common ones were alcohol and the temperature of melting ice and arbitrary, as were the values of these
mercury {sometimes oil). that of boiling water at atmospheric reference points themselves. In fact,
The liquid thermometer consisted pressure. (The melting point of any there's no reasonable argument in
of a thin vertical glass tube that bal- solid substance and the boiling point support of assigning the temperature
looned out at the bottom in the form of any liquid at a given pressure are 0 to meltrng ice-as if meltrng ice had
of a small bulb. The bulb and lower also constant, but water and ice are no temperaturel
part of the tube were filled with liquid. more readily available.) Not only that, there's an assump-
hr addition to berng functional, the old In 1742 Celsius proposed the fol- tion hidden in our division of the tem-
Florentine thermometers were cluite lowing prescrj.ption for dividing up perature scale into equal parts-S0,
beautiJul. If you saw them, you might the thermometer scale. A thermome- 100, 180, or whatever. We assume
even consider them works of art- ter is put in contact with melting ice. that the volume of the liquid in the
such a refined artistic approach was Once heat ecluilibnum is established, thermometer is exactly proportional
taken in creating these scientific in- the height of the licluid in the ther- to its temperature.
struments. mometer is marked. Then boiling If we denote the volume of the
water is used liquid at the temperature of melting
instead of ice, ice as Vo and its volume at the tem-
and the new perature of boiling water as V, and
level is marked these temperatures themselves as to
wrth anumber and t, division of the temperature scale
that differs into equal parts means that
from the first v-vo
by 100. The
space between
t-t 0
the two marks where c is a constant. If we assume t0
is divided into = 0, then V -Vo= cttotv =Vo+ ct.
one hundred Can we verify that volume depends
regular inter- on temperature as a linpar function?
vals, each in- Obviously we can'q because we'd have
terval corre- to use a thermometer in the verifying
spondingto the experiment, and in constructing the
change of one thermometer we've already assumed
degree in tem- that volume is linearly dependent on
perature {fig.2). temperature.
Curiously There's an old story about a seaport
enough, Cel- in which a gun was fired every day, ex-
sius took the actly at noon, so that ship captains
temperature o{ leaving the port could set their chro-
Figure 2 boilmgwateras nometers (which were used to deter-

1$
OUAlllTUIl,I/IIAIURI
mine longitude during the voyage) to -r0=_=_73.
100
l-i.
the time of the shot. One of the 0.3661
captains wanted to know how confi- We can see the difference between
dent he coultl be that the gun was this new scale and the old Celsius
really fired at midday. He found out scale (fig. 4). According to this scale,
that the artillery man determined the the temperature of melting ice is not 0
time with the help of a local watch- but 27 3.LS degrees, and zerc tempera-
maker's "very exactwatch." And the ture is 2 73 . 1 5 degrees lower than the
watchmaker told the captain he set temperature of melting ice. This zero
his "very exact watch" by the shot in temperature is called "absolute zero."
the port. Obviously, under these cir- It's the temperature at which an ideal I
cumstances, you can't judge whether gas must have a pressure equal to zero,
the watch is of good quality or the gun if such a temperature could be ob-
is fired exactly at noon. Similarly, tained and the gas would remain in
when we use a thermometer whose the gaseous state. Since gaspressure
scale is constructed on the assump- can't be negative, the temperature in
tion that the volume of its liquid is this scale can't ever be negative.
proportional to the its temperature, Perhaps you've aheady recognized
we can't tell whether that assumption this new scale in my rough sketch. It's
is valid. the Kelvin {or absolute) scale. The
Equally important, in measuring M temperature in this scale is called "ab-
temperature we have to allow for the solute temperature," whose standard
fact that the actual readings depend on notation is 7, and it's measured in
the liquid used in the thermometer or Figure 3 degrees Kelvin. The temperature of
on some of its other physical proper- melting ice is 273.1SoK, that of boiling
ties. Weneed a standard thermome- water 373.15oK, and so on.
ter for grading all the others, so as to sure at the temperature 7o of melting But for most practical purposes, the
make ther readings compatible. How ice. Following the tradition of the Cel- Celsius scale reigns throughout the
do we solve this problem? At present sius scale, let's say world (although most Anglo-Saxon
the standard thermometer is the so- countries still use Fahrenheit). The
called "gas thermometer of constant 7- To: 100. (3) temperature in this scale is denoted as
volume." t and is expressed in degrees Celsius
The di{ference in pressure at the (abbreviated "C). According to this
temperatures of boiling water and scale, the temperature of melting ice
Gas thermomstsrs altd an a[snlule
melting ice is divided into 100 equal is 0"C, that of boiling water 100"C, and
lemperalure scale intervals-that is/ we still assume the so on. Obviously, fC : (7- 273.15)'K.
From experimentation we know linear dependence of temperature on Those who use the Kelvin scale are
that the pressure of hot gas is greater pressure at constant volume. Of course, almost invariably physicists.
than that of cold gas. In the gas ther- we can't iustify the assumption, just And now, finally, we're in a posi-
mometer, then, pressure at constant as the captain couldn't judge if the tion to $asp the real meaning of tem-
volume is the quantity that indicates watch was correct by the gunshot or perature.
the temperature. The thermometer the gunshot was on time by the watch.
shown in figure 3 consists of a con- The assumption of linear dependence So wlalis lemrct'atul'e?
tainer A filled with an "ideal gas" (that is simply the basis for the method of According to the method o{ meas-
is, any gas at low pressure) and a measurement. uring temperature I've just described,
manometer M attached to the con- Now, with the gas thermometer the pressure of gas of mass M, consist-
tainer to indicate the pressure. If we we have no need to assigr 0 to the tem- ing of N molecules in volume V, is
put the container in melting ice, then peraturc of melting ice. We can calcu- proportional to its absolute tempera-
in boiling water, and measure the pres- late it. In fact, if temperature is di- ture 7. We can write equation (4) in
sure at these temperatures/ we find rectly proportional to pressure, then the form
that the pressure at the boiling pornt is PT P Prt
1.3661 times greater than the pressure i=,
'(l 'o
14) T T. (l
(s)
at the meltingpoint. So we have
Since the ratio on the left side equals which suggests that the ratio of pres-
f:
Y0
r.:oor. (2) 1.3661, this is the same as TlTo = sure to temperatlue of a gas at con-
1.3661, or T = 1.3661T0. Substituting stant volume is constant. On the
where p is theprcssue atthe temperatwe this value for 7in equation (3), we get other hand, we have equation (1) for
7 of boiling water and po is the pres- 1.3661 To-To= 100, which gives us gas pressure:

20 OUAlllIUll,l/ttlAY I ggO
find a length in inches, we need to the number of molecules in the con-
D:
2N=.l: know that 1 meter is about 40 inches. taineris = (M/p)Nr. So we can deter-
-tv Equation (7) is usually written in the mine the^fBoltzmann constant k if we
Substituting this value of p in equa- form know the m ass M,the molar mass pr,
tion (5), we get
,
wnere
u =10r. t8) the volume of the container V, and the
pressurep0 of the gas at temperature
r=? ry',, , (6)
, 1 VP't To
3VP0 (e) Such measurements have been per-
A NT 0
This equatron is true for a gas in'a formed many times, and all of them
closed container at constant volume. The coefficient k is called the grve the same value for the Boltzmann
So the number of gas molecules Nis "Boltzmann constant" after the Aus- constant:
constant/ and the ratio Tof psis also trian physicist Ludwig Boltzmann
constant, as we saw earlier. As a (1844-t9o6l. k = 1.38 . 10 23J/"K.
result, the coefficient at E in equation From equation (9)we can inJer how
(6) is a constant for any gas, and we to obtain the numerical value of the You see how small k is. So we con-
have Boltzmann constant. To do this, we clude that the mean kinetic energy of
1 r-t
T='-AE, l/) take a container of volume V and fill it chaotic motion of one molecule is a
^) with gas of mass M lthemass can be tiny quantity.
where A = NIV . Tolpo (a constant). determined by weighing). Then we It's precisely this energy that deter-
Ecluation (7) means that the absolute put the container in melting ice (whose mines temperature. At 1"K the mean
temperature of a gas is the mean ki- temperature To is 27 3.15'K) and meas- kinetic energy of a molecule E equals
netic energy of chaotic motion of one ure its pressure with a manometer. If
molecule, measured in degrees Kelvin we know the mass M, we can easily m?12 : 3. r.38. lo-23
2
and not in energy units (loules). The determine the number of molecules
coefficient Ais afactor that relates the N. In fact, if the molar weight of the = 2' 1.0-23 J/molecule.
energy and temperature units. The gas in grams equals p, the number of This equation can be viewed as the
situation is similar to the one we moles of gas in the container equals relation between one degree Kelvin
encounter when working with differ- Mlp". Andsince each mole contains and one joule/molecule.
ent units of length. For example, to \ molecules {Avogadro's number), Before we leave the subject of tem-
perature/ we should clarify the rela-
tionship between temperature and heag
281 440 227 s00 t\Mo concepts that have been consid-
ered virtually identical through the
centuries.
We know now thatheat is the en-
elSV o{ chaotic motion of particles and
400
212 .-q-, that
5/J .
this energy can be transferred
1
kom one body to another during heat
1

exchange. So heat isn't a quantity that


characterizes the state of a body-we
3A0 can't say it's contained in the body.
32 2 73 But temperature charucterues the state
of a body because it's determined by
the mean kinetic molecular energy.
There's no essential relation between
2A0 temperature and heat. We can only
say this: If two bodies hqve differcnt
temperatures, the body at the higher
temperature gives heat to the other.
* *279 *t7j AA
a Temperature is the quantity that de-
138 IUU ,.r*irr.. whether a btdy gives heat to
other bodies or gets it from them.
And that's lust how the great Scot-
tish physicist Iames Clerk Maxwell
( 183 1-1879) defined temperature way

OR oF 0 back when.

Figure 4 CONTINUED ON PAGE 49

21
OllIll|TUilI/IIATlJRt
HOW DO YOU FIGURE?

Challeltuo$ in ilysics and malh


Anchurian candidate & seven dwarfs, river raft & spaceship,
circles & cubes, expanding polygons & merging lines,
a pan of water & a string of polarized digits-
that is to say, something for everyone

tUlath
M6
Three equal circles. Three circles equal portions. The dwarf to his right
with the same radius z all pass through then does the same. The next dwarf
point H (fig. 1). Prove that the circle follows suit. And so they continue
passing through the points where pairs until the seventh dwarf pours out all
of circles intersect (that is, points A, B, his milk into the other six cups (divid-
and C) also has the same radius z. r gure r ing it into equal portions). Then it
turns out that each of the dwarfs has
M7 exactly as much milk as when they
Double parity. Prove that if each of All the voters are divided into eclual started. Find the amount of milk
the numbers x11 X2r ...r x, is equal to +1 groups, each group is again divided originaily contained rn each cup if the
or-1, andx, x2+ x2x3+ ... + xn_rxn+ xnxl into a certain number of equal smaller total amount was 3 liters. (V. Guten-
= 0, then n is divisible by 4. (A. Leon- groups/ then these smaller groups are macher)
tovich) divided into still smaller equal groups,
and so on. The smallest groups choose
M8 representatives-known as "electors. "
Physics
TWo similar polygons. When the sides These electors choose representatives
of a convex polygon are moved out- of the next larger groups/ and so on. P6
ward by the same distance 1, they fall Finally, the representatives of the larg- Imminent collision. The pilot of a
on the corresponding parallel sides of est groups elect the president. The spaceship moving at velocity y = 1
alarger similar polygon. Prove that election proceeds at all stages by ma- km/s notices an asteroid of diameter d
circles can be inscribed in these poly- jority vote-in a split vote, the opposi- = 7 km straight ahead at a distance of I
gons. (N. Vasilyev) tion wins. Miraflores has the right to = 8.5 km. The astronaut immediately
divide the electors into such groups as switches on the emergency engines,
M9
"Demouatic"
he wishes and can instruct his sup- which in a negligrbly small interval of
election. In the coun- porters how to Yote. Will he able to
l>

x
try of Anchuria, ruled by President organize such a "democratic election"
Miraflores, a presidential election is and get reelected? (32nd Moscow Math
u
coming around again. There are ex- Olympiad)
->
actly 20 million voters in the country
and only one percent of them (the M10
regular Anchurian army) supports Swen dwarfs. Seven dwarfs sit around
Miraflores. Naturally, Miraflores wants a circular table waiting for Snow White.
to get reelected, but on the other hand Each of them has a big cup, and some
he wants the elections to appear demo- of the cups contain milk. The first
ctatic. By "democratic election" Mi- dwarf pours out all his milk into the
raflores means the following. other six cups, dividing it into six Figure 2

22 ottAilrtll.,tit.lAY loso
time impart an additional velocity of
Av = 300 m/s to the ship. This addi-
ru
The cube's magneticfield. The current
P10
Mergingrails. Imagine you're down at
tional velocity can be directed in any l flowing in a circuit formed by the the railroad tracks, looking far off down
way the astronaut chooses. Can a four edges of a cube creates magnetic
a the line. Calculate the distance at
collision be avoided? (A. Andrianov) field of induction Bo in the center of which the rails seem to come together.
the cube (fig. 3). Find the value and {It's assumed you understand the physics
direction of the magnetic field created of the observed phenomenon and can
P7
on the river. A wooden raft is
Ra:t't in the center of the cube by the current estimate the other magnitudes invoived.)
pushed perpendicularly from a river- l flowing in the circuit formed by the (P. Zubkov)
bank such that its initial velocity is v. six edges displayed in figure 4. lM
The raft's traiectory is shown in figure Tslpin) SOLUI/ONS ON PAGE 59
2. The X on the traiectory shows
where the raft wili be at time t after the
operationbegan. Assuming that the
velocity of the current is constant and
equal to u, graphically plot where the
raft will be at times 2t,3t, 4t ... . I
Poterayko)

P8 TX
Coolingwater. Apan contains 1 liter
of water that can\be brought to a boil
by means of a 100-W heating element.
^W AI

How longwill it take for the water to D


cool loC when the heating element is
Figure 3 Figure 4
turned off? (A. Z:Iberman)

American,4ssociation of Ph,ysics Teachers


Attention, Students! The American Association of Physics Teachers (AAPT) offers student memberships
at substantially discounted rates. For only $29 you will receive all member benefits, including a subscription to
The Physics Teacher. AAPT is geared to all levels of physics education and has something for everyone!

. Internatiornl Physics Olympiad


Each year twenty students are selected from across the nation to participate in the International Physics
Olympiad. Five of those students travel overseas to compete with students from thirty other countries. This
year, the Ollmpiad will take place in The Netherlands.

. American-Soviet Exchange Program


Once again, AAPT is sponsoring a student exchange between Washington, DC and Moscow. Fifteen students
from the Soviet Union will visit and study here, while American students do the same in the Soviet Union. Both
groups will also meet together for ten days at the University of Maryland (ust ouside Washington, DC).

. Metrohgic High School Physics Contest


Over 22,0ff) students from 1200 different schools have entered the 1990 AAPTMetrologic Contest. The top
thirty scorers receive a laser for their school.

o Products
Members receive a 25 percent discount on any order over $20. This includes books, audiovisuals, posters,
T-shirts, and computer programs.

For more information about AAPT or a ftee products catalog, contact:


AAPT, 5112 Berwyn Road, College Parh MD 20740, (301)345-4200

23
OllAllJIUlt4/lt4AIII C1lAI.I.IlllGE
\

ilr{
d
I

/ \ \\.
\\,'

I
\',',
fi

/'t
AJ
The UBulnBIry 0l puptllalion UBltEIis$
The power of mathematical modeling is applied
to the phenomenon of hereditary change
from generation to generation

by l. M Yaglom

ATHEMATICS OCCUPIES our attention on acertain characteris- of the father's genes (and either of the
a privileged position among the tic, or "trait," transmitted from one mother's)has the same odds of being
sciences. Natural sciences such generation to the next and therefore passed on to the offspring. So a pair of
as physics or chemistrY studY called hereditary. The {ur color of black hybrid gurnea pigs can glve birth
the real world we live in; social sci- guinea pigs, for example, is such a to brown guinea pigs-in fact, one
ences study human society and its trait. The physical unit of inheritance quater of their offspring wi1l be brown,
features'. These sciences have in is the gene. A trait is determined by a if there are enough o{ them.
common the fact that they acqulre pair of genes, either gene can be one o{
their knowledge from experience. In two types/ G and g. The {irst one, Problem
1. Find the proportion of dominant, hy-
contrast/ mathematics studies such denotedby the capital letter G, gives
brid, and recessive offspring among a suffi-
abstractions as irrational numbers {for the dominant form of the characteris- ciently large number of descendants of each
example, 2hl and things like lines, tic (in our example, the color black); of six possible mating couples: \D ,D), lD,R),
which by definition have no width but the other, denoted by g, gives the re- lR,R), lH,D), \H,R), and \H,Hl.
only length. Mathematical knowl- cessive {orm (the color brown). The
edge proceeds from the construction words "dominant" and "recessive" BioloUiual Npttlatiolts altd Usllnetric
of abstract concepts. A mathematical mean that the outward appearance of
concept can be defined by grr.rng as is a hybrid guinea pig, H, with a genetic poillt$
done in geometry, a full list of axioms makeup determined by the combina- A bioiogical population is a suffi-
that adequately determine its proper- tion of genes (or "genotype") Gg, is the ciently large group of organisms that
ties. same as that of a dominant guinea pig belong to the same species and form
The outstanding role played by D, with the genotype GG-that is, mating couples only within the group.
mathematics is largely due to the fact hybrid guinea pigs are black. In con- Population genetics studies the changes,
that it
can be applied to both the trast/ recessive guinea pigs, R, with from one generation to the next, of the
natural and social sciences-that it the genotype gg, look different {rom proportions d, h, and r, which repre-
can, in fact, "modeI" physical and the domrnant and hybrid ones-they're sent the numbers of dominant, hy-
human phenomena. Mathematical brown. brid, and recessive organisms relative
modelirrg consists of establishing a The law o{ transmission of traits to the total population. So we can de-
correspondence between specific fea- can be stated as {ollows: an offspring scribe the population P by three non-
tures of a phenomenon by using mathe- receives one of its genes from the negative numbers ld,h,r) that are sub-
matical methods. We'll use the ex- father and another, independently of ject to the constraint
ample of heredity-more precisely, the first one, frorl the mother; either
o population genetics-to illustrate this
I The
d+h+r=1.
6
unique power of mathematics. Austrian natural scientist
Gregor Johann Mendel I1822-1884), the
abbot of a monastery in what is now
This simple observation suggests that
E tterediry Czechoslovakia, discovered the we use the following device for the
! Following the classical theory pro- fundamental laws of heredity that mathematical modeling of biological
posed by Gregor Mendel,l we'll focus underlie the modem science of genetics populations.

25
OUAlllTUIIl/IIATlJRI
and y are the d- andr-coordinates of the random choice of a parent; second,
the projection Po of point P onto side through the random choice of one of
DR. Consequently, populations that the genes. It's important that all genes
have a fixed gene pool composition in the gene pool of the entire popula-
givenby the ratio I : y {r * y : I ) corre- tion have the same opponinuty to par-
spond to points lying on the perpen- ticipate in the process so that the
dicular to side DR passing through proportion of descendants with the
point Po{f,0,1). second G gene is equal to the proportion
Now we've come to the main point f in the gene pool, while the propor-
of our discussion. Let's consider tion of g genes equals y. h the fust case
population P, correspondrng to point dominant organisms are formed, and -9
Pld,h,rl in triangle DHR, andpopula- in the second-hybrid ones. So the
tion P', which represents the next gen- tatios d', h', and t' of D, H, and R
Figure 1 ' eration of organisms bred by mating in
population P. Population P'also cor-
organisms in the next generation are t
If the side of tiangJe DHR is equal to 2a
(a : altitude of the triangle
1l?t: since the
responds to a point in triangle DHR. d,:f -d+h12,
is equal to 1), its area is given by the So we have a transfortnation of point P ltt - ., - l. l) - ,
ecptation So* : 3: Snr,, - So.,- + S*. = ad r':0,
+ ah+ ar. Therefore,d- h- r= l.'
H Now let's consider the structure of
Let's consider an equilateral tri- the transformation /,: P->P'of triangle
angle DHR whose altitude is equal to DHR that is generated by the equa-
1 . It's important that, for any point P tions given above. Comparing the
of the triangle, the sum of the dis- coordinates of pornts P'll, \,0) and
tances from that point to sides HR,
KD, DH is constant and equal to 1 (fig.
1). We'll call the numbers d, h, t
coordinates of the point P with respect
to triangle DHR andwrite P = Pld,hSl,
as for Cartesian coordinates. Their
relation to the usual three-dimensional
\
a-
ffi t\
P0(f, 0, y) (notice that Po is the prolec-
tion of P onto DR-see figure 3), we
see that point P' lies on side DH at a
:
distance of D P' DP ofromvertex D.
Let P, be the intersection point of lines
PoP and DH-that is, the projection o{
P onto line DH in the direction per-
coordinates in space is illustrated in pendicular to DR. (In what follows
figure 2. we'11 simply say "in direction h.") k's
In this way we can assign to each Figure 2 evident that DP, = zDPo, or DP' =
population P-determined by the ra- Pc4tulatictn Plclh,rl utrteslxntcls to point DP 112. In other words, to get point P'
ttos d,h, and r of the dominan! hybrid, P in sltoce yvitlt cctordlnate s (x.y,z) : = /,{P) we first have to find point P,i
and recessive organisms-a point in (213) d,hfl . N
tltese polttts bdong tct which is the projection of P in direc-
triangle DHR, which we'1ldenote by eclttilateral tilangle DF{R. r.r.hich is rhe tion h onto DH, ard then find the mid-
j111p1'.1'6ljo11 ol =,) 7t
)- z
the same letter P : Pld,h,r).It's worth 1t1,117,'x-
v.ith the pctsitive octdnt (x, 0, y > 0, z point P' of line segment P,D.
noticing that populations consisting , 0), Tlte clistunces ficutt poittt P to tlte And so the inherited transforma-
only of dominant, hybrid, and reces- sides of tritutfle DHR Lte equal to cl,h, tionf, we've been examining in this
si.ve organisms correspond to points
D(l,O,Ol, Hl},l,O), and R(0,0,1), respec-
tively. into poir-rt P', which provides a rn;rthe-
Problem
matical framework for studying the
2. Find the distance between points P(d,h,z) transrnission of inheritance frorr one
ar,d Prld'h'rr) by means o{ their coordinates gcnelation to thc ncxt.
with respect to triangle DHR (use iigure 2).
txamples
The heredity of a population is de- Interbreeding with dominant or-
termined by the composition of its ganisms. Let's suppose that organ-
total "gene poo1. " We can visualize isms in population P = Pld,h,r) inter-
this by looking at our triangle. Let's breed only with the dominant ones,
denote the relative portions of G and g D, so that one of the genes of every
genesby f and y = I - f fora certain descendant will be dominant. The
population corresponding to point second gene, inherited from parents
Pld,h,4 in the triangle. It's easy to see belonging to population P, is chosen
thatf = d+h12, y =r +hf 2, andthat f in two steps, in effect: first, through Figure 3

20 OUAlllTUlt4/ltllAY 1 SSO
P|fl21/, ,y l2) mdPo{f, 0, y), we see that the frequency oi Gg and gG pairs {the
point P' is obtained from P by the proportion -h' of hybrids) is 2fy; and
similitude with center H and scale the ftequency of g pairs (the proportion
factor Yz (fig. a). This means the z' of recessive organisms) is y2. So we
transformation /, that turns P into P' have
can be seen as a combination of ( 1) the
projection in drection h onto the line d'=12=ld+hl2)2,
joining midpoints M and N of sides h' : Zty :Zld + hl2l lh12 + r),
DH ard HR of our triangle and {2) the .t _ ^,)_lLt) , ")).
similitude with its center at midpoint
l 7 of MN and a scale tactor of Vz. (Note that d' + h' + r' = (l + y)2 = 1.)
Looking at the construction given These formulas express one of the
above, we can easily see that the he- principal laws of population genetics,
( reditary transformation f , changes the so-called Hardy-Weinberg law,
Figure 4 triangle DHR (the set of all popula- which appears in every modern text-
tions) into line segment MN (the set of book on the subject. They describe
,""tio, i, the result of two transfor- all populations exactly half of which how the composition of a population
mations performed sequentially: first, are hybrids) and possesses a unique changes when there is panmixia-
the parallel projection in direction fi fixed point Tl/4, y2,, %) (a stable populanon that is, when there are no additional
onto line DH; second, the similitude that preserves its composition). The factors that may influence the forma-
("homothety") with center D and scale sequenceof generations P', P" = f2lP'1, tion of rnating couples. {It should be
facarVz. ... is depicted as points of line MN; pointed out that the male and female
From this we can infer that the each point is transformed into the portions of a population are assumed
transformation turns the entire tri- next by the similitude with center 7 to have ec1ual shares of the dominant,
angle DHR (the set of all possible and scale factor 1/2. There's no need by hybrid, and recessive types.)
populations) into one line segment- now to make any projections. Conse- Now let's look at the geometrical
side DH (the set of all populations that quendy, populations P', P", ... approach meaning of the transformation / de-
don't contain recessive organisms). It's the stable population 7. scribed above-that is, the
important that the only point that Hardy-Weinberg law. First of all, we
Problem
! doesn't move (the "fixed point" of the see that the composition of the gene
3. Letf ,,r,,be the transformation corre-
transformation) is point D, which is sponding to the interbreeding of population
pools oi populations P and P'is the
what we'd expect from our under- Pld,h,rl with a fixed population Pla,b,c). lHere same. This clearly follows from the
I standrng of genetics. Generations that a,b,c, 0 and a + b + c= 1, of course.) For equations
follow P'as descendants of the origi- example, f ,* arrd f oro are the transformations
nal populaaon will correspond to images f , and f ,discussed earlier. Find the formulas l' = d' +h' =f2 + ly =f(f +y) = f.
12
for the transformation /,,,. and illustrate its
of pornt P' after sequential similitudes geometrical meaning. Also, find the stable
with center D and scaie f actorYz, and populations. What happens when multlple As we know, this means that points P
they'll tend to come closer and closer repetitions of transforrnation {,,,. are per- arrd P' lie on the perpendicular to DR.
to the pure dominant population D. formed? Let's assum eDPo= x.In that case,

Interbreeding with hybrids. Now Panmixia and lhe ]lat'dy-Weinheru law


'/ : r'rr : -t sin 60"
suppose the organisms in population The hereditary transformations con- 2
P = Pld,hsl interbreed only with the sidered above are primarily of interest so that
hybrids H. Using arguments similar to teachers. Scientists in the field of
to those given above, we infer that haU population genetics mainly study he- h' - 2ty :2(r-y)y : J3.(r-.+)
the descendants acquire the G gene reditary transformations P' = /{P) such
from their parents and the other half that the population of descendants is In an orfinary (Cartesian) system o{
the g gene; then in either half the generated by a random choice of mat- coordinates, this equation gives the
proportion of descendants having G as rng couples. This is called "panmixia" arc fI of a parabola when 0 < x < DR =
the second gene equals l, and those (from the Greek words pan, " all," and 2 I 3'\Iig. 5 ). The equation for a parali-

having I as the second gene is y. So Ior mixis, "mixing" ot"rnattng" ). Obvi- ola in our "coordinates relative to tri-
the next generation P'ld',h',r') we have ously a random choice of mating couples angle DHR" reads -h2 = 4dr, since h'2 :
and then of genes actually amounts to 4l'y'= 4d'r' . It's not hard to see that
d'=f12=dlZ+h14, a random choice of gene pairs from the the parabola passes through points D,
h'=f 12+y12:112, gene pool of population P. Then the R, and 7(the midpoint of line segment
r'=y12:rl2+h14. frequency ol GG pairs-that is, the MN) and is tangential to lines DH,
proportion d' of dominant organisms RH, and MN at these points.
Comparing the coordinates of pornts in the next generation P'-equals f2; Point P'=flP) is where the perpen-

OUAlllIUllil/FIATURt
2t
=0 (the total extinction of hybrids) and k : blindness an exclusively male illness.
infinity (the total extinction of D and R Statistics do, however, tell us that 8"/"
organisms)i
of men suffer from Daltonism while
only0.5% of womendo. TheAmeri-
The Hardy-Weinberg law was found can biologist T.G. Morgan explained
in 1908, independently, by a young the difference this way.
Engiish mathematician, G.H. Hardy, The geneforDaltonism, whose domr-
working at Cambridge, and an Aus- nant variant ensures normal vision
trian physician, Wilhelm Weinberg, while the recessive produces color
in Vienna. Historians of science even- bhndness, is contained in the so-calIed
tually discovered that the law had X-chromosome. This chromosome, a
been stated five years earlier, in a together with the Y-chromosome, de-
somewhat di{ferent form, by an Eng- termines a person's sex (as is the case
lish biologist, W.E. Castle, but the with other mammals and some in-
Figure 5 work had gone unnoticed. Curiously sects). The cells in a woman's body
enough, Hardy, an eminent specialist contain two X-chromosorrres; a lTran's
dicular to DR drawn from P intersects in number theory, was a partisan of cells contain one X- and one Y-chro-
the parabola, so that the transforma- pure science. In his largely autobio- mosome. An individual inherits one
tion f is the proiection of triangle DHR graphical book, A Mathematician's chromosome from the mother (the X-
on arc fI of the parabola in direction h. Apology, he strongiy supports the cause chromosome, of course) and another,
From this it follows that all the of pure mathematics against applied which determines the sex, from the
points of arc II are fixed-that is, all science, which he considered (perhaps father. So a man receives the unique
populations P(d,h,rl for which h1 = 4dr without sufficient reason) boring. Not gene for Daltonism (or any other char-
are stal:le with respect to transforma- surprisingly, Hardy's excellent calcu- acteristic related to sex-that is, re-
tion /. This result seems paradoxrcal. lus textbook is called A Course of Pwe lated to the X-chromosome) from his
It requires that, for a large enough Mathematic.s. Nonetheless, now more mother. A wornan receives one gene
population in the absence of muta- than9O"h of all references to Hardy in {rom her father and one gene from her
tions and selection (natural or artifi- scientific books and articles have to mother.
cial), evolution always proceeds ln do with his paper on population genet- Now let's consider population P
one step. Any further interbreeding ics, which he wrote in his youth, and and its series of descendant popula-
doesn't change the composition oi the not his brilliant papers on nurnber tions P', P" , ..., generated by random
gene pool: iI P' = flPl, P" = f(P'\, and so theory. mating {panmixia). We want to study
orl then P' = P" = ... . But in real h{e the the dynamics of the genetic srl.tlcrure
ideal conditions of interbreeding that A lew remat'ks on [altonism of these populations. In particular, we
we have assumed never hold-we can In the simple genetic scheme we've want to determine the proportion of
expect only some crude approxima- been using atrattthat can exist in two recessive women and men with the
tions of them. forms is determined by a pair of genes, genotypes gg and g-that is, the per-
So we see that the Hardy-Weinberg either of which can be o{ two types. sons of each sex who suffer from Dal-
law only gives an initial approxima- But the number of genes, as well as the tonism.
tion of the real processes that take number of their types and laws deter- Three female genot),pes-GG, Gg,
place in biological populations. Fur- mining their outward expression, may gg-and two male genotypes-G, g-
ther approximations are given by more be qute different. Let's look at a more are possible. We'll denote their pro-
sophisticated mathematical models sophisticated system for the trans- portions in the female and male sub-
(see, for example, problem 4). Obvi- mission of inherited traits. sets of population P as d,h,r,6, and p;
ously, we need to compare the results Some hereditary diseases are much
obtained by these models with obser- more corrunonin menthan inwomen.
vations of living nature. These include color blindness (an ina-
bility to distinguish certain colors,
Problem usually red and green) and hemophilia
4. Let mating couples in population P be
{a deficiency in the normal coagula-
formed randomly (as with panrnixia), but
assume that the hybrids have less chance o{
tion of blood, which results in pro-
survival than the doilinant or recessive or- longed bleeding after even minor inju-
ganisms. More precisely, only the kth part oi ries). Color blindness was first de- R(0,0,0)
the total number of hybrids of the next gen- scribed by the eminent English chem-
eration surwives. (For k > 1 this means, on the
ist |ohn Daltorr(1766-1844), and the D(1,0,0)
other hand, that only the l/kth part of D and
R organisms survives.) Construct the corre-
condition came to be called "Dalton- I
sponding transformation of triangle DHR. i.sm."
Figure 6
What's the nature of the transfonnation for k Dalton mistakenly considered color

28
OUAlllIUll,l/llllAY 1 SSO
(see figure 6). Consequently, points ancl r\ - 2I- : r' intersect surf;rcc ,S along
6
P ', P " ,... are uniquely defined by their straight lines, r'r,hereas thc rntcrscctitxs of 5
R, ancl a11 other plrnes para1lc1 to axis 7r arc
projections P6', Ps" ,... onto plane ft : 0.
p:rr:rbolas.
We can infer from the equations f '= (f 7. Find thc coorclinates oi the lirnit
,rFlr-j'ffi, + 6)i2, 6'= f, andfigtue 7 that segment popnlattot-t f'' [r1, 11.,trrt of the coorclir-r:rtes
rrr'nt t
Po%, ir parallel to line I definedby 6 = ( I-, c\, fi ) of thc initial population 1r.

and intersects diagonal RD, of 8. Lct a popnlation P(t, i\,7r1 bc t-rtcrbrcd


-21, rvitl-r a iixed popul.rtion l),,(f,., b,, 7r .) :rs fo1-
face RDD,R, of the prism so that the
I.r,r .1 1'111.1]. 1,,.1 1,,,111.1 I r r rl l.r rr' r';rtr,lotrrll
1

intersection pornt divides the segment couplc'cl t,ith r-nales frot-n |,


lvhile male
r+d in the ratio of 2:1. We'll call the trans- individr-rals oi 1'are ranLlornly couplcd r,ith
2 fonnation that tums Po into Po' a"shear ieurales irom P,,. Wc'11 dcr-rotc the c'lescen-
compression" with a scale factor of dant pr4rulation by P, : t,(1']1.
Figure 7 (a1 Frnclthe coorclnates f., b., l. andall
- 72. It's not hard to see that the points thc rcst of thc parar-nctcrs of poptilation P irs
the proportions of genes G and g in the Ps', Ps" ,... obtained from Po by repeat- Iun(lt,rtt\ ol t]tt c,,,,rrlitt;ttc. (,l l.llt,l I.
gene pool of the female subset of P are ing the transformation belong to line 1b) Lct
f and'y. The corresponding quantities PrPo' andapproach point Po-, which is P. = f\tlP ). r'. = f(P.). ... P, = l,!-
p,, .

for population P' are given by the where this line intersects RD,. Point
frncl thc coorclinatcs oi thc popnlatron -P,,, rrs
equations Po- and all the points of segment RD,
functions oi thc coorclir-r:rtes of P ancl 1),,.
are fixed during our transformations. icl Exarnine specific c:rses that corre-
d' =16, h' =lp+y5, r' = y p, "Liting" points from plane -h = 0 onto sponcl to r,rrrioLrs con-rpositions oi the gene
6,=f,p,=y; surface S, we obtain the following pool 1', for cxarrp.ls, f, : P i 1, 1 ,01, u.hrch is
f '= (f + 6ll2,y' : ly * pl12. statement: the composition of subse- the pure chmin:rnt popul:rtion P,, as n cll as
the casc u,herc popnlation P is onc of thc
quent generations of population P tends
rlanv 1.ro1ulations that arc stablc rclatirrc tcr
(Can you explain why?) to approach (very quicldy) that of stable the transiormrtron tlescribecl in thrs article.
We'll use a geometric model to population P"; the cortesponding point, ldl Describc thc n-raps 1,,: 1) >2 and /-,,:
study the transformation P->P'. Ow- like all points coresponding to stalale P >P ,, as tr;lnstonnirtions oi thc pnsm. Q
ingto the constraint d + h + r= 5 + p = populatrons, belongs to the line of
I.M. Yaglorr is tt tlctctctt' r.ti plty5j6al 4pi
f + y = 1, we can express all the parame- intersection of surface S and plane I =
rn ath ent ttti t: tr 1 sclcrrcl s.
ters determining the composition of 6. (Show that this llne is aparabola! )
the population by means of tfuee para- For any stable population I = 6 we
meters-say, l, 6, and l. So to each can find all other quantities determin-
popula+ion we can assign the point ing the composition of the gene pool: lftl]tal's happeninU?
given by the Cartesian coordinates (1, SL..rnrner stuciy . comcetlrons ner,v books

6, 1i). Since the inequalities p = y = 1 - 6, d = 62, h :26p, r = pz. . . ongo ng act v t es


c uDs and assocra-
t o.rs f ree samp es .
contesis urtrat-
ever ts f yor tr nk t s of nterest to
0<l<1, In particular, for p = 0.08 we get I = Qttantum readers. et us Knovr aboLrt ll
0<6<I, 0.006. These values are roughly the He p us fHappenings and tre Bu iet n
0 <h <2r, same as the ones given above for the Board \,! tn shoi't ner,'rs items frrsthand reporls
and anno!ncerxents of upcon';rg events
h.2y = 2(1-r) frequency of Daltonism among men
and women.
must be satisfied {the last two because tftl]tatt ott yottt' lniltd?
of the constraint hlz = f - d : y - r), A linalwond lvrte to !sl We \,,/art to knolv,lhat you trink
ol Qtanlun. What do you ke the most?
these pornts iie in the triangular prism In this article I've tried to show how
What,r,,ou d you ke to see more of" And
DHRD rH 1R, bounded by the planes 6 geometrical methods illustrate prob- yes ,rhat don t you ke about Oir.-,niiirn?
= 0, 6 = l, h = 0, h = 21, h = 2 -2t (hg. 6). lems in population genetics, enabling We \ilant to .nake t even better bri r,ie
The hereditary transformation changes us to solve them by elementary meth- need your ne p.
point P {f, 6, h) into point P' (l', 6', h'l ods and obtain important results. I
such that point P' and all subsequent hope you now feel capable of produc' lIUltal's our addre$s?
points P" , P"' , ... belong to surface S, ing similar arguments and solving the
problems given below. Quantum
defined by the ecluation
17 42 Connecticut Avenue NW
h=2I-4f5+26. Problems Washington, DC 20009-1 171
5. Let the scales along the axes h, l, 5 be
equd,to 3"'12, 1, 1, so that the prism DR ... Hl
This fact follows from the equation is regular. Prove that the distances from Becomealactorinlhs
point P(f, 6, h) to the bases of the prism are
h'=t(t -6)+ (1-r)6
=f+6-2f6
equal to 6 and p, and that the distances to the
lateral faces are proportional to the values d,
h, t.
OUANTUM eqmtio]l!
=21'-41'6'+26'2 6. Prove that the planes 5 = c (a constant)

2g
OUAlllIUl\ll/IIAIUBI
IN YOUR HEAD

Ballpark eslimales
How to impress your date and amaze your friends 1
with offlhe-cuff answers to questions of magnitude

by David Halliday

OME PROBLEMSOF PHYSICS Ifby " accurate" you mean an an- The volume wom per tum is then
involve calculations of the high- swer good to three sigrrificant figures,
est possible precision. Many prob- you're right. But in a problem of this ,, _v _(4E)w!_(2nR):Wh
u'=,v= = t _40R:wh-
= t
lems, however, call for only an ap- kind, "accurate" means "within a u.2"R
proximate answer. Physicists pride factor of ten either way"-that is, Notice that we've replaced nzby 10,
themselves on being able to solve such over or under. Actua11y, it's hard to be which is certainly close enough for
"order-of-magnitude problems" quicldy that far wrong in estimating the input our purposes.
by breaking them down into their com- data. But there's no need to replace n2 by
ponents and makrng appropriate com- I get it, Where do we start! 10, My calculator shows 9.87,
mon-sense estimates. We start with a plan. We'I1 esti- You might feel that you're improv-
Here's a tlpical problem: mate the volume of rubber wom from ing the precision of our answer by
the tire and then divide by the vo1- doing that, but you're not. Our other
On average, how many atoms ume of an atom. That will give us our estimates will be so approximate that
of rubber are wom from an answer. Let's deal with the tire fust. such precision is mrsplaced. Not-on1y
automobile tire every time the Okay. But I don' t see any way to that, 10 is a much simplernumber to
wheel goes around? guess what volume of rubber is worn deal with.
from the tfue every time the wheel I accept that. What next!
Problems of this kind are often called goes around. We've aheady made great progress.
"Fermi problems" a{ter the great physi We can get an estimate by guess- We've reduced part of the problem to
cist Enrico Fermi, who was a great ing the volume of rubber wom during quantities we can estimate. We'l1 do
practitioner of the craft olproposing the life of the tire and then figuring that soon. Meanwhile, let's think
them and solving them cluickly and out how many revolutions the wheel about atoms.
cleverly. makes during that time. Dividing I've been wondefing about that,
No doubt you have a few questions. will give the volume of rubber lost per What is a "rubber atom," anywayl r
Yes, I do. Does this problemhave tum. I'm sureyouwon'tfind it in the peri-
any pr a ctic al s ignif ic anc e ! Let R be the outer radius of the tire, odic table!
Probably not. Although the prob- I4l thewidth of the tread, h the depth You're right, of course. Rubber is
lem is an interesting link between the I
o{ wear, and the distance traveled made up of long chain molecules formed
worlds of the very small {the atom) and during the life of the tire. The num- from carbon, hydrogen, and oxygen
the very large (the automobile), its real ber of tums N is the total distance atoms. We're interested here only in a
purpose is to help you understand how traveled dividedby the length of the sort of generic atom, whose radius we
to make estimates. tire's circumference: labe1 z.
But there arc no numbers. How can I see. Then the volumeY the
L ^of of
T
we even start! geneilc atom would be the volume ,J
We have to estimate the starting 2nR' a sphere of radius r, or (4x13)f . Right!
numbers-theradius of atirg the amount in which 2ruR is the circum{erence of You could say that. It's a little
of wear ... the tire. The volume of wom rubber better (and simpler) to put the volume
But that's iust guessing! How can V is the volume of a cylinder of thick- at(2rli-thatis, the cube o{ the diame-
wepossibly aruive at an accurate an- ness h: ter. That treats the atomsas little
swer! V = l1nR)Wh. cubes and makes some allowance for ?
30 otlAt'tTUt.l/nloil* rBoo
the empty space between them. Shalll workthis out onmy calcu- 2. How many gallons of gasoline
Now we divide to fud our answer. lator for you! are consumed each year in the United
Right! Nol It's a point o{ honor not to use States by private automobiles? O
Richt. The number of atoms wom a calculator when solving Fermi prob-
away per tum is lems. Let's rewrite this ecluation by
vr 4oR2wh 5R2wh collecting the integers and the powers Adapted from the forthcoming book
Essentials of Physics by David Halliday,
'" vo of ten:
Ler\3 Lr3 /5 .3 .3
, = [arr, '4'
*) . ,0,, .
Robert Resnick, and lohn Merrill with
permission o'f the publisher, lohn WiJey o)
Now we're ready for our estimates. Sons, Inc. David Halliday is professor
Let's take them one at a time: You can easily see that the number emeritus of physics at the University of
l in the parentheses is about 20, so that Pittsbwgh.
R (tire radius) = about I ft or30 cm n = 2 . 1018
atoms per turn.
or3/10 m, Shouldn't we round that o;t'f to L018
I W ltreadwidth)= about 4 in or 10 atoms per turn!
cmor 1/10m, Yes, indeed. The "2" isn't
fr (depth of tread wear) : about I /6 justified by the precision of
in or 4 mm or 4/1000 m,1
I (tire life) : about 50,000 mi or B .
our estimates.
So-
Lafge or srnarr?
107m, When someone asks the Do you consider the answer to the tire problem
r (radius of an atom) = about 10-10 "tire question" at aparty land (1018atoms/turn) large or small? No answer is pos-
m.2 it never fails to come up, be- sible untilyou've answered the necessary auxiliary
lieve me!), you can r,ow gaze question: Large or small relative to what? As a pure
In putting these numbers into the at the ceilingtor afew min- number, 1018 seems large. lt's 10,000,000 times
above expression for n, we must be utes andsay: "About... 1018 greater than the number of stars in the Milky Way gal-
careful to choose units consistently. atoms per tum/ more or less." axy, for example.
Using meters/ we find That's how quickly Fermi But the problem deals with 1018 atoms, not 1 018 as

5.3.3.4 himself solved problems like a pure number. This number of atoms is about
tt--.
this onel 10,000,000 times greater than the number of atoms
10. 10. 10' 1000 . 8. 107. 10-30
in a typical small bacterium but about 10,000,000
Try your hand at finding limes smaller than the number of atoms in a glass of
1
You might estimate the depth of uead ballpark estimates {or these water.
wear to be LlZ in (12 mm). If so, your Fermi problems. Our conclusion: You can only compare physical
calculations will be sLiglrtly different. That's 1. The population of Bos- quantities of the same kind. There are no absolute
okay-these are estimates.-Ed.
standards of "large" or "small."
2 ton in 1980 was about 56Q000.
Physicists always use this as an estimate
of the radius o{ an atom. It's a good number How many high school teach-
to know. (The radius of a nucleus, by the ers were there in that city in
way, is estimated to be 10 rs m.)-Ed. that year?

r" "I:*1
r \ilf
, i"' {,,j
:., t
iui i
;r'
*
lei
ii
// ,,"//
cl ,/ /
r J

t
.I r ^ l}l C
q
-o
r/ i C
i' f -I
o

q/\ Z
Vf ht c
i

OUAlllIUllll/OIPARTl|4IlllI
3t
i I - 9H .599 i 1:=oi
H * i!q :?
ijE
i
=x :3 ju EX :AEJ F ilH"E
Iv i E
li(J : A * E3
'.r. u HE !i+E
3,
YE;EE-
-A i-i
3 =U!+ (/ CS
FE SE

H ; ; &I
=.voa*
j'
.9 =E .E
;'9 FH
U'o
E: =
.2
F --d$
6.
g
E
:E
i'r Ei;-+- msr[;g
E nEEE g:E E *
H
X € =EE
E
y e f E= : €EE
tI- iA F 'D
E; T i ;Ft.; ;3
P; .l:aiAE+!ilE H

'P tr ts o !.1 E
I;!9E
-b E ?,tti
F uEP
!j3v+
E
E
o. i€
6 -Eq qt u ct tr': 3 =
i
',1
€ 8EEE E;EEEHH tf
' a l.+=
= -=;tEjEgr;tet E u i''l ts: F ,.
H
iE *E E3E+
=rf r dEEpt€'a Els iIGE € r;E4 s
C\. 8Fn€?6r
Io E o-.r=.jil ie?=nEt
5
H -o

trsE:-ihE€ EE.EaEI:
il sE3f
n
>:!

s €a.) FilSH;€
l-l'iX6:H
EgTHEU;
=;nHtF-
'Jq,ion-.-iG-rqi
,IdGo*So
II (/) .i .t e F - d E 9 - u yia-, dv,qi

tr
I
a)
f, vt3E ;;;E€=
'9.€IHx5. EEAE€bE
E:j-l€A
U
t-- O =fr
Ei
*EE$f =?EPsaB
€E;7i,3'ii!Efrr4?
LU tF a)
=x r*!saiztnii'1 {ii
o_ o €; E€^E f
cr >- xd >.xi H= >.; H cr.'-l >.xI
El€^ H€ F-G 3E€
o 1-
-, E
.=E iEsEEn:;E Ei:YEE
O E-4-r€*;+*; E i-!*i iia;
*
a tr
ar
- o
=
(- :,
HU!'!'U tr q > i: \\
o
-.i
- '=
U
o c5
-
-
o
=
(- sJEfiuqXSH
&.
tx r<
c.)
Q<T H?
Ji
5E g H='a?3,-E
C.= I E u ES
J C=T f, >v
AF ^
H 6r -
-
!
=:
i v P

tF o vJH:--lu^\d

Y C=T
t- q)
-F(U
:a
^.jq
E
:
io;
v'i
zii
i-lju*4.'==E=
i r;:E
zdidFsEE-;
E
a 6 .i-i Y
F S gEZ
U
E IE U
-.C ^.;
\J >x .S.r:
6,u
-

cn
; r\
>V(JU
it-l:-^=! 3rI
sd' 9.2

3
H;eiaEHq;
Xtr9''=-E'=E
-- f, i C Hi€E Etr g
o :!oo+:
.{--.i
J-U\,
g*Flr.= 8ij .ai ;:rrl*&E3
- ! - r .,
-llEo'"-E-cE
-
: - d U .l
a) L d= o 3 !4 9-c
\ q
HUHV
H
-\+
H QO-
u
kEe;XiFE;
{ ^'H^-i

!
o..i O -X
.ilv.i'I
^. ! o-' =faF=U--cE
.
H
v u
.-.r->^.].==
J S 5 t ; EE fr .r L
d.-

H-HE::J - -Lh-:-JG3
- i.;
-.

.b E; ;r !
H*HO!U h L r,6
=rt
G F
eE!
9.3- E x €F'aE
H H 6E Ee=
C)
tr q,F c-Q
-
g Eb'!eEHtiIr
trii=: .g€
r-
tr_Lr X (d IV.:E=.I.9
iH€. i !; EI
r;9 lir
FEE:qEyff,8-"
t!Y!

r+r *= E
'io4 \ C.=
$.v
tjrE
7 C !- d;: I 6i
C y y, or O
(Jlr
d-
g
g;
'
=
4.:le!r\-

u epeYFq11x=
F+d UJ
io Hl ts]? t Q E -t ^Y 9=
I H * EE E X HE J
i- _= = -:l
EsE:rriE*:sEAEEegc*g ;+fSsci r:;:Es:
Et g ;ie ii A#r:; E*gs E $,* ,e EEEii r E u u i,.a
t ti * i 1+i l[i*s:€ *e iga !E EEs :eE I:E gii:
u
E u
IEEEEEi;:f
U:E: n:tE;EfEi+: #€FiEtEE €€EtE+s
s:'* }E ilE!E nE*E; i IiEi:;$ tE EF$EE
E= R
; H=
U
gE p

6'a;r*:;SEE!!EE *.Ei I iE rirE; EE€i g^:


B
CTEI
-s::E t
8.6 E --H 9.
F
=
?,)
Hg;BlEPEE H+ Ej * Eg:
i=: *
e, -dff4jt.ooc.9i.alEo-F9E-ij.z:i:q:4k.iEs,^c,.]EEU1o:<
r t raz*, .='t EE E
: x'; A

=i
:*;EEiE E ISEE€i$E €=u
[I gg;gEI.E
1
4
eE$E 3H E EEE

aou.{ry ,{r1ug {q YY
CONTEST

When days ore monlhs


"Then the tides gradually pushed her
far away: the tides that the Moon herself causes in the
Earth's waters

by Arthur Eisenkraft and Larry D. Kirkpatrick

I T SEEMS PARADOXICAL THAT the Earth's tides. As the water moves aheady occurs in our solar system.
I the Moon's attractive force causes to fom the two bulges that we experi- Pluto and its moon Charonhave this
I ,, ,o move lartner lrom tne Eartn, ence as two high tides each day, there's relationship. They move as if they
I br, ,hr,, exactly what happens. a lot of friction of the water with the were two balls on the end of a massless
The distance between the center of ocean floor and with itself. There are baton-each always has the same face
the Earth and the center of the Moon also tides in the "solid" earth. These toward the other. Pluto's rotational
increases by about 3 to 4 cm per year. result in a conversion of some of the period is ec1ua1 to the revolutionary
No doubt you're aware that the Earth's rotational kinetic energy into period of Charon-6.4 Earth days.
Moon's gravitational {orce produces heat at the rate of about 2 billion This leads us to our contest prob-
horsepower (1.5 trillion loules) each lem: For the Earth-Moon system/
second. how long will an Earth day be when
This may seem like a lot oi energy, it's the same as a month? Let's make
but it's only a trny, trny fraction of. the a few simpli{ying assumptions. We'l1
total kinetic energy of the Earth. Sma11 neglect the revolution of the Earth and
as it is, though, this change is measur- Moon around the Sun, the motion of
able. In order to lose kinetic energy the Earth around the Earth-Moon cen-
the Earth must slow down, but not ter of mass, and the rotation of the
very much! The length of the day Moon on its axis. We'I1 assume that
increases by 1.6 milliseconds each the Moon's orbit is a circle in the plane
century. of the Earth's ecluator and that the
But how does this cause the Moon Earth and Moon are uni{orm spheres.
to move father away? This occurs Please send your solutions to Quan-
because the total angular momentum tum,7742 Connecticut Avenue NW,
of the Earth-Moon system must re- Washington, DC 20009. The best
main the same. The loss rn the Earth's solutions will be published in the next
angular momentum must ec1ual the issue of Quantum and their creators
gain in the Moon's angular momen- will receive free subscriptions to Quan-
tum. This requires an increase in the tum for one year. o
size of the Moon's orbit and a corre-
spon&ng increase in the length of the Arthr.u Eisenl<rak is the chab of the science
month-that is, the length of time it department and a physics teacher at Fox
Lane High School in Bedford, New York.
takes the Moon to go around the Earth
Larry D. Kir\patrick is a professor of physics
once. So both the lengh of the day and at Montana State Univercity in Bozeman.
the length of the month increase. At Drs. Eisenkr aft and Kirkp atick serv e as
some time in the fistant future, a day academic djrectors for the US Physics
on Earth will be a month long ! Team that cofipetes in the International
rFrom "The Distance of the Moon" in Physics Olympiad.
Cosmicomics by the Italian writer Italo Although this slrrchronism of day
Calvino 11923-1985). and month may seem very strange, it

34 oUAtrtIUtrt/tlaAY rsso
CONTEST

Al sirres and $EUEn$


curious phrase that means "in disorder,"
-a
but from chaos can come beauty, and order can lie
hidden in apparent randomness

by George Berzsenyi

o
6
I
c
6

)oo
c
-o
c
G
O
c

I N THE SECOND ROUNDOF (American Invitational Mathematics


I ,n" USe n U,h.matical Talent Search Examination) that was originally posed
I lsponsored jointly by Rose-Hulman by Professor Andy Liu of Canada. His
I Institute of Technology and CO- problem asked for the smallest integer
MAP) conducted in the pages oi the
quarterly Consortium, students were
@fu- multiple of 15 whose digits are
0 or B. If you know of a similar
either

asked to find the smallest positive problem, I'd appreciate a note describ-
integer multiples of 84, 88, and 89 that ing where you came across it.
could be expressed in base 10 using So here's our challenge to you: Is Please sendyour solutions to Quan-
the digrts 5 andT only. Their response the Roseberry Coniecture fiuelor all tum,1742 Connecticut Avenue NW,
was most encouraging. Nationwide, integers that are not multiples of 5? Washington, DC 20009. The best
over 150 students found the answers- If you don't yet feel strong enough solutions willbe published in the next
7 677 6, 677 6, ar:d 557 7 67, respecavely- mathematically to tackle this prob- issue of Quantumand their creators
and many provided beautiful analyses lem, maybe you can turn on your will receive free subscriptions to Quan-
leading to their solutions. computers and-after devising effi- tumforoneyear. O
Mark Roseberryt ahigh school sen- cient search algorithms-gather fur- George Berzsenyi rs the chairman of the
ior from Kentucky, also provided a ther evidence in favor of the claim. Or, Department of Mathematics at Rose-
computer printout showing that all alternatively, you may wish to ad- Hulman Institute of Technology. He was
positive integers less than or equal to dress similar problems using digits on the committee in charge of the USA
224that are not multrples of 5 have an other than 6 and7 . Matlpmatical Olympiad for 1 1 years, chaired
the committee in charge of the AIME for 6
integer multiple consisting of 6's and The first problem I mentioned is years, and is presently conducting the
7's only. based on a problem in the 1984 AIME t l S A Mathematical T alent S e ar ch.

0lJAilrlJlt|/c0irTr$r 35
LOOKING BACK

The $Bct'sl 0l the Uenel'ahle Coopet'

It was an age-old time-saving trrck everyone used,


but no one understood-that is, until
a certain mathematrcian's curiosity was piqued

by M.B. Balk

OFTANNES KEPLER ( 1 571-1 630), hole, called the "bunghole," on its But why he proposed it just this way-
the court mathematician of the side right at the middle (that is, its I'm sure I can't tell you. "
Austrian emperor Matthew I and fattest part). The fellow would just
a famous astronomer, was ob- shove a brass measuring stick diago- Kepler recounts how he deciphered
serving with curiosity and admiration nally into the filled barel's bunghole the secret of Austrian cooper in his
as a young vintner cailed out, easily until it hit the lower edge of the bot- bool< The New Stereometryl of Wine
and cluicldy, the capacities of a succes- tom cover (fig. 1). Then he'd simply Barrels, Mostly Austrian, As Having
sion of wine barrels o{ different sizes. read o{f the number on the mler where the Most Advantageous Shape, and
Kepler recalied the time-consuming it stuck out of the hole. The barrels the Remarkably Convenient Use of
measuring procedure used in the vine- were all different-large and sma11, the Cubic Ruler With Them, With an
yards of the Rhine country. The work- "potbellied" and "skinny"-but this Addendum on Archimedean Stere-
ers there would painstakingly fill each didn't worry the fellow at all. He'd ometly. Although this book was pub-
barrel using a container called an "am- give the answer each time with the lished nearly 375 years ago, it's still
phota," which served as a standard same speed and confidence. An un- cluite instructive today, as we shal1
unit of volume. They'd count the kind thought occurred to Kepler: "He's see.
number of amphoras the barrel held pulling a fast one!// Kepler first considered the case of a
and then bum the number into its side "Don't worry/ Honorable Mathe- cylindrical barrel (fig.2). Suppose ND
with a red-hot iron so that-God for- matician of His Majesty," theyoung = ),, the length of the generatrixAB is
bid!-they wouldn't have to repeat man said, as if reading Kepler's mind. 2x, and the diameter of the barrel's
this boring procedure for that particu- "This method of measurement is bottomADis2y. Then
1ar bar:rel. sanctioned here in Linzby the city
But here in Austria-what a differ- authorities, and the cooper's guild x2 + 4Y) =)vz, (1)

ence! If you've never seen abaruel vouches for the precision of its re- and the barrel's capacity can be found
close up, you might not notice it has a sults".
"For anybaruel?"
"I don't know about any batel, btt
for all banels here in Austria for sure,"
joked the vintner.
"But how can you be sure this method
is correct?"
"What I don't know, I don't know.
I won't lie to you. People say that
years andyears ago theVenerable Cooper Figure 2
lived herg and he proposed this method. tThe old name for space geometry-
Figure 1
Ed,

38 [ttANrtlttl/trtAY 1 sso
by applying the formula was somervhat off the mark (not t = t,,,
that is, but f > r0 or I < t,,), the vintner
v =rhn ()'r -rr)r 12)
will not suffer because of the miscal-
It's easy to express this capacity in culation: the actual capacity vof the
terms of the distance ), and the ratio I barrel will necessarily be Jess than q,. ,./
of the generatrix to the diameter of the And so the vintner will declare the
bottom lt = xly\. Ecluations (lland 12) capacity vo and be paid accordrnglv by
imply the client, but in fact wili give the \
client slightly less wine, namely v.
v-Zil.1,4+l)'r. il\ "BLrt thcrr, no self-respccting,
"These formulas show," argues
Kepler, "that the capacity of a cylin-
clrical barrel does not deper-rd only on
the distance i. To be able to usc the
Austrian cooper's method we nlust
deal only with barrels lvhose ratio / is chfier as little as pos- i
flxed 1r = ro). What is the best choice {or sible fron-r the volume
thrs ratio? What is thc rlost profitable n corresponcling to
choice for thc ratio ot thc gcncratrix's the best value of ,. ,
length to the diameter of the barrel's the parameter
bottorll Let's argue from the vint-
ner's poilt of view. His interest hes tn
the choice, arrong all values of t, of
the value t: Iu such that, of all bar-
rels with distance ND equal to
7., the one chosen has the nraxi-
mum volurne v,,. Then he I
can calculate this volurne
accordrng to equation (3).
And if the craftsman .,tr,'{l'
who made the bar- ':, ',. "' sake, one that has a
re1 did not succeed ;.' first clerivative for all

ixii*ti'i:J'"
e)octb,llut
\i \
,,',::' r > 0); and suppose that Io
is some fixed value of the
rndependent variable t. Then,
for a small increment fi-that is,
t = to + -h, we have

f (t) * f (to) = f' lt u) ]t + slh\ . h,

where cr(h)->O as h->0 (see any cal-


culus textbook).
So ii /'(t,,) t 0, the deviation of //tJ
fuon f(tr) will be, for small values of h,
67 practically proportional to h. But il to
lltlt.,,,i,, (namely, I,,). The buyer's inter- is a maximum (or rninimura) point of
C
1,1,f est in this is even stronger. It the given function, then/'(ro) : 0 and
:o
=
";liir tums ol-rt that this state o{ affairs rs the deviation equals alhj . h, where
o
o
o ,'.1 ,i.: t: also achieved by choosing the a(fi)->O as h->0. This rneans that
a
, ,.-,il! parameter t that yielcls the greatest near the extreme value t0, a sma1l de-
I .,:.'
''
value for the b:rrrel's volume v." viation o{ parameter t from to affects
ri."" Since we are familiar with the no- the variation of the function l/tl much
tion of derivatives, we can easily ius- less than it does near some value of t
tify Kepler's reasoning. (It was much where "t''lt) *0. (Kepler calls the vari-
harder for Kepler-his book appeared ation "insignificant.") In our case-
some 70 years be{ore the differentral see equation (3)-it's trost advanta-
calculus was bom!) Let's rnake use of geous ro choose the pararneter I i1')
the following general prhciple. Assune such a way (t : tn) that the cylindrical
f (t) is a function of t {for simplicity's bar:rel has the greatest capacity (1,). tn

OUAlllTUltl/I.OOI(IlllG BACI( 3I
this case, when there are small devia-
tions of t from ro (which are inevitable o
in practice), the deviation of the bar-
rel's volume v from the maximum 0 t I 27 64 t25 256 i43 512
volume vo will be insignificant.
Figure 5
Kepler then computes (for a given
)")the value of the ratio t that yields the ni{icant. Then arnong all the barrels "what an Austrian barrel real1y is: it is
maximum capacity for the barrel. Using for which the distance ND equals the one whose generatrix ANB is about
the notion of the derivative, we can do given )", I will choose the one with the 2y'times longer than the bottom's di-
this much faster and more easily than greatest capacity." ameter. Since in choosing the stave
Kepier could. Let's use equation (2) for Following the procedure outlined length PNQ we have to take into
v. Then by Kepler, we can express the capacity account the thickness of the top and
v of the barrel in terms of the distance bottom of the barrel and the fact that
v' lx) =Yzx(),2 - 3x2) = 0, ),, the radius y of the smaller base of the staves stick out a little beyond the
which works out to the truncated cone, and the distance z top and bottom, the staves must be
I =Atr (where F is the foot of the perpen- about one and a half times longer than
/^
VJ dicular dropped from point D to the the bottom's diarneter."
Using equation (1) to gety I we get larger base of the cone). Let's perforrn Not only that, Kepler also under-
these calculations, using the formula stood how to find the capacity o{ any
, ) ..2 ,
+)'-=t- 2)'"1 for the volume of a truncated cone and "Austrian baruel" by means of an ordi-
-x-=
bearing in mind that the altitude of nary ruler: just measure the distance
7. the truncated cones here ecluals ND and apply equation (5) to iind the
' r/ t''' volume vo. "Butthe vintner called out
DF =J r.:-z:. the capacities immediately, without
,,,:i= {1=1.1t /r Sinrl'tle tn.tnipulations tltcn y'icl.1 counting, " Kepler recalled. "Appar-
l+l
'). ently his ruler is not an ordinary one,
For this value of parameter t the capac- l : :lT. [r'] + r--- r')- + r't-- - r' r'l
' ." L' -'l but has marks that are positioned dif-
-1
ity of the cylindrical will be
barrel ferently. How is this ruler calibrated?"
(for a grven value o{ },) and will i6)
greatest Kepler turned his attention to a se-
eclual Kepler was able to find the correla- cret amphora the vintner mentioned.
fi ..1 tions among the various barel dimen- He guessed that it was probably some
'o=ir,,/rn' (5)
sions {that is, the values ol y andz1.or kind of little barrel as well, whose
"Now I will corsider the more general a given ).) that produce maximum capacity is taken as a unit o{ measure-
case," Kepler writes, "in which the barrel capacity. We can obtarn Kepler's ment. But i{Austrian coopers produce
staves (that is, the boards that make results by solving the following prob- barrels with an optimal ratio of gen-
up the sides of the barrel) are bent, not lem: using derivatives, prove that eratrix to diameter (that is, t = 1.5),
straight as they are for a cylindrical among all the barrels of the type de- surely they must use the same ratio in
barrel. Such a barrel can be repre- scribed (with a given value ), for the makrng the standard barrel-amphora.
sented with a sufficient degree of pre- distance ND), the one that has the Assume that for the standard barrel-
cision as two identical trtncated cones greatest volume is the cylindrical bar- amphora the distance I'1,O, equals I,(fiS.
joined together along their larger cir- rel whose generatrix is 2/'times longer 4). Then, taking the capacity of the
cular bases." (See figure 3.) than the bottom's diameter. barrel-amphora as the unit volume,
Now let's use Kepler's idea that, ecluation (5) grves us
N
near an extreme value, a small devia-
tion of the rndependent vanable leads 1= --r-13.
to an insignificant change in the func-
3\/31
l/ z tion's value. In our case/ this means Using equation {5) again, \are get the
that if the dimensions of some barrel following adequate approximation of
don't deviate much from those of the
best barrel (that is, a cylindrical barrel
with the ratio AB: AD =2'/), these de-
K viations will have an insignificant ef-
Figure 3 fect on the barrel's volume. So the
volume of abanel differirlg only slightiy /trr
"Of course," Kepler continues, "I from the "llest" one can be computed
am accepting a certain amount of according to the same formula-ec1ua-
imprecision, but i{ the banel is not too tion (5).
Figure 4
"potbellied, " this error will be insig- "It is now clear," writes Kepler,

38 oUAltTUnrt/lltAY r sso
the volume of an "Austrian barrel": met with the Emperor's Mathemati- when the coopers of Linz construct a
cian at the Linz City Hall. barrel, they are guided by only one
vo : (tr/)',)3 barrel-amphoras.
consideration: that the staves be one
"Honorable Elder," began Kepler,
So, in ordelr to find v, it's enough to "I am interested in the method Aus- and a half times longer than the di-
know the ratio )"/)", (rather than the ffian coopers andvintners use to measure ameter of the bottom."
actual value of )"). In fact, you don't the capacity of barrels." "Absolutely correctt " affirmed the
even have to know the ratio itsel{, just "We11, you see/ Honorable Mathe- startled cooper.
its cube. The values of ()"/),,)3 are i4 matician of His Majesty," objected "Further, to measure the capacity
fact the markings on the ruler-that the elder, " this is a trade secret o{ our of the barrels you use a mler calibrated
is, we have to calibrate it according to guild. This secret has been handed according to the law of'cubes."
the "law of cubes." At distances of trr, down from generation to generation "You are nght again," said the elder
}l't,3Xt,... from the end of the brass by our craftsmen since the days of the of the cooper's gurld with even greater
ruler, the numbers 1, B (= 23), 27 l= 31, Venerable Cooper." surprise, looking apprehensively at
... are written (fig. 5). To state the "I had surmised that at one time Kepler. "Are you endowed with su-
general case: at a distance kl., (where there existed an outstanding geome- pematural powers/ Honorable Kepler?
k is any positive integer) from the end, ter who taught this method to your I have heard that, as the Emperor's
the number k3 is written. If you stick guild. I believe I have managed to Astrologer, you can read the past and
this ruler into a barrel so that its tip guess his secret". the future from the disposition of the
touches the lower edge o{ the barrel's "Then tellme, Honorable Mathe- stars. Could it be that the Venerable
bottom (point D), the reading on the matician of His Majesty, what the Cooper's secret was deciphered by means
ruler at the bunghole (point l/) will secret of the Venerable Cooper is, in of astrology? "
indicate how many barrel-amphoras your opinion, and I promise to con- "No," answered Kepler dtyly, "I
that barrel holds. firm whatever parts of your explana- computed it by means of mathemat-
tion are corect." ics. " And there was nothing more to
IN rsr auruuN or 1 6l Kepler's
5, at "My reflections have led me to the be said between the mathematician
request/ the Elder of the Cooper's Guild conclusion," answered Kepler, "that and the cooper. o

. LOOKING BACK

clude that the entire history of radro is


confined to the 20th century. This

Fl'om lhe conclusion would be wrong. In look-


ing at the history of radio, we have to
remember that this technology stemmed
from discoveries in the theory of elec-
prehistot'y ol radio tricity. And the scientists who made
the most important contributions in
this field areFaraday, Maxwell, and
Hertz.
This article originally appeared in 1984 taraday and his eil[Bl,ilneltts
to commemorate the 125th anniversary The most important discovery was
made by Michael Faraday in 1831.
of the birth of the great Russian physicist Faraday found that electr.icity can be
Alexander Popov generated by changes in a magnetic
field. This phenomenon is called
" electromagnetic inciuction. "
by S.M. Rytov Faraday's tremendous experimen-
tal work was guided by the concept of
interaction through a medium, which
ADIO IS ONE OF TI-{E, DISTINC- outer space. It exerts a tremendous he visualized as lines of force traveling
tive features of modern civlliza- influence on our daily life. through space. This stood in direct
tion. It's a means of communica- Not that much time has elapsed contrast to the concept of immediate
tion and an instrument for scien- since the first experiments in radio action at a distance, supported at the
tific research, both on Earth and in communication, so we might con- time by the authority of Newtonian

OUAnjIlJflll/tOOI{IlllG BACI( 3g
cal quantity (similar to electric cur- Maxwell himself felt that the elec-
reng caused by electncal charges moving tromagnetic theory of light hadbeen
in space) that he called "electric dis- created by Faraday. He wrote some-
placement." The discovery of electric thing to that effect 1n 1864, refemng to
displacement was the direct result of ideas that Faraday had expressed in
his efforts to create a mathematical oneof hispapercin 1846. Butperhaps
structure for electromagnetic theory. a distinction should be made between
The beauty of Maxwell's ecluations, a general idea and a precise statement
which describe a multitude of physi- that can be verified by experiment.
cal phenomen4 appeals to every physi Maxwell's theory not only supplied a
cist or mathematician, and ther mathe- general framework for previously known
matical structure provides guidelines electromagnetic phenomena, it also
for research in theoretical physics even provided a means of predicting new
today. phenomena and describing them quan-
An important conclusion that fol- titatively. The most important phe-
lowed from Maxwell's theory was the nomenon it predicted was the exis-
Michael Faraday existence of transverse electrornag- tence of electromagnetic waves. But
netic waves, which have a finite propa- neither Faraday nor Maxwell iat least
physics and such eminent physicists gation velocity given by the equation at the early stages of his work) had
as Ampdre, Weber, Kirchoff, and noticed the direct connection between
Thomson {Lord Kelvin). Faraday's ideas ( =- 1
optics and electromagnetism, which
En $,,
encountered considerable opposition, V became apparent only after Maxwell
but he forged ahead and made one im- The constants of dielectric suscepti- had worked out his theories explain-
portant discovery a{ter another. bility(Eo) and magnetic susceptibility ing phenomena in electricity and
Faraday used little mathematics- (po) had been measured previously. magnetism.
his method of research was almost The value of c tumed out to be equal to
purely experimental. He derived his 193,088 miles per second. This was ller[ and his "liuing Bqtlaliolt$"
ideas under the direct influence of very close to the speed of light ob- Electromagnetic waves were {irst
experiments, turned to experimenta- tained in optical experiments by Hip- detected in the laboratory by the Ger-
tion as a way of testmg the truth of his polyte Fizeau-L93,118 miles per sec- man physicist Heinrich Hertz in 1888.
ideas, and expressed his results in ond. This 1ed Maxwell to conjecture Hettz's experiments have long been
nontechnical language. that light consists of electromagnetic considered remarkable examples of
vibrations. Maxwell {irst mentiqned experimental ingenuity and theoreti-
lli|laruuell and his lnatholnatis$ this brilliant discovery in a letter to cal foresight. The Russian physicist
f ames Clerk Maxwell understood Faraday dated November 19, 1861. O.D. Khvolson called them "eternal
the profundity of Faraday's ideas and Neither of the values given above are classics." Hertz also made important
decided to create a mathematical frame- quite accurate. Maxwell no doubt contributions to the mathematical
work for them. This paved the way for understood that the speed of light had framework of Maxwell's theory. He
the systematic study of electromag- beenmeasured soheplayed admired its beauty and wrote that its
netism. In his famous book ATreatise down the numerical disagreement. equations have a life of their own-
on Electicity cmd Magnetism, he wrote: that they are wiser than we arel even
"I have therefore thought that a trea- wiser than their creator, and give more
tise would be use{u1, which should information than was put into them.
have for its principal object to take up These words proved prophetic. The
the whole subiect in a mathematical second half of the 20th century has
manner, and which should also indi- seen the flourishing of so-called "gauge
cate how each part of the subject is theories," which from the rhathemati-
brought within the reach of methods ca1 point of view are generalizations of
of verification by actual measurement.// Maxwell's equations to suit the needs
Proceeding along these 1ines, he did o{ subatomic physics.
qurte a bit more. Maxwell introduced Immediately af.ter Hertz's experi-
the concept of the electromagnetic ments, many scientists proposed us-
field, finally rejected the concept of ing electromagnetic waves for com-
action at a distance, found that a change munication without wires. It's inter-
in an electric field in time generares a esting to see what Hertz himseJ{ thoug}rt
magnetic field, and derived the equa- about it.
tions that bear his name. To do all this In 1889 an engineer from Munich
Maxwell had to introduce a new physi- James Clerk Maxwell named Guber put the question to Hertz

40 OUA[IIU[t/l/l1llAY 19SO
iron filings,l I aimed at constructing a
Popou and his "lhunder deteclor"' device that would automatically de-
By the time Popov began to work stroy contacts between filings caused
on the transmission of messages by by electric vibrations." His invention
means of radio waves/ the physical was a great step forward in radio tech-
phenomena involved and the main nology. k1 constructing his apparatus
parts of his apparatus were known. It Popov not only achieved electrome-
was well known, {or example , that a chanical amplification but also used
spark discharged by a Leyden 1ar, an what we now call feedback.
electrostatic machine, or lightning The new device could sense the
consists of damping electric oscilla- distant electrical discharges of light-.
tions. Hertz had shown that such ning, so Popov called it a "thunder
oscillations generate electromagnetic detector," even though it also detected
waves. So the means of generating discharges from an electrostatic ma-
electromagnetic waves were already chine and a large Hertz vibrator as
available. The power of the spark had well as the rapid strokes of an electri-
Heinrich Hertz to be increased, but this wasn't con- ca1ly oharged ebonite rod.
sidered a serious obstacle-it was a As we come to the end of this
directly. In 1897, three years after mere technicalrty, so to speak. article, I'd like to quote {rom Popov's
Hertz died, Guber sent Hertz's reply The mainproblem was how to de- report to the Physical Section of the
to a German research foumal, where it tect signals at a distance. For this Russian Physico-Chemical Society on
was published without comment. Ll pulpose Hertz had used a sma11 spark May 7,1895: "Concluding, I would
his letter Hertz wrote that the lines of generated in the tiny gap of a ring. But like to express the hope that my appa-
force of magnetic and electric fields this method, of course, couldn't serve ratus, after further development, can
travel like light rays onlywhen their any practical purpose. be used to transmit messages through
vibrations arc y ery rapid. Then both Five years before Popov invented space by means of rapid electrical vi-
kinds of line forces are inseparable his device, an apparatus called a brations, as soon as a sufficiently
from each other, so that the rays of "coherer" had appeared. It consisted powerful source of these vibrations is
waves studied in Hertz's papers can be of a glass tube with two electrodes at found."
called "magnetic" or "electric" inter- each end and metal filings between Actually, though, progress was made
changeably. But vibrations in a tele- them. When an electromagnetic wave during the early years of radio com-
phone wire are much slower. So radio travels through the tube, tiny sparks munication by increasing the height
waves o{ very low frequency are re- run between the filings at their points of the receiving antennas. But ad-
quired because lHel.tz thought) the of contact so that the resistance of the vances on all fronts continued at such
frequency of the radio waves should filings drops from hundreds of thou- a pace that as early as 1897 P.N. Ribkrn,
be the same as that of the transmitted sands o{ ohms down to less than a Popov's assistant, managed to trans-
sound; consequently, the wavelength hundred ohms (or even less than ten). mit radio signals to a distance of five
would have to be very large. Hertz The coherer is an excellent device for kilometers.
suggested that radio waves could be allowing a high-frequency signal to The 20th century has brought many
focused by concave mirrors to obtain a short a circuit with an electric battery new inventions and improvements in
signal strong enough to detect. He in it. Unfortunately, as soon as the
saw no other means to achieve that coherer has done its job, it quits. You
end. But for very long waves, these have to give it a shake to restore its
mirrors would be the size o{ a conti- high resistance. So this device was ob-
nent! viously not a receiver constantly ready
I'11 make no comment about this to detect a signal.
letter either. Let's just say that even a Popov transformed the coherer into
genius has a hard time trying to guess a reliable radio receiver by inventing a
the solution to a technical problem method of restoring its large resis-
decades in advance-in this case, wire- tance automatically. kr an article that
less telegraphy. appeared in fanuary 1896, he wrote:
Of course, mere suggestions that "Having achieved good results in making
electromagnetic waves be used for the sensitivity constant by means o{ a
communication were insuf{icient. A tube with platinum electrodes and
way had to be found to do it. Enter,
stage right, Alexander Popov and his I This refers to Popov's earlier work
apparatus for detecting electric vibra- on improving the parameters of the
tions. coherer itself. Al exan d er Stepanovi ch Popov

OUAl\lTUll/l/LOOI(Il\lI BACI( 41
radio technology-for example, tele- other plate B, andfinally through the
vision, which plays an increasingly coil of the electromagnetback to the
important role in our lives. In 1931, battery. The power of the current is
the centenary of Maxwell's bifth, G.G. not sufficient to draw the anchor of
Thomson, himsel{ an eminent experi- the relay; but il the tube is subjected to
menter/ said that the discovery of ra- the action of electric vibration, the
dio waves has influenced the entire resistance instantly decreases and the
development of civilization. Radio current increases enough for the relay
waves help draw the Earth's inhabi- anchor to be attracted. At this mo-
tants together and have led to social, ment the circuit running from the
educational, and political changes that battery to the be1l that is interrupted
we are sti1l in the process of assimilat- at point C closes, and the bell begins to
ing. {unction. It shakes the tube, decreas-
ing its conductir,rty, and the relay breaks
the bell's circuit. In my apparatus the
0utline and desct'iiliolt olPorutt's
bell above the tube; when it rings, it resistance of the filings after a strong
deuhe hits the middle of the tube. (A rubber shaking may reach 100,000 ohms, but
Here's an excerpt from Popov's 1896 ring keeps the tube from being bro- the relay (which has a resistance of
paper "An Apparatus for Detecting ken.) It is convenient to attach the about 250 ohms) draws the anchor at
Electric Vibrations": tube and bell on a common vertical currents of 5 to 10 milliamperes {the
"The figure illustrates the positions plank. The relay can be placed else- limits of adjustment)-that is, when
o{ the constituent parts of the appara- where. the resistance of the whole circuit
tus. The tube with filings is sus- "The apparatus functions as fol- drops below 1,000 ohms." O
pended horizontally between the two 1ows. The current generated by a 4- or
clamps M and -A/ by a light watch S-volt battery flows from the contact
spring, which is ben t rn aztgzagso as P to the platinum plate A, then across
to be more elastic. There is an electric the filings contained in the tube to the

CONTINUED FROM PAGE 7

If you like what you see... They're even located inside some bones,
sometimes donm to the smal1 bones of
the feet. As a result, the respiratory

OUANTUM
system of a duck, for example, takes
up about 20% of its body volune (2%
for the lungs and l8'/. for the air bags),
while a human bemg's respratory systern
takes up only 5%. A bird's lung, as
can be delivered to your door next fall. opposed to a mammal's, is a set oi thin
pipes connected in paral1eI, open from
Clip the coupon below and mail to: both sides and surrounded by blood
vessels. The bags are connected to the
Quantum magazine
1.7 42 Connecticut Avenue NW
lung in such a way that air flows
through a bird's lung in the same di-
Washington, DC 20009
rection during both inhalation and
exhalation. So il the breathing proc-
Please send one year 14 issues) of QUANTUM to: (please print) ess only the volumes of the bags change
while the volume of the hing remains
Name practically constant. Since there's no
need to inflate the lungs, surface-ac-
Address tive substances aren't needed rn a bird's
lungs.
City State_Zrp Maybe you thought it was a silly
question, "How do we breathe?" I
wonder now what you think of the
I have enclosed a check or rnoney order for $9.95, payable to Quantum answer: "Not as well as birds!" O
magazine.
K.Y. Bogdanov is a doctorul candidate in
physical and mathematical sciences,

42 OUANIllll/l/]lllAY 1 SSO
accordance with Article 242 of the
QUANTUM SMILES Criminal Code, theft is the unlawful
appropriation of commodities belong-
ing to another pafiy. In the present
case no theft has been committed,
since the saline solution was not mis-
appropriated; rather, it was returned
Disol'del' in lhe coul'l! in its entirety to the brewery's main
pipe."
The brewery owner appealed this
judgment, now arguing his case this
When "jrtst the facts, ma'am" way: "The issue is not the theft of
saline solution but the theft of energy.
just wonT do If the saline solution is used to cool the
defendant's cellar in addition to my
own, I have to pay more for electricity
by V. Fabricant to operate the central cooling unit."
The ruling of the Court of Appeals
was, from the pornt of view of physics,
I N A PROVTNCTAL TOUA{ OF CEN- tity." So the power mmpany triumphed only half convincing: "The saline
I tral Russia at the turn of the cen- in the end. solution acquires heat from the re-
I turv, the owners of an electric power A more sophisticated legal issue tailer's cellar; therefore, energy be-
I pr*. caugnt a ceftaln Ivanov m the arose in another case. Beer produced longing to the brewery is not being
act of using electricity " fuee of. charge," by a certain brewery was stored in a stolen. On the contrary, the brewery
so to speak, inhis home. Hehad sur- cellar some distance away from the is receiving gratuitous energy from
reptitiously connected a wue to a power bottling plant. The cellar was cooled the retailer."
line belonging to their entelprise. The bypipes containing a circulating sa- We all agree the judge was wrong,
company sued Ivanov for theft of its line solution that came from a central but not everyone can corectly ex-
property, but the malefactor won the cooling unit. The main pipe connect- plain his error. Can you? O
case in court. According to the exist- ing the cooling unit and the cellar
ing criminal code, theft was defined as happened to pass near the cellar of a
pertarning to an object possessing mass retailer. The first story can be {ound in
Electicity and Heat by B. Bulyabash and
and size. The Senatg however, which After a while, the owner of the V. Gurevich, Moscow: Nauka
considered the case on appeal, deter- brewery discovered that the retailer Publishers, 1978. Thesecond story is
mined that electrical energy may be was using the saline solution to cool taken from a German scientific
viewed as propefty/ "being mobile real his own cellar and sued the retailer for monograph, "Questions of
estate/ without being a material en- theft. The judge, however, ruled: "[r Thermodlmamical Analysis," by P.
Grassman.-Ed.

ILL CAME TO MR. COHN'S


place right a{ter lunch. At that
time of day the Texas sun turns
all the cheeses of Texas into gooey
Ahol'$E is a hol'se
cream cheese, a1l the water of Texas
into boiling water, while al1the in-
habitants of Texas can only dream of a (ol cout'$E, ol cout'$g)
quicker and less painlul end.
When his temperature fell below
the boiling point of iron and all the ice
in the house had evaporated, Bill fi- Even cowpokes get the blues
nally recovered the power of speech.
"You understand, mister Cohn, it's when they place a few bets
a1l because of that Stanleyl He's a
friend of mine, so to speak. Until the
and lose, and lose ...
other day I only bet against him
once't."
by A.S. Yarsky

0lJAlrTU]tlisllllil.ts 43
"And how did that bet go?" Mr. you can't handle but I'll be
Cohn was struggling hard to stay awake. able to ride?'
"WelI, mister Cohn, Stanley was "And I answer without
goin'off on this trip-Natchez, I think battin'an eye: 4 Y-'' -A---4-
*-
it was, in the upper straits of Red "'Well, Stanley, mark
River. Me, I been there, so I says to my words: I'11 use that
him: pony to take you to the
" 'That's seven days on horseback.' nearest gtaveyard.'
" 'Aha,'says Stanley, 'and to Baton "But he keeps right on:
Rouge?' tt tIfiglte there's a horse
"'Aday,' Itellhim. Like that, Bi11.'
z rr$d
toLafayette?' " 'OK/ I says, /I bet ya
o
" 'About four days. It's a tough 10 to 1 there ain't. Dig out 6
N
road.' a buck and let's have a 6
Z
"'Andhowlongto-2/ Andhe look!' c
o
names this hole in the wall. " 'Take your time, Bi1l, o=
U
" 'Never been there,' I says. 'I guess and get your ten smacks a
itbe atleast ten or twelve days.' ready. The horse is right :
" 'You know, Bi11,'says Stanley, here in your stable. Look!'
'these four towns are situated smack "Then he picks up this notebook I But then I remember the terms of the
dab at the four vertices of a rectangle.' use to do my accounting/ opens it to an bet, and I figure everything ain't lost
" 'So what?'I says. empty page, and writes: yet.
"'So this. I can te1lya exactlyhow aca-c " 'Listen here, Stanley,'I tell him,
(10)
long it'lltake ya to get fs And he b d b-d' 'that horse rides under you/ no argu-
names that hole in the wall agin.
-./ "We11, mister Cohn, {irst thing, I ment there. But you said I couldn't
" 'You mean you been there?'I ask askhim: handle it. My twenty to your twenty
him. " 'Stanley, why'dya start your count says I'11 ride that mustang easy as pie!'
" 'Nope,'he says, 'ain't never been.' from ten, there on the right, in paren- " 'All right, my twenty says you
" 'Then how the heck d'ya know? zathes?' won't.'
Somebody tellya?' " 'That's how much you owe me. "We11, mister Cohn, I lost that race
" 'No siree. I just know it's eight And that's theplace to write thebill, too. Mister Cohn, you don't make as
days!' Bill-right in your account book!' much money in a week teaching school
"Welf mister Cotm, the argument "'Not so fast,' I tell him.'S'been a as I lost to that son of a gun Stanley in
goes on and on and ends up in abet. I long time since I went to school liut I half an hour ... AnO dang it, I still can't
get on my palomino and-can you can teIl you this: your number ten figure out how the heck that lame
beat this?-I make it to that hole in horse is lame on all four legs. And you mustang-I mean that incorrect equa-
the wall in exactly eightdays! Wish ain't getting that ten spot'til it takes at tion-gives correct answers ! In fact,
I'd never seen it. Eight days later I'm least one step.' that's what I came here to ask you
back here, and Stanley tells me how " 'Then take a gander at this,' says about."
there's this'theorem' thing.l Turns Stanley, and he writes: "Science requires sacrifices, " de-
out I dragged my tail to the Red River 89 8-9 clared mister Cohn, "and you, Billy
to prove some wordl don't even know ,-1:
2-3' boy, were sacrificed at the altar of
whatitis!" " 'Check for yourself,' he says, 'and science! First of all, whoever told you
"Your friend Stanley's a clever fella." fork over the dough!' an incorrect equation can't give cor-
The heat was falling off, and Mr. Cohn "Now tell me, mister Cohn, have rect results? Ba-lo-ney! Even the
was recovering a bit. you ever seen a guy make a ass of biggest liar of them all can acciden-
"Too cleverby aha7[," Bill snorted. himself and then admit it? Or a guy tally tel1 the truth. And besides, while
"And so, mister Cohn, the other day who had no use for ten bucks ? you were telling me about your lost
this Stanley comes by to see me/ we " 'Hey, Stanley,'I says, 'I'11 bet you 'races,'one after the other, I discov-
shoot the breeze about the weather, another ten that your gelding can't ered the underlying logic of your dis-
beef prices, and all that, and then he up take another step!' grace. Take a peek at this formula:
an'says: " 'Then fork over another ten, 'cause q
" 'Listen, Bill, what d'ya think, can here it goes.' Andhe writes: c-=Q(2_Q\
aV d)'
I find a horse in these here parts that Never mind where I got it. What's
595-9-42 important is that this is just another
1
39 3-9 -6 3' way of writing that same equation
"Il ABCD is a rectangle and S is an
arbitrary point, then SA2 + SC2 = SB2 + "L,ose two races in row-that's never (10). It looks a little dilferent, but it's
5D2." -fry andprove it.-Ed. happened to me before, mister Cohn. really the same ecluation. But now,

44 OUAIiTU[l/]i{AY lSSO
Billy, you cantake arry b mddanduse hour will net you the tidy sum of "Look,misterCohn,"hesaidina
them to compute a and cl That's it. twelve dollars a day ...1" whisper. "I made that up mysel{ ..."
Simple and foolproofl Want an ex- Mister Cohn fell silent for a mo- "These examples aretivial," Mr.
ample? Easy enough. Name any two ment. Cohn declared pitilessly. "We won't
numbers. Two and seven? Fine. Let's "You know, BiJt you should assign bother with them. Now, you ask your
take b : 2, d = 7, andworkit out: tlis impossible task to someone who's friend Stanley to come up with one
a 2/^-. 2\t:. 2 12 24 richer than Stanley. Otherwise the more fraction that can be simplified
c 7\- 71 7 7 49'
-:-t, idea won't last for more than a couple like this-and doesn't, of course, con-
Sowecantal<e a:24, c=49, or a=48r days." sist of identical digits. Remember,
c = 98 ... Now let's check it with "Let me wory about that, " replied Blll-if there are two-digit numberc
equation (10): Bill. "But still, I'd like to knowwhy in both the numerator and the de-
24 49 24-49 that horse can'trear its head. And one nominator, there are only four frac-
1111 otherthing, misterCohn: whatifhe tions likethat! I'vewrittenout three
48 98 49-98 knows this one, too?" of themforyou. LetoldStanleyfind
L11a1 I L_I "If he does"-mister Cohn was the fourth one.
Incidentally, you can double, or triple, getting hot-"let him find irreducible ,a @ @EL
the denominators/ or numerators:
24 _49 _24-49 tion(ro)!,,
and.cldthatsatisfuequa-
fractions alb
wl{AT r&AcTI?N
6 21 6-2t'
Multiply them by ten i you want to!" @.-, Dr0Jy'r $ffsrgB
__.@ #s
Satisfied with his explanation, Mr.
Cohn took a deep breath. Bill looked
Is kvr.tAwav (ra) r,[#fififrr_ CI0$$fiI
over the computations carefully and
summed it all up. trygffry CINsr
"Twenty at most. Won't win back
more than that from Stanley using 'r#f#iififl.Iyg^s
G 's "Oh, yes, I almost forgot," contin-
this trick."
"Now hold on a minute," said Mr.
T ued mister Cohn. "Ityou repeat the
last digit in the numerator several
Cohn impatiently, "this Stanley of ARfrIRRNWCBIIN? times, and the first digit in the de-
yours may already know the rule I
discovered. But here's something he
,,€ # @- nominator the same number of times,
our incorrect canceling continues to
most surely won't be able to do, sim- "A11 right," said Bill. "But what if yield correct results. Trust me, Bill,
plybecause it's impossible! Bill, just he knows that one, too?" but veri{y, as someone once said:
ask him the following question: Can "Enough, aheady!" Mr. Cohn cut 19 tgg 1gg9 1
his number 10 horse rear i* head!! In him off . "This is gettrngalittletedi- ss: g%= gggs= s'
other words, ask him to satisfy rela- oso. Tell me, Bill, do you like the You can dare Stanley to calculate this
tion (i0) for a > c! But don't forgeg Bi11, {raction 19195?" in his head:
that all these letters-a, b , c, and d- "Looks like a regular old fraction to t6666666666
must be positive integers. You got me. Reducible, ain't it?..." 66666666664'
that?" "Yes, indeedy," Mr. Cohn exclaimed, "We11, Bill, is that enough?"
"it's reducible all rightl But how?
Look at this. fust cross out the identi
IS RETATIONT cal digits-the last digit in the nu-
merator and the first digit in the de-
(to)PossrBIrp nominator:
lg1
I,ITIIEII a,>c ? 9s:
@ @ '
Check it out! Flere's another one:

Bill's sullen face lightened up a bit.


16 _!
64 4'
"If everything is like you say, mis- And another: '
ter Cohn, I'll charge him five dollars 49 4"
for every hour he sits around wracking 98 8'
his brains. Five bucks an hour-that's A weird expression came over Bill's
fair enough, atn'tit?" face. He grabbed a pencil and wrote
"Don'tbe so greedy, Bilt" Mr. Cohn with a shaky hand:
objected. "This is something that'll s7 _72_3V6
set you up for life. Even fi{ty cents an 5s n &az'

IUAIJIUl|l/SillIIIS 45
t*
t-
L

lqo
q
/7lU[S I

-n
,lt.FRt
.)(l0Rs
,fJtr
sA \5
)4'A r-/ ).,>'\
t .l

t\
f-z=.-'
twl
z'--t"'-
'o;-?----A If--tt
;> t-;H---,
I =----
t
I

;---_;?-r4.& ;:127
.1 z-s

"Yes, mister Cohn, thanks.


Bill got up, iooking sheepish. "I don't
Bg1-//
k ^E
V's:'Vs'
mister Cohn. Less than nothin' ..."
Stanley turned red, muttered some-
have ... I mean ... uh, until I win my thing unintelligible, and ran off.
money back ..." "Clever Iittle thing, ain't it?" he A few days later Mr. Cohn had to
"How much?" asked Mr. Cohn, said. "The number three is incor- visit a nearby town on business. Over
getting right to the point. rectly taken out from under the square by the general store he noticed an
Nowpoor Bill completely andut- root sign, but the answer's correct advertisement:
terly embarrassed. He stood up, wiped anyway. But that trick didn't do its
the sweat fromhis forehead, andheaved maglc for me: Bill figured everything NOVELTY!!I
a big sigh. out in five minutes and even wrote a M.Y. Gardelman's 2
"Well, I actually need three sev- generalizing equationl That totally Living mathemadcks for the millionsl
enty-three". messed up my plans ... But no matter. Useful and amusing applicationsl
"Three dollars?" Mister C-ohn cleared That isn't why I came. Actual1y, I Onlv $3.731
his throat in disbelief. "Take forty for came to askyou... I mean... well ..."
starters." In the comer of the poster someone
"No, mister Cohn. For what I have had written:
(-irr Ernq
_,
in min{ I need exactly three seventy- 3y
14J
three That's right. Thanks, mister
. ..
Cohn..." 4y 4'
About a week later, as you may
have guessed, Stanley came to visit
Mr. Cohn. His appearance was elo-
fu*ivotfi -€sCe=*
WHAT F8BilTUfi,&
13

quent testimony to the fact that he "How much do you need, Stanley?"
wouldn't be a source of income for Bill
or for any other fast-talkin', free-wheelin',
mister Cohn asked point-b1ank.
"If it's not asking too much, lend
[S HIBDESI
wager-lovin' cowpoke. 11s-tt Stanley took a deep breath. BEHTffiD THXS 6)
"G1ad to see you," said Mr. Cohn.
"Sit down. So tel1me, how are things
going?"
"Lend me three seventy-three.
you back in a couple days."
I'11

"You krrow, Stanley," said Mr. Cohn,


pay
SI}TF&EFTCATIf,IE
__a* * @>
{
O
"What can I say, mister Cohn? Life handing over the money/ "that's the
2
goes on ... I just stopped by to show second time this ridiculous sum of An amalgam of the names Martin
Gardner (the great American puzzle
you this little doohickey." $3.73 has cropped up. What's it all master and science writer) and Yakov
Stanley walked up to the table, about?" Perelman {the most famous Soviet
took a sheet of paper, and wrote: "Oh, just a lot of nothin', really, popularizer of physics and math)-Ed.

40 otlAirTrtrt/rrtAY rsoo
AT THE BLACKBOARD

Cunstl'uclims with colnpa$s alone


geometers version of "one hand tied behind my back"
-the
by Dmitry Fuchs

MONCTFtr,COUNTLESS GEO the book Euclidius Danicus by the that it leads to the required result is
meffic consff uction problems there Danish geometer Georg Mohr left to you in each case.
are/ one often encounters those {1640-1697) appeared. There it was
requiring construction of a figure shown that all problems reducible to Staftemenlollhe nesult
"with straight edge alone" or "with quadratic equations can be solved One can scarcely hope to draw a
compass alone." Yet it has been known geometrically with mmpass alone. More straight line with a compass/ so all our
for centuries that the lack of a straight than a century later the problem was problems will be to construct a certain
edge in no way narrows the circle of restated and solved by the Italian Itxerwn point (in the plane).
possible constructions. Everything Mascheroni (1750-1800). Since then TsroRr,n. Suppose that a point M
that can be constructed with straight the resulting statement has been called can be constructed from the points A,,
edge and compass canbe constucted the Mohr-Mascheroni theorem. A .." A, with straight edge and compass.
with compass alone. will be presented
proof of this theorem Then M can be constructed from A,,
The idea of constructing with below. ..., A, with compass alone.
compass alone was suggested long ago In all the construction problems To prove this theorem we have to
by the Italian scientist Giovanni Ba- that follow I'll restrict myself to de- know precisely which constructions
tista Benedetti (1 530-1 5 90). In 167 2 scribing the construction. The proof can be performed with a straiglrt edge-

o
E
Y
E
t
o
c

OUA[JIUIYl/AI TIII BI.ACI(BOAfl D


Figure 2 Figure 3

o
P

that is, with a ruler. With a ruler we with centers A and B through point O desired point.
can draw a line through two given and then draw a circle with center O Problem 6. Let a circle S with
points and find its intersection points and radius AB. Let P and Q be the center O and different points A and B
with straight lines and circles con- points where this circle intersects the be given. Construct the circle passing
structed earlier. But since at the very two circles constructed before; then through point O and the points where
outsetwe were only given some points, the arcs OP and OQ are equal to arc a straight line AB intersects circle S.
each of our straight lines must have AB. Now draw circles with centers P {We assume that line AB doesn't pass
been drawn through two points con- and Q through pornts B and A, respec- through the center O of circle S and
structed still earlier; similarly, each of tively, and take their intersection point intersects this circle at two points.)
our circles passes through a point R. Finally, draw a circle with radius Prove that the constructed circle is
constructed earlier and has another OR and center P (or Q, it makes no di[- precisely the set of points symmetric
point constructed earlier as its center. ference). The point where this last to the points of line AB with respect to
So in the course of the construction circle intersects arc,4B is the required circle S.
the ruler must be used only to solve point C. Construction (fig.4). Tkough point
one or the other of these elementary Problem 5. Let a circle S with O draw circles with centers A and B.
problems: center O and a point P be given. Con- Designate their other rntersection point
Prcblem 1. For given points A, B, struct the point P' on the half line OP P. Then construct the point P'sym-
C, D, construct the point where the such that OP . OP' = 12, wherezis the metric to point P with respect to circle
straight lines AB arrd CD intersect. radius of circle S. (Such a point P'is S (see problem 5) and draw the circle
Prob\em2. For a given circle S, its called "symmetric to point P With with center P' through point O.
center C), and points A and B, con- respect to circle S.")
struct the points where the circle S Construction Case 1: point P lies Basic construclions
and the straight line AB intersect. outside circle S (fig. 3). Draw a circle Corntuction for problem 2. I{ point
with center P through point O. Let Q O does not lie on line,43, then we can
Auxilial'y coltslr'ttction$ and R be the pornts where it intersects use the construction from problem 6
From now on, by a "construction" circie S. Now draw circles with centers or even this simplified version: we
we mean a construction with com- Q andR throughpoint O. Thepoint find the point P exactly as in the
pass alone. We'lIbegin by solving four where these circles intersect (other problem 6 construction and then draw
auxiliary problems. than O)is owpoint P'. (This construc- the circle with center P and radius
Problem 3. Let rr,vo different points tion is also valid when point P lies equal to that of circle S (iI we know the
A, B be grven. Consffuct the point C of inside circie S but the distance from it center of the circle S, we can measure
thehalf lineAB such thatAC:2A8. to point O is greater thar' rl2.l its radius with a compass); the points
Construction (fig. i). Drawacircle CaseZ: point P lies inside circle S. where this circle intersects the given
with center B through point A and, Using the construction from problem
starting from point A, mark off three
subsequent arcs on this circle spanned
3, we construct the pomts P, Pr, ... on
the half line OP one after another such
Fioure 4 P'o
t
bysegmentsof length AB. Theendof that OPr=LOP, OP5=\OP,... untilwe I

the third segment is the recluired point reach a point { that lies outside circle
C. S. Then, using the previous construc- A
Problem 4. Let a circle with center tion, we find the point P', s).rynmetric +
O and an arcAB on itbe given. Con- to point { with respect to S. Finally,
struct the point C dividing arc AB into we construct the points P' ,, P' ,, ... ort
two equal parts. the half line OP', such that OP'z:
Construction ttis.2l. Draw circles \OP' t, OP's:3OP' t, ... . Then { is the

48 OUAIiTUltl/ll/lAY l SSO
circle S are the desired points. If point and S, intersect and construct the point ruler and your compass. Science can
O happens to lie on the straight iine P' symmetric to pointP with respect hardly bail you ott then. O
AB, then this construction won't do: to circle S. This is our point.
point Pwillmergewithpoint O. Then Dmitry Fuchs js a researcher in l.M.
we apply another construction (fig. 5): Conuluding relnarfi$ G elf and' s m ath and b iology lab or atory.
(His article "BendThis Sheet" appeared
we draw an arbrtrary circle with center If the construction problem data in the I anuaty 1990 issue o/ Quantum.)
A (or with center B fi A : O) that inter- do not consist of points only, then it
sects the circle at two points; then we may fifin outthat a ruler is needed to
designate these intersection points C solve it. Consider this problem, for ex-
CONTINUED FROMPAGE 21
and D and divide the arcs CD andDC ample: given a curve c (drawn in the
plane) and two points A, B, find the
Figure 5 points where curve c intersects the
[o we really need fiis iling called
line AB. In general, it's impossible to
do this without a ruler. Nevertheless, "telnpel'aUre"?
a problem Like this can sometimes be Scientists had worked out the con-
o reduced to problems of the kind we've cept of temperature and introduced it
----+- been looking at, and then they can be as a physical quantity long before they
solved with compass alone. Here's an understood its real meaning. But now
important example: a circle is drawn that we know what it means, is it
in the plane; find its center. This can worth keeping this archaic quantity?
be done in the following way: mark Wherever we're accustomed to talk-
three different points A, B, C on our ing about temperature, degrees Kel-
into two equal parts each (see probiem circle; it's well known that one can vin, and so on, maybe we should think
4); the dividing points are the ones construct the center of the circum- of what they really are-the mean
we're looking for. circle of triangle ABC with straight kinetic energy of a particle-and measure
Constructionfor problem 1 (hS.6l. edge and compass; so this can be done them in joules.
Draw an arbitrary circle S such that all with compass alone. (This problem There ate vety good reasons why
the given points lie inside it and its has a much simpler solution---+an you we don't do this.
center O doesn't lie on either of the find it?) By the way, we see that in Do you think a doctor, for instance,
straight lines AB and CD. (This can problems 2,4,5, and 6 it was necessary would find it convenient to diagnose
easily be done by sight, but such a to indicate the centers o{ the given a patient as sick because of a mean
practice doesn't iit in with our "rigor- circles il advance. molecular kinetic energy of
ous" construction.) One may proceed Not every construction that can be 5.64 . l0 zt J? It's easier and more direct
as follows: take an arbitrary circle performed with straight edge' and to talk about a temperature of 100.4"F.
and, using the construction from prob- compass can be pedormed with straight Besides, using joules instead of de-
Iem}, find the points where it inter- edge alone. (A proof can be found in grees might cause confusion. Con-
sects Lines AB and CDi tal<e any point Numbers and Figwes by Rademacher sider this: 100 | of energy usualiy
of the circle (different from the points and Toepliz, for example.) Neverthe- implies that 100 ) of work canbe done.
previously found) as point O. Then less, a theorem by Steiner asserts that But if the temperature of a body is 100
we follow the construction from prob- any construction that can be done |/molecule (a fantastic value for tem-
lem 6 and get the circle S, that passes with straight edge and compass can be perature/ by the wayl, that does not
through O and the points where circle performed with straight edge alone i{ a mean we can get that amount of work
S intersects line AB; then we get the single circle with the center marked is from it.
circle S, that passes through O and the drawn beforehand. So let's stick with degrees-agreed?
points where circie intersects line
S So if you're going to make con-
Exercises
CD. After that we denote as P the structions with straight edge and 1. Find the values that correspond to
point (other than O)where circles S, compass and discover that you've lost absolute zero temperature in the Fahrenheit
yourruler, don't despair: you can do and Rdaumur scales.
any of your constructions with com- 2. Calculate the mean kinetic energy of a
Figure 6 molecule at 1,000"K.
pass alone. It's worse if you've lost
3. A nuclear explosion geoerates a gas
your compass, but everything will be ball at a temperature of approximately 20
okay if you can borrow one from million degrees Celsius. Find the mean
*"q somebody just for a moment: you
draw a circle, mark its center, and
kinetic energy of one particle in the bali.
4. The unit o{ energy in atomic and
nuclear physics is the electron-Volt (eV).
return the compass to its owner- Find the mean kinetic energy of a molecule
now you can do without it. at room temperature in electron-Volts.
The worst case, of course, is when 5. Calculate theBoltzmann constant in
you've managed to lose both your the Fahrenheit scale. e
0UAl'lIU]il/[I Illt BLACI(80[Rl 4g
MATH EMATI CAL SU RPR SES I

tl'iuinU ottt'tnlay inlo $ttlnlner


lntroducing a new column on
mathematical oddities, quiddities, and just plain surprises

by John Conway

ERE'S A SIMPLE LITTLE ARITHMETICAI GAME. 3. There's no need to continue the lines after this point,
You start with two rows of I's joined by a zigzag of t's, because these l's form a ngzaglxtlike the one you started
as in the example, where the dots mark the places in with but upside down, so that the complete pattern would
which further numbers are to be inserted: be a repeating one:

11111111 1111111111
1 t22421324r
1 1377r2s731
1 1 10 12 3 I 3 17 5 2 I
I 1 3 17 5 2 I 10 t2 3 1
1 1257313771
1 132412242r
1 1 I I I 1 1... 1111I1111L
Now you fill in more numbers by the rule that whenever Can you explain why?
four numbers form a diamond Some similar things happen when you replace multi-
plication by addition. This time you start with two rows
N
of 0's joined by a zigzag of 0's, and you complete little
WE diamonds by using the formula
s
E +W = (l/+ S)+ 1,
the productE14/mustbe 1 more than NS, so that
so that
E:(Ns +t)lw.
E:{N+S+t)-142.
In our example, here's what happens:
Of course, now 0's close the rows:
111111i11
t22421 0 0 0 0 0 0 0 0 0...
13771 01256430.
i 10 12 3 I 026109620
13r7521 05101210410
t25731 0281112720
13241 014810840
1 1 I 1 1 1 1 1. 02365410
0 0 0 0 0 0 0 0...
Try it with other zigzagpatterns. There are some odd
things you'll notice when you play with these patterns. Can you explain why the numbers never go negative,
1. A11 divisions "come out exactly" to give whole why they close by getting back to 0's, and why these 0's
numbers. form an exact copy of the originalzigzag, now the right
2. Eachline "closes" by gettingback to 1. way up?

50 [ttmrIttll/trtAY lsso
Some interestrng pattems tum up if we drop the require- Another hint: The polygon for our first example is this:
ment that there be azigzagof l's joining the top and bottom
rows. Multiplicative hiezepattems arise in which all the
numbers in any given row are the same and positive. For
instance,

J2 vt, 12 /2 v5
11111
and

/^ r-
VJVJVJVJVJ-
/= /-
22222
\/3 J3 \/3 \/3 v'5
1 I 1 1 1 1.
CI
You can see that the numbers aren't all whole any more.
What are they? Hint: The answer is connected with
polygons.
The multiplicative frieze patterns in which all the
entries are positive whole numbers are also connected
with polygons. Can you see how?
|ohr-r Conway is tt ytrofess. u' oi trtatltentatics ttt Prxtceton Lltit,elsits,.

Comic relief for the serious student.


MATHEMATICS AND HUMOR
Edited by Aggie Azzolino, Linda Silvey, and Barnabas Hughes

Read this unusual collection of


What drd the zcorn say limericks, riddles, jokes, and car-
toons that poke fun at the usually
when he fina [Iv drer^r uD ? serious subject of mathematics.

\N You'll find that mathematics can


be funny and that mathematicians
can laugh at themselves.

A"w,-.J:'\ Share a good laugh with your


friends! Order your copy now.
58 pp., #266, $4.50.

National Council of Teachers of Mathematics


1906 Association Drive, Reston, VA2209l
Tel. (703) 620-9840; fax (703) 476-2970

OlJANIUllll/llllAI1lInllAIICAI. SURPfl IStS 51


IN THE LAB

Ullalker in a tnliltler tnloltderlilltd


But since it's almost summer, let's fake the snow,
make some ice cream, and watch the smog roll in!
by Alexander Borovoy

EARLD WALKER'S WONDER- scale by filling a plate with a


ful book The Flying Circus of \' smooth layer of granulated sugar
Physics has been translated into
Russian and has become so popu-
"L T or sa1t. Press a spoon against
you'l1 hear a faint creak. Now
it-
lar in the Soviet Union thatit sold out wet the granules or melt them
long ago, despite several print runs.1 slightly on the stove-see if you
Answers to many of the questions can get them to creak some more.
posed in this bookinvolve settingup Now I'111et |earld Walker have
clever experiments or carrying out the floor: "sometimes snow crack-
interesting observations. In this ar- les when you walk in it but only
ticle I'l1follow up some of Walker's when the temperature is far enough
questions and supplement them with below freezing. What causes the
some curious stories. I'11 also describe noise, and why does its produc-
some additional preparatory experi- tion depend on temperature?"
ments. All these questions and ex- half a century ago. This epic was one
periments have to something do with of the first superproductions in the Jlowlo rutths "ice" ilt ice cl'eam
cold phenomena-a last gasp of win- movie industry. It's renowned for the When ice cream first appeared, its
ter before thelaz-l days of srmmer set alternately rousing, suspenseful, and method of preparation was kept stnctly
in. plaintive background music provided secret. At many European courts,
by Sergey Prokofiev and for its render- chefs tried using snow or ice to freeze
"$queak, ing of the climactic battle scene on the mixture o{ cream, sugar, and fruit
squeak"-lllanmk is
juices-the mixture would be chilled,
Lake Chud. It was there that Russian
collliru! forces, led by Nevsky, de{eated the but it wouldn't freeze. They finally
The great {ilm firector Sergey Eis- better trained and better equipped Teu- had to resort to what is now cal1ed "in-
enstein, famous for the ciassic film tonic knights in 1242. dustrial espionage." And what did
The Battleship Potemkin, is also the Few people, however, are awate these culinary spies find out?
genius behind the film Alexander that the film's central episode, "When my grandmother makes
Nevsky, which is stil1 popular in the the battle on the ice, was actually
Soviet Union though it was shot over shot at the height of summer!
Thefilm shows snowandice on
Lake Chud, while in fact it was as
1
"What's a flying circus ? " you ask? hot as 85"F. Here's how they did
The phrase dates back to World War I
and meant airplanes {lyrng rn formation.
it. A large flat areawas covered
As time went by, it came to include with a mixture of naphthalene
stunt flying in air shows (no doubt and salt. When the actors walked
because vintage biplanes often take part over this "snow," it creaked just
in them). Andwho is |earldWalker? as real snow does when it's bit-
He's a professor of physics at Cleveland
terly cold.
State University. His book, published by
You can easily duplicate the
|ohn Wiley & Sons, is available in
paperback-in English!-Ed "Nevsky solution" on a smaller

52 ottAllrtttlt/tltIY 1 sso
English industrialist Harvey sent ways so. Here's what the Russian
samples of new armorplates for writer Constantine Paustovsky had to
warships to Russia. Dunng test- say in his bookThe Golden Rose:
s ing the shells from the huge heaq, "The French artist Monet came to
guns, instead of breaking through l.ondon and painted Westrninster Abbey.
the plates, crashed against the Monet was working on an ordinary
f>
armor without damaging what- foggy London day. Lr Monet's picture
ever was behind it. Then the the Gothic outlines of the abbey ap-
Russians asked that the tests be pear through the mist. The picture is
repeated. Inthe second series of now universally recognized as a mas-
tests, the shells broke through terpiece.
-.ffi the armor plates. And in a later "But when the painting was exhib-
series of tests, after certain im- ited, it caused confusion among the
homemade ice ctear-rr," writes Walker, provements, the shells began to Londoners. They were astonished that
"she packs ice around the ice cream pierce holes in the armor. Monet painted mist with a crimson
container, and then she salts the ice." Why did that happen? How had the hue, even though everybody knows
So the secret is plain old salt! "Why design of the shell been modified? that mist is gray.
does she add the salt?" Walker asks. The tips of the shells now had spe- "At first, Monet's impertinence
Before trying to answer the ques- cial caps made of soft steel. The cap caused indigrration. But the indignant
tion, maybe you can conduct a couple would spread and melt upon impact. Londoners went outside, looked at-
of simple experiments. Take some tentively at the mist, and saw for the
crushed ice from your refrigera- first time that it was indeed crimson."
tor and gradually add salg mixing Walker also agrees with Monet. "If
it all together. (It would be nice- you've lived in alarge crty," hewrites,
but not essentialfor ourpurposes- "you almost certainly have spent part
to make ice cream during the ex- of your life in ahaze. Why are such
perimentl) You shouldbe able to hazes brown?"
get the temperature of such a What causes a mist's coloring is not
mixture down to *20'C. an easy question to answer. It's not by
Everyday sodium chloride is accident that I've saved it for the very
used most often for this kind of endl Take a close look at the color of
cooling. But more arcane sub- fog smog orhaze in your town and in
stanceswillalso do: KCI NaNOy the countryside. Maybe you can even
and so on. In fact, the champion do some experimenting and come up
temperature reducer is CaClr. If you This prevented the shell from crack- with a method of producing "mists"
mix 42 g of this salt with 100 g of ing, and it served as a kind of lubrica- of different colors. If you do, tell us
crushed ice, you can work the tem- tion for the shell after it broke into the about it--don't hog your fog! O
perature down to -55"C! armored plate. So, under tremendous
So how did your experiments turn pressure/ steel can serve as lubrica- Alexander Borovoy, a doctor of physics
out? And what's your answer to Walkels tion. Admiral Makarov, a talented and mathematics, teaches in the School of
question?2 scientist and as well as a worthy sea- Natwal Sciences atthe Kwchatov Atomic
Eneryy Institute, He has recently worked
man, invented this soft steel cap {or ar- on problems associaud with the Chemobyl
sl0tillu homls skirl $tat8-01-l]l8-art tillery shells. accident andwas pafi of a scienffic group
And now for something completely that examines the damaged nuclear power
arlll0r different. "When you are ice plant.
Engineers of ten prescribe lubrica- skating, " Walker asks, "why do
tion to reduce friction. Different your skates slide along the ice
materials play this role-sometimes, surface? " That is to say, what's
it seems, the most unexpected and the iubrication in this case? (Maybe
inappropriate. By way of example, I'11 it's not so completely different
tel1 you about a case where steel acted after all.)
as a lubricant (though we can't exactly
call it "cold, hard steel"). "Tfil'ottgh l]te misly misland t[e
At the end of the Iast century, the
dt.slry dusk ..."
2Can you figure out how to lower the Damp winter days are always
temperature of the ice-salt mixture misty. We're used to thinking of
without extracting heat from it?-Ed. the mist as grayt but it's not al-

Ot|AIIIUllil/ItJ T1lI I.A8 53


HAPPENINGS

Summor in llleul Yol'k illtd Tarlu,


ltdaltd [Uluscotnl
Students from different countries mix it up
academically and socially

ULY 1989 FOUND 15 SOVIET the chance to attend the a1rcady shown that i{ we take 6 points
students winging their way to Soviet-American school in Tartu [see inside or on the border of an arbitrary
the United States, and a month the essay below by Tania Edwards- convex polygon of. area 1, we can al-
later 47 American students flew Ed.l. ways select three such that the area ol
off in exactly the opposite direction. On |uly ll,after an 1l-hour flight, the triangle with vertices at these points
They were all participants in the Sci- we arrived in Washington, where we is less than 7+ (condition 1 ). Ithad also
ence and Mathematics Intemational were met by representatives of the been proved that, in an arbitrary con-
Summer Instrtutes sponsored by Kvant American Association of Physics vex poiygon of unit area, we can posi-
magazine (Moscow), the Intemational Teachers. And late that night we were tion 4 points such that the area of any
Educational Network (Washingtor! DC), already at the international summer trianglewithvertices at thesepoints
and the American Association of Physics institute on Long Island in New York. will be greater than l+. Here'swhat we
Teachers. The Soviet group basically Classes were conducted in three had to determine: For which convex
consisted of winners oI Kvant'sprob- subjects: mathematics, physics, and polygons is 5 the least number of points
lem contest who also had an active biology. Dr. Alexander Soifer of the needed to satisfy condition 1, and for
command of English. The American University of Colorado taught mathe- which--{? A solution to this problem
students had to pass some challenging matics; Dr. Edward Lozansky, direc- was valued at 50 dollars. (We didn't
tests, but knowledge of Russian was tor of the summer institute, taught manage to completely solve either of
not demanded. physics along with Dr. Alexander Buzdiq these problems.)
Rather than describe the program and Dr. E. Trifonov of the Weizman In the physics classes we solved
in the abstract, we'll let two of the krstitute in Israel gave lectures in mo- problems from many branches of ele-
participants give their candid impres- lecular biology. mentary physics, and there was a physics
sions. Ivan Arzhantsev was a student The math classes were devoted to tournament in which 12 teams com-
at the Physics and Mathematics School mmbuutoric gmmetry and gaph thmry. peted.
No. 45 in Kiev; Tania Edwards had In addition to traditional problems, At the lectures on molecular biol-
recently graduated from the Washing- several were proposed that were of ogy, Dr. Trifonov explained the geo-
ton Intemational School and now at- real scientiJic interest. And there was metric andchemical structure of DNA
tends Brown University. a material reward for those who could and described the latest experiments
solve them. in studying the inner structure of the
Oun rnrp ro AmERrcAwas preceded Here are the problems. Let's take a cell. The lectures were vqry interest-
by a two-week summer school where convex quadriiateral of area 1 and a ing and informal. (The level of biologi-
we had daily classes in physicg mathe- point inside it. The vertices of this cal studies in American schools is
matics, and English (including prac- quadrilateral and the internal point generally higher than in ours, which
tice in conversational English). This form 10 triangles. Of these, let's take cannot be said of the level of physics
experience was not only useful but the triangle with the smallest area. and math.)
pleasant. We had interesting meet- The task is to find the greatest possible We also heard a series of lectures on
ings with members oI the Kv ant edi- value of this area. Solutions to this a wide variety of topics in physics,
torial board, American scientlsts, and and a similar problem with a convex brought together under the general
even a businessman. pentagon were valued at 20 dollars heading "Forces in Nature." They
Of 26 candidates for the trip, 15 each. The next problem had to do were given by the Nobel Prize-winner
students were chosen. The others had with convex polygons. hr class we had Sheldon Glashow, and the views of

54 OUAllIUlll/lllAY I 9SO
such a great specialist were of tremen- After school let out we took a three- cang I went to study math and science
dous interest to all of us. Another day trip to Washington in vans/ stop- at the University of Tartu in Estonia.
Nobel Prizewinner, Dr. Pronin, gave a ping at Princeton University, Phila- My experience proved to be far beyond
lecture on ryrethods of determining delphia, and Baltimore on the way. the simply academic. I not only leamed
molecular structure/ but we didn't But the thinglremembermost is our from some of the best professors in the
enjoy this lecture nearly as much. visit to the Trump Plaza casino in USSR Academy of Sciences, I also had
The final days of the summer insti- Atlantic City, where wallets are emp- the opportunity to interact with Esto-
tute were taken up with math and tied by the pitiless laws of probability nians and Russians who taught me
physics olympiads. The institute's in the midst of feverish eyes and the about cultures very different from my
organizers promised the winners free tinlde of coins. And finally, Maryland own and who had a tremendous im-
trips to the US or France next sum- University (right outside Washingon), pact on how I perceive my country and
mer.1 where we spent the remaining 10 days. those around me.
In addition to completing a broad Seventeen American kids who are The program involved studying for
academic program/ we also just re- planning to visit the USSR came here three weeks and then traveling around
laxed a lot during those two weeks, for three days. We attended several Estonia, Leningrad, and Moscow for
ukingpart in tennis and chessmatches, lectures on physics and worked with one week. At Tartu, we were received
playing volleyball, and canoeing on computers. We took several trips into very warmly since we were the first
the river. Students from four coun- Washington and visited the National Americans to ever stay there. Each
tries participated in the program: the Gallery of Art, the Library of Con- one of us had Estonian buddies who
USSR, the US, France, and Switzer- gress, and NASA headquarters. A showed us around the town/ intro-
land. We rea11y got to know each other highlight was our visit to the Soviet duced us to their friends and family,
because the room assignments en- embassy, where we met with Yuri toid us about their li{e and traditions.
sured an international mix. Ossipyan, Vice President of the Acad- We, in tum/ taught them about Amer-
During the zummer school we drove emy of Sciences and editor in chief of rca, gave them T-shirts, and shared
into New York City, visited the Kvant. our music.
Brooldraven National Laboratory and Without a doubt, we got good lan- In the morning we had math and
hit thebeaches of the Atlantic Ocean. guage practice on this trip, we studied physics classes taught primarily by
We were greatly impressed by our physics and mathematics pretty seri- Soviet scientists. It was very exciting
visit to the suburban home of one of ously, but mainly we saw something to be able to learn things we were
the American families, where for sev- worth seeing-America! never exposed to in our schools, sur-
eral hours we played volleyball and Arzhantsev rounded by Soviet classmates. In the
tennis and swam in their pool.
-Ivan afternoons we saw the city, went to
The kids from each countryput on Lesr suraunn I nAD the incredible op- hear Estonian folk music, saw muse-
show one evening-an improvised portunity to participate in a uniclue ums, played volleyball, and painted
performance illustrating something from program. For the month of August, cabins on the beach of the main lake as
the past or present of their country. It togetherwith about 40 otherAmeri- a souvenir to the city. Onweekends
was myfate to play therole we went to different
of Ivan the Terrible2 ... places such as the beau-
On the last day of the tiful old city of Taihnn,
summer school prizes were the capital of Estonia,
awarded to the winners of where we stayed in an
all the olympiads, touma- Olympic hotel. We saw
mentq and contests. A lot forests, farms, and the
of amusing prLes and sou- highest point in the re-
venirs were given away, arrd public, ate potatoes
there was a little artistic around the clock, went
contest in which Mikhail to concerts/ and experi-
Kapustin gamered the most enced the public baths.
points by improvising on We also went camping
the piano. near the city of Pdrnu
rThe winners were Soviet and on the island of
students in both instanc€s: Ivar Saaremaa, where special
Martin in physics and Ivan permission was required
Arzhantsev in math.-Ed. to enter since it was a
2The Russian tsar who military base. Endless
ruthlessly centralized state Ann Gailitski of New York works on a physics experiment with Mati nights with the guitar
power in the 16th century.- Pim of Estonia at the Science and Mathematics Intemational Summer bythefue made us oblivi-
Ed, Institute in New York. (Photo cowtesy o;f Tatiana Lozansky) ous to everything around.

O U A II I U lll/ II A P P t I'J I I'I G S 55


HAPPENINGS
Of course, these experiences were
incredible, but what was most impor-
tant is that we got to reaily see'how the
political changes, spurred by gJasnost,
were taking place all over the country.
Contrary to or.1l expectations, we were
received by mayors and by well-known
The Americalt Heuiotts
activists, one of whom is a current
Estonian representative to the Con
gress of People's Deputies. We even
had opportunities to ask them ques-
[Ulalhglnatics League
tions about their personal views and
plans. Such contact was very exciting
and led to numerous political discus-
Teamwork is the key in this summer
sions with the Soviet students. competition
On the negative side, in the camp
itselJ we soon found ourselves right in
the middle of the ethnic conflict be- by Mark Saul
tween the Estonians and the Russians,
who were also part of the program.
The Russians were ffeated quite poody.
They lived separately from us, didn't ACH ILINE, OVER ONE THOU- Team questions follow. Teams attack
go on any of the trips, and had no one sand of North America's most able this set of ten "quick" problems by
cheering for them at the games. When and interested young mathemati- dividing up the work in any way they
we approached Estonians with clues- cians gather for a celebration of wish. This event is followedby a more
tions, we received bitter responses: their field of interest-the annual com- traditional event/ consisting of pairs
"They were the occupiers, they don't petition of theAmerican R.egrons Mathe- of cluestions to be answered individu-
deserve to be treated we11." Our friend- matics League (ARML). ally. The team score will be the sum of
ship with the Russiang howeveg geatly This competition is the largest on- the individual scores.
improved in the second part of the site event of its kind in North Amer- A relay round follows a break for
program when we started to realize ic4 drawing more than sixty teams of iunch. In this event, the team of
what was happening. It was especially students from all over the United States fifteem members is divided into three
good when we traveled to Russia-to and from Canada. The teams engage or five subteams, working separately
the ancient fortress-city of Pskov, the in a daylong sequence of events, com- on chains of questions. The answer to
beauti{ul palaces of Leningrad, and prising a variety of individrral and team each question forms part of the next
the museums and shows of Moscow. contest questions. question, and only the final answer is
Like the Estonians, the Russians were Students begin aniving at the cam- scored. Since an error in any one ques-
eager to show us about their life and to pus of the host college on a Friday tion can skew the final answer/ suc-
leam about ours. afternoon. For the past several years cess in this event demands a balanced
When it came time to leavg no one this has been Pennsylvania State Uni- team in addition to nimble individual
could believe that only a month had versity. Some years, telecommunica- minds.
gone by. It felt like we learned im- tions have allowed simultaneous At the conclusion of the competi-
mense amounts not only in classes competition at this site and at Duke tion, scores are tallied and prizes are
but from our new friends that we University in Durham, North Caro- awarded to high-scoring teams and
couldn't say good-bye to. When we lina. Friday evening is devoted to individuals. Publishers and profes-
arrived in Paris, we walked around as recreational activities, including talks sional organizations donate books or
if in afairy-taleworld, bewilderedby by noted mathematicians and teach- plaques. An awards cererr\ony honors
the overflowingmarkets and the oblivi- ers. the winners.
ous people walking by. When our The competition begins in eamest Teams for ARML are formed in a
plane finally landed in New York, we on Saturday momingwith the "power variety of ways. Many large cities and
all burst into applause. We came question." This event, unique to ARML, suburban areas have longstanding tra-
home different-more objective and consists of a single complex question, ditions of these competitions, andteams
more knowledgeable about the world often broken down into separate parts. are often formed as a result. Rural
around us. And I think the other The team must work together for an areas of a state often band together to
participants would a$ee that the feel- hour to produce a single answer paper/ send a team. Some teams represent a
ing we all shared was one of gratitude typically including examples, proofs, single state, and often a state chooses
for the country we live in. extensions, applications, and gener- two or more statewide teams to repre-
--Iania Edwards alizations. sent it. Teams from Canadian prov-

50 ottArrtltlt/tuAY rsso
inces have recently joined those from of their days on the team. Coaches sion. ARML contest questions have
the United States. Criteria for team give generously of their time-and provided students with ideas forpize-
membership are, fot the most part, left sometimes of their money-in sup- winning research projects and fine
up to local tradition. port of their teams. expository papers.
The backgrounds of team coaches An interesting by-product of the But the most important source of
and chaperones vary as much as the ARML on-site format, and particu- stimulation at the ARML competi-
teams themselves. Frequently, a teacher larly of the power question event, is tion, and the one source that is diffi-
or math team coach assembles the the possibility of continued investiga- cult to find in other mathematical
team, but sometimes the coach is from' tion even after the conclusion of the contests and events/ lies in the inter-
the faculty of a local college or unlver- contest. ARML contest material is de- action of the students themselves. Team-
sity. Many teams are accompanied by signed by the authors to be open-ended. mates share ideas and pool intellec-
alumni of ARML, who have retumed Even the most humble relay question tual resotuces. Students on rival teams
as coaches to rekindle the enthusiasm can often have an interesting exten- meet informally, exchanging solutions
to problems and posing new
challenges to each other.
Ilte 108$ Afittlll powet' qtts$tiolt Some play the game of uying
A convex n-gon will be called "Pythagorean" if it has integer sides, it is to find flaws in the day's
cyclic, and its longest side is a diameter for its circumscribing circle. It shall contest problems. These inter-
be denoted by Pn or Pn:(a,b,...),where a,b, ... are the lengths of its sides. We actions lead to the forma-
shall always use the letter d for its longest side. (Thus P3 is a Pythagorean tion of a network of student
triangle. Notice that it would be a right triangle.) mathematicians. Friendships
L There is a theorem that states (in part): If a prime d is the hlpotenuse of started over lunch tables at
a Pythagorean triangle, then d2 is the hlpotenuse of two Pythagorean tri- ARML are often continued
angles, d3 is the hypotenuse of three Pythagorean triangles, and so on. long after the day's end. For
I.A. Find two P3's for which d : 25. many students, the shared
LB. Find three P3's forwhich d: 125. enthusiasm of the ARML
II. Ptolemy's Theorem says: A convex quadrilateral is cyclic if and only if competition becomes a cen-
the product of its diagonals equals the sum of the products of the two pairs of tral experience in ther lives.
opposite sides. For more injormation on
II.A. If the P313,4,51is reflected as shown in the figure, a quadrilateral EFGH ARML, please write to Ste-
can be formed. (It will not be a P4, as FG is not an integer.) Multiplying each ven Adrian, ARML Execu-
sideby5 produces aP4. Findthe sides of this P4. tive Director, R.D. 5, Box
133, Kings Ridge Road,
Mahopac, NY 10541USA,
orBarbaraRockow, ARML
Corresponding Secretary,
Bronx High School of Sci-
erce,75 West 205th Street,
Bronx NY 10468 USA. O

H Mark Saul is the computer


II.B. Find a P4 wirh two equal sides and with d = 25 that is different foom the consuhantfcoordinator for
answer to part tr.A. (Note: Two lln's arenot considered different if their sides Bronxville School in N ew York
and president of the American
are equal but in a different order.)
R egions M ath em atic s Lea gu e.
tr.C. Show that a Pn must exist for all integers n > 3. (This may be done by
describing how to create such a Pn.)
III.A. For the P3:(a,b,dl, d2: a2 + b2. Provethat for the P 4:(a,b,c,d), d2 > az
+b2+cz.
Itr.B. Given the P4:(a,b,c,d), prove that iJ d > 2, then d must be composite.
Itr.C. If all the diagonals of a Pn are integers, we will call it "super Pythago- The 1989 ARML cantest
rean" and denote it by sPn. questions were created by
m.C.1. Show that the area of any sP4 must be an integer. (Hint: One Gilbert W. Kess/er, Canarste
Htgh School (tetil ed/. Braa^-
approach mlght be to first show that the area of any sP4 must be rational and
lyn, NY. Harry Ruderman,
its perimeter must be even.) Hunter College Campus
m.C.2. Assuming that the area of every sPB and anery sP4 is an integer, show SCnoal \teltted). t\e* Yor(
that (for all n > 4l the areaof every slln must be an integer. (You may do this part NY, and Larry Zimmerman,
even if part ltr.C.l has not been completed.) S'LUZ'/V oN pAcE 62
Brooklyn Technical High
School Brooklyn, NY.

O lJANIU IIl/IIAPP IiI I !I G S 5l


HAPPENINGS

Bulletilt Boal'd
Batlel'y-dl'iutn ideas earn scholal'shhs
The eighth annual Duracell NSTA reducing the time duringwhich cur- Ten students were given $500 schol-
Scholarship Competition really got rent flows rather than cutting back arships, and25 students received $ 100
the creative juices flowing. To enter the current. Albert Masaki Hunting- cash awards. The top winners ex-
the competition, a student had to de- tont a senior at Pleasant Hill High piained their devrces to an audience of
sign and build a device that is educa- School in Oregon, came up with the educators and scientists at an awards
tional, useful, or entertaining and is Portasynth, a compact/ lightweight, luncheon during the recent NSTA
powered by one or more Duracell bat- higlrly efficient keyboard with 63 keys annual convention in Atlanta, Geor-
teries. |erry Pratt, a senior at Ashland designed to be practical for trips into gia.
Higfu School inWisconsrr, was awarded space. Matthew |ared Klam, a senior To find out how to enterthe ninth
a $10,000 schoiarship for his "Knock- at the Wisconsin School for the Deaf, annual Duracell NSTA Scholarship
Out" Keyless Door Lock. You pro- devised a Walkie-Talkie Device for Competition, write to Katie Rapp, Na-
gram the device's microcomputer by the Deaf, which incorporates a port- tional Science Teachers Association,
tapping out a rhythm {a favorite tune, able screen and keyboard for sending 1742 Connecticut Avenue NW, Wash-
the Morse code of someone's name, and receiving messages. Robbie GIen ington, DC 20009, or call 202 328-
whatever), and the door can't be opened Seibert, a junior at Grand Rapids High s800.
from the outside unless the encoded School in Minnesota, invented a Speech
knock is repeated. |erry says his Knock- Leader-a talking compass that uses a
Fl'ee 3-[ relort brings lnohular
Out lock is an improvement over other directional sensor/ speech processor
electrical locks because it's tamper- chip, leveler, and alarmclock. Kurt [ltrl8$lolil8
proof from the outside. Thorn, a sophomore at Shoreham- A colorful new report from the
Five other students each received a Wading River High School in New Howard Hughes Medical Institute,
$5,000 scholarship. Theodore Gielow, York, created the Multipu4)ose Angle Finding the Critical Shapes, shows
a senior at Newport Hafior High School Measurer for recording the degree of how the answers to many riddles of
in Califomia, invented the High-Effi- mobility in the joints of persons who modem biology and medicine lie bur-
ciency DC Light Dimmer, which are in physical therapy after an acci- ied in the intricatg three-dimensional
decreases the brightness of a light bulb dent, for example, or who are arthritic. shapes of our bodies' molecules. Re-
(for example, a night light on a camp- It's based on apendulum hooked to a searchers in the field of "structural bi-
ing trip) without wasting power by variable resistor. ology" increasingly depend on com-
puters not only to do their
mathematical computations
but to help them see their
results. This beautifully il-
lustrated report includes a
stereoviewer to help the reader
visualize molecular shapes
in three dimensions. This
kind of view provides in-
sights into the mechanisms
of previously untamed dis-
eases and help.s scientists
design new drugs to combat
them.
For a free copy oi Finding
the Crilical Shapes, write to
the HowardHughes Medi-
cal Institute, Communica-
tions Office, 5701 Rockledge
Drive, Bethesda, MD 20817.
Award-winning devices from the 1989 Dwacell NSTA Scholarship Competition (left to right):
"I(nock-Out" Keyless Door Lock, High-Ef t'iciency DC Light DimmeL Envtuonmentally Dtuected
Watering Deuice, Speech Leader, Portaslmth, Wakie-Takie Device for the Deaf

58 OUAIIIUlirl/1IlAY 1 SSO
SOLUTIONS

tulallh
M6
For each of the three given circles,
draw radii from its center to the points
where it intersects the other two (the
thin lines to points A, B, C,H in figure
1 ). Three rhombi appear (with com-
mon vertex H) whose sides are ai1
equal to r. We can imagine them as
representing three faces of a parallele-
pipedwith commonvertexH. Draw
the three other faces-that is, three
new rhombi with common vertex O.
Their new sides (the thick iines OA,
OB, OC in figure 1) are also equal to r. Figure 2
And this is precisely what has to be
proved. (Notice that our reasoning You can find a detailed discussion hal{ of the n products equals -1, and so
also holds when B lies outside triangle of this problem in the remarkable book n=4k.
AHC.I Figure 2 creates a more con- M ath em atic al D is cov ery by George
venient view of the situation bybreak- Polya.
ing apart the parallelepiped and elimi- Tfestrip between the exteri.or polygon
nating the distracting portions of fig- M7 Po and the interior one P, can be cut
ure l. Imagine that the numbers x1, x2, ..., xn into rectangles with altitude I (whose
We'llleave it to you to find other are written clockwise around a circle. bases are the sides of P,) and quadrilat-
solutions to this problem and lots of Let k be the number of times the Po when
erals left near the vertices of
interesting facts of triangle geometry number 1 is followed (clockwise)by the rectangles are removed (fig. 3).
related to it. You may have noticed, -l in the sequence xt, xz, ...t xnt xl Take these quadrilaterals and fit them
for example, that triangle ABC is con- (which we'll call the "basic sequence"). together by parallel translations so
gruent to the triangle with vertices at Notice that the number of reverse that their irrner vertices (the ones origi-
the centers of the three given circles; changes (from -1 to l, clockwise) is nally on P,) all coincide. We then
that B is the orthocenter of triangle the same. [Aary?) The totalnumber of obtain a polygon P circumscribed to a
AHC; that the points symmetrical to negative 1's among n products xlx2l circle with radius 1. Its vertex angles
B relative to the sides o{ttarrgle AHC Xrx3t ...t x;r' {the "product sequence") are the same as those of P, (or Po) and
lie on its circumcircle; and that all is equal to the number of sign changes its sides are equal to the differences
f our circumf erences play equivalent in the basic sequence, which is 2k. between the sides of Po and P, .
roles-that is, every group of three But the sum of the members of the If the sides of P, are proportional to
circles has a common point. product sequence is 0 only if exactly the corresponding parallel sides of Po
with a coefficient k < 1, then the sides
of P xe clearly proportional to those of
Po with a coefficient 1 -k. So Po and P,
are similar to P, and al1 of them arc ctr-
cumscribed to a circle.
In this solution we've tacitly as-
sumed that the similitude between Po
and P, is the natural one (that is, the
correspondence between the sides is
the one induced by the outward mo-
tion). It can be proved that if any
similitude exists, the natural one ex-
ists too. We'l1give another solution,
however, that doesn't rely on this as-
Figfre Figure 4 sumption.

0lJArrrrlJl|il/$0r.lJil0ils 59
nice. After seven iterations the first
dwarf gets back all his mil( therefore,

5xr: xr+ x3 + ... + x7.

Since x, >xo, this is possible only if x,


= xz= ...: xr. Suppose xis the smallest
portion of milk added; then by as-
sumption
x+2x+...+5x=3,
so that x = ll7, and the answer follows.

p6 Physius
In order to avoid a collision, the astro-
naut must change the velocity of the
Figure spaceship so that the angle between
the initial direction toward the aster-
Let the inner polygon P, be the Thus, Bl27 = (213)(2l3ltzl3) in the oid and the new heading is greater
image of Pn under a similitude with a previous example. Since we have than the angle cxo determined by the
coefficient k < 1. Then the image { of .p116l, condition
(315l[
P, under this similitude can be con- :
= 3111/.5? .28:| sln (r^
structed from P1 in the same way as P, ll
1+dlZ
was constructed from Po but with a k-
= L77,t47l2o,ooo,ooo,
d
{old reduction of all distances-that it follows that a well-orgarized9-stage 2l+d
is, by moving the sides of P, the dis- election will guarantee Miraflores's 0.292
=
tance k toward the interior (fig. a). reelection with less than 200,000 red
ThenthepolygonP, (theimage of Pr) voters. An even smaller number of (see figure 71. Nter the additional
can be constructed by moving the reds, 154,025 = 38 .25, is sufficient in a velocity Av is imparted to the ship,
sides of P, the distance ft2, and so on. lO-stage election, since maximum deviation from the initial
The intersection of all the polygons heading is provided if the vector Av
. . is a single point O. Each .3 . turns out to be perpendicular to the
P P P
o, 1, 2,
. (3
I sY I 4 (s l9l' = t64,O2s I 20,000,000.
side of Po moves the fistances l, k, k2, vector v1 = v + Av (fig. 8 )-that is, if
... toward O, so that the total distance M10 sina= Lv/v=0.3.
to O is equal to the sum of the infinite You may have found it easy to guess
geometric progression 1 + k + k2 + ... = and verify the answer: the portions of So by switching on the emergency
llft -kl.It follows that P and P, are milk are 0, | 17, 217, 3 17, 417, 5 17, 6 17 engines, the astronaut can change the
both circumscribed to circles with liter. When the dwarf represented by heading of the ship by an angle 0 > Gc,
center O. the solid circle in figure 5 has given his and there will be no collision with the
portion of milk to the others, the dis- asteroid.
M9 tribution turns out to be the same as
Let's call Miraflores's suppofters "reds" the initial one but rotated by one- P7
and the other voters "blues." The seventh of a full turn. So after seven Let's consider the raft's motion in a
situation illustrated in figure 5, where iterations (repetitions), the original frame of reference anchored to the
eight red voters out of a total of 27 distribution is recovered. current-that is, moving at a speed of
voters guarantee the victory of the But it's not so simple to prove the u. In this context the raft has an initial
reds in a 3-stage election, clari{ies the answer's uniqueness . Let Sxobe the velocity v' = v - u and moves in a
main principle: in each group where portion of milk that
the reds win, they have a minimal the kth dwarf pours
tul, o
majority, whereas in each group won
by the blues the vote is unanimous.
Using this principle, we see that Mred
out (xu Jiters to each
of the other dwarfs), ) sl7 i
wherek =1,2,...,7.
votes out of a total of Nwili win in an Without loss of
appropriately organized r-stage elec- generality we can alt ilz sl7 217
tion if the ratio M/N can be repre-
sented as a product of r fractions each
assume x, > xo for
all k, since all the t 317 zlz I qlz tlr
larger than If 2. dwarfs are equally Figure

$[l otllrTUrir/l,tAY l sso


Figure Figure B (at point O) by the paper very close to each other-for
-+ current lflowing in example, at the distance 1= 1.5 mm.
Moving the sheet away from your
eyes, estimate the distance at which
the points merge into one point. If you
have normal eyesight, this distance
straight line. Here the ra{t's velocity about the y-axis, circuit ABCDA is should be d = 2 m. The angle ct - l/d is
v'is reducedbecause of the force of transformed into itself, and so the the least angular distance between the
water resistance. (If the resistanc6 vector Bo should also be transformed points at which they are distinguished
were absent, in time t the raft would into itself. Therefore, by the cork- by the eye as two objects. This angle
be at point c with coordinates x c: 1tt t screw (or right-hand)rule the vector char acterizes your eyesight.
y"= vt.l The displacement of the raft Bo must be directed along the y-axis. The rails will seem to merge when
relative to the nverbank in time t is a kr projections on the x-, y-, and z-axest the angular distance between them is
combination of its displacement s,: o. If the rails are l' = 1.5 m apartl they
Bo= lB,,Br,B,l = lO,Bo,Ol.
v't relative to the water and the dis- seem to come together at the distance
placement of water s- = ut (fig. 9). To find the field B created by the d' - l'f a* l' dll =21<n.
During time 2t the water displace- current flowing in the circuit lB CGHEA
ment will be twice as great. From (fig. I l ), we use the superposition prin-
point O let's measure ofl a segment ciple. Notice that exactly the same Brainlea$8r$
with a length of 2s-along the x- B6
axis and draw a straight line Keeping the goat in the boat, take the
parallel to s, {through the point dog and then the cabbage across the
with coordinate2s*|. This line river. Take the goat back and leave it
intersects the raft's trajectory on the riverbank (all by itsel{). Take
at the point markedby the blue the two wolves across the river. Re-
cross-the raft will be here at
2t after the raft was
ii tum with the dog. In the last crossing
take the dog and the goat across.
tirrre /
launched. I
Similar constructions can be B7
used to find where the raft will Theproblem isbased on thefact that
be at times 3t, 4r, and so on. Figure 9 adding the mean value of a set of
numbers to this set gives a set w'ith the
distnbution of current on the edges of same mean value.
PB
It' s clear from the condition that the the cube will be obtained iI we take
po\{,er frorn the heating element is three circuits, ABCDA, DCGHD, ar-.d
equal to the energ.v leavlng the water ADHEA, il each oi which the curent F
and entcring tht cttr irrlnttlent o\ ut' is 1. The combination of these three
trme. (The temfre rature oi the u'ater circr-rits creates the reqr-rired iield B in
doesn't change u,ith trme rrhile the the center oi the cube. According tcr
element is on.) So ii the heatrng e1e- the superposition pnncrple, B is equal
ment is turned off, the energ)- trans- to the vector sum oi the three fieids
ferred over time by the u,ater rvl11 be cruateJ l',r-cach of thc crtcuits:
100 W. Recalling that heat flow Q :
B :B.r.,.r, - B,'c;.i. * B.,--.r.,
Cm. LT and that Q = Pt (where C is
speci-fic heat, m is ntass, 7 is tcmpera- =\o,8,/o' -,-B o or - \o,o,B \ D
ture, and P is power), we find that the : \-B ,'r,B ,',\.
,
Figure 10
water will cool down by one degree ',8
during the time So the vector B is directed along the
cube's principal diagonal DF and its
, =9!i'LT value is Bo3Yl
C
P
The induction B can be found by
(,1.2 . 103J/kg .'c) . lkg . .l'c simply adding together the fields of all
100 w six edges of the circuit, but this re-
=42s. quires more cumbersome calculations.
P9
From considerations of symmetry, it Pl0
You might proceed in the following D
follows that the induction Bo of the
field created in the center of the cube way. Draw two points on a sheet of ilgure r r

OUAl1lIUllll/SOI.UIIOl|lS 0l
88 x39
28s I.B. 135,t20,t25l,,
V S,lm )251, arrd (44,117,125).
2555 The first two come from the
.+855 17 ,24,251 and 13,4,51triangies.
11115 A general approach would
B9 be to use the fact that the
The fiver can tell which way is up by orpressions k(ff? - r?1, k\nn\
watching the bubbles coming out of andklmz + nzl,wherek, m,
his breathing apparatus orby droppiirg and n are positive integers
a pebble. with m > n, produce all Py-
thagorean triplets. Setting
810
Combustion occurs when there is an
d
N
m2 + t*: 125leads to m: lQ"
n - 5, which produces
influx of oxygen. Under ordinary * (75,1N,125), ot rri:ll, n=2,
conditions on Earth the in{lux of oxy- ! which produ crcs 144,117,1251.
gen is due to convection: near the E Settingm2 + n2 = 25leads to
flame heated air, which is fufrter, ascends i m:4,n=3,producing
together with the products of combus- i (7,24,251, for which we then
tion; colder air, containing oxygen, 7. Since the atmosphere at those usek = 5.
takes their place. [r the state of weigfrt- heights is very sparse/ the number of tr.A. Ptolemy's Theorem produces
lessness there will be no convection, molecules per unit of volume is too FG =715. The answer is(15,7,15,251
and the flame will die from a lack of small to impart an appreciable amount (in any order).
oxygen. of energy when they collide with a sat- II.B. Using sides (a,x,a,25), where
ellite. each diagonal is(625 - aLlvileadsto x=
l(aleidoscoru 25 -2a2125. Positive integral /scome
8. If the Moon ever had an atmos- from a: 5, 1 0, or 1 5. The last leads to
1. Particles of smoke take part in pherg it disappeared over the eons of the solution in II.A. The other possi-
Brownian motion and move off little its existence. Amid the myriad mole- bilities are 15,23,5,251 or (lO,l7 ,10,251.
by little so that the density of the cules of atmosphere, there were a1- (Either answer/ in any order, is accept-
smoke decreases. ways some whose velocity of thermal able.)
motion would achieve the escape ve- tr.C. Apply the theorem given in
2. The Brownian motion of the fat locity for the Moon. partt for any appropriate d, to find n -
droplets is weakened. 2 P3's of. hypotenuse d"e. Build these
9. In putting the pieces back to- triangles in
a semicircle of diameter
3. The number of collisions of a gether, it's practically (as opposed to d"e. Connecting successive points on
liquid's molecules with the surface of theoretically) impossible to position the semicircle produces an n-gon. Suc-
a particle is proportional to the surface the fracture surfaces at a distance where cessive applications of Ptolemy's Theo-
area, whereas the mass of the particle the forces of molecular attraction will rem to find dach side of the n-gon
is proportional to its volume. So the be "noticeable." shows each mustbe rational. Multi-
larger the particle, the harder it is for plying all sides by the least common
molecules to move it. A Brownian 10. Because of close contact over a denominator involved produces a Pn.
particle must also be small enough long period of time, the atoms of the IILA. On diameter AD, draw the P4
that collisions with molecules will be nut andbolt intermix along the bound- ABCD,withAB: a, BC = c, CD: c,
uncompensated. arybecause of diffusion and the two AD : d, BD = e, and angle BCD = 0.
pieces "lock." Thelaw of cosines shows that * = a2 +
4. The rate of diffusion increases as d -2k,cos 0. Since 0 must
= az + W + c2
temperature increases. 11. Work to change the level of a be obtuse, cos 0 is negative, so d2 is
liquid in a capillary is performed by gteater than az + b2 + c2. Notice that
5. Because offrequent collisions, the energy of molecular interaction. this easily extends to any Pn.
the molecules move in zigzags. Their III.B. Using the P4 described in so-
actual paths are much longer than 12. The pressure would increase. lution III.A, with AC: f , Ptolemy's
their perceived change of position would Theorem yelds bd + ac = ef =ll* - a'l
imply. (il - c2ll4 Squaring both sides, simpli-
ARIUIL potnlet' quesliolt fying, and dividing each term by d
6. Water vapor diffuses slowly thrcugh produces bzd + Zabc: da - a2d - czd.
the film of lacquer so that the wood I.A. (7,24,251 and ll5,2O,25), the sec- Since all are integers, d must divide
dries evenly throughout and the ball ond of which is five times the sides of 2abc. If d is a prime greater than 2, it
doesn't crack. (3,4,5). must divide at least one of the a, b, c. IL

02 otlArrrlUrir/rirAY rsoo
dla, for example, then a > d, which is SUMMER PROGRAM IN MATH AND
impossible. So d is composite or equal
to 2; therefore, d is composite. SCIENCE FOR
m.C.l.( 1). Using the P4 described in
solution III.B, we first show that its
HIGH SCHOOT STUDENTS AI\D TEACHERS
You are invited to participate in an exciting US-Soviet exchange program: the
area must be rational. The area of
triangle ABD = aef2,whichis integraf 1990 Science and Mathematics International Summer Institutes to be held at
since one of the legs must be even (see LaSalle Academy, Long Island; the University of Maryland, College Park; Moscow
solution I.B); the areaoftriangle BCD State University, USSR; and the University of Tartu, Estonia.

= lbclzl sin 0, which must be rational


since sin 0 = e I d (by theextended law HIGHLIGHTS OF THE PROGRAM
oi sines); thus, the area of. the P4 is * Advanced mathematics, physics, computer science, and molecular biology
rational. courses
m.C.1.(2). We next show that the * Russian language and literature
perimeter of the P4 must be even. We * Irctures by prominent scientists
first note that all primitive Pythago- * Visits to scientific laboratories
rean triplets (generated by tn2 - n2, * Discussions and debates
Zmn, andm2 + n2, where mandn areol * Cultural enhancement from the international group of participants
opposite parities) must yield an odd * Excursions to New York and Washington in the US and to Leningrad and
hypotenuse and exactly one even leg; Moscow in the USSR
if a Pythagorean triplet has an even * Chess, sports, sandy beaches, films, concerts, and more
hypotenuse, each leg must have at
least the same degree of evenness as The Institutes are coordinated by the National Science Teachers Association
the hypotenuse. Now by Ptolemy's (NSTA), American Association of Physics Teachers (AAPT), National Council of
Theorem, b: lef - ac)ld, and we are Teachers of Mathematics (NCTM), and International Educational Nefwork in
given that this is integral. There are cooperation with the Brookhaven National Laboratory and the USSR Academy of
now two basic possibilities: (1) d can Sciences.
be even {then both ef and ac will be of
a higher degree of evenness, making b
For more information, please ffll out the coupon and mail to:
even and the perimeter even) or 12) d
can be odd (then either e and / are even
while a and c areodd, making b odd; or Dr. Edward D. Lozansky
e and f are odd while a and care even,
NSTA
77 42 Connecticut Avenue, NW
makrng b odd; or e artd f are of opposite
parities, as are a and c, making b eveni Washington, DC 20009
(202)362-7855 or (202) 328-s800
rn each of these situations the penme-
ter will be even). Thus, the perimeter
is even in every case. Please clip and mail
m.C. 1.(3 ). With s as the semiperl-
meter/ Hero's Formula for an inscnbed Last Name First Name
cluadrilateral grves area = fis - a) (s - b]
(s - c) (s - d)l'1; since s will be integral,
Address
the area is the square root of an inte- City State zip
ger. But for the area to be rational a1so,
it must actually be integral! Home Phone (
I1I.C.2. This is done by induction.
We will just indicate the basic ap-
nroachhere: Given the sPn ABCD ... Please check ifyou are a high school student or teacher:
I
I - --l- ondiameterAV, letthe areaol
::-= . =!C be K, the area of ACD ... _ High school teacher
I, :. ,: =-: :he area of TUVbe K" . main subject you teach
\\-: ," -:. - -.-.:-.:---: ::'ra nf anr lln -
1'a::; -- - - .:. -:-:-lj' :hrs is our
--- _ High school student
"trI:::--:- - ---'. - ,-:-:::'-: :'l .

Pl;.s; send me _ additional brochures and application forms to circulate among

T.t t' *
:.:. -: : .::11 \:uJents u ho might be interested in participating in this program.
r\ -n ----:.:--
K" is rnt-=,

OUAI\IIUM'SOLUTIOiIS 03
CHECKMATE!

Symmetry ott lhs uhesshoal'd


Sometimes it happens by chance, sometimes by design,
but it always has a certain charm

by Yevgeny Gik

HE "SYMMETRY MOTIF"
arises often both in the composi- (12.
12. Bd3xezl Nd7-c5
... Nd7 f(r would have been bet- N-NN
"\NN'NfN
:s N
NsN

tion of chess problems and in ter. )


N\N s*.**ffN ssN
actual play. This geometric theme 13. Bel-c2 Bc8-94
N*NUN\N
is a lot of fun and seemingly inex- l+. Qtll xd+ Bg4xr3
NN*- N oN
15. g2xf3
*N\r N 'N N
N\\s\
haustible. Rf8-e8
Flere's a game from a published 16. Rf1 dl Re8-e2
collection of chess curiosities. It was 17. Bc2-f5 s7-s6 .\\N .N*g.N -...\
played at the championship of the 18. Bf5-h3 Nc5-d7 .\N.N N *\
USSR in Moscow in 1931. 19. Bc1-e3 Bc16-e5
20. Qd4-c4 Re2xbL
M. Botvinnik-N. Rlrrmin )1. R:r1-c1 G. -\.larlson 1921. To rvin.
lCorlmentilg on this garre, Bon-ur-
1. d2-d4 d7-d5 nik notecl that 21 . d5-d(r rrould hale 1. \e'-d5l Ne5-d7
2. c2-c4 c7-c6 lecl to r.ictorl' lrore r1-rickh-. 1

,Altern:rtives: 1. ... Ne5-f7 or Ne5-


3. NgI-f3 Ng8-f6 zt. Nd7 b6 g(.2. Be2 h5; 1. ... Ne5-c6 2. Be2-b5.)
4. e2-e3 e7-e6 22. Qc4-e4 Qd8-d6? ). Ke6-d(rl
5. Bf r-d3 Nb8-d7 122. ...Be5-d6 would have been lBut not 2. Be2-b5? Ke8-d8l 3.
6. 0-0 Bf8-d6 stronger-now it's all over.) Bb5xd7-stalemate. J

7. Nbl-d2 e6-e5 23. f3-f4 Be5-g7 2. Ke8-d8l


B. e3-e4 0-0 24. Be3-c5 Qd6-d8 Nd7 b8 won't save black: 3.
{2. ...
9. c4xd5 c6xd5 25. Bc5-e7 Qd8-e8 Be2-b5+ Ke8-d8,1. Nc15 b6 Nb8-c6l 5.
10. e4xd5 e5xd4 26. d5-d6 Qe8-b5 Kd(rxc6l; nor rvill 2. ... 5d7-f8: 3. Be2-
The opponents hadplayed out the 27. d6-d7 Nb6xd7 h5+ Ke8-d8 4. Nc15 i6l KdS c8 5. Kd6-
Slavic Defense, and there was no de- 28. Bh3xd7 e7.\
sire to copy moves on black's part, let A few moves later, black resigned. 3. Be2-d3
al0ne white's. But after ten moves the Here arc- some possible ways of
players were no doubt startled to see The basic idea of the "symmetry- workrng tlls positron out: 3. ... Kd8-c8
the perfect symmetry on the board, as asymmetry" genre of chess problems 21. Nd5-e7+ KcS d8 5. Ne7-c6+ Kd8-eB
well the unusual concentration of
as is that an externally symmetrical 6. Bd3-g(r*; 3. ... Kd8-e8 4. Nd5 c7+
pieces on the d-file. Nevertheless, the position has an asymmetrical solu- KeB d8 5. Nc7-e(r+ KdB-cS 6. Bd3-a6+;
preference in a symmetrical position tion. Of course, there can't be abso- 3. ... Ndz b8:1. Nd5-b(r Kd8-e8 5. Kd6
generally goes to the player whose lutevertical symmetry on the chess- c7i 3. ... Nc17 f8 4. Nd5-f6 Kd8-c8 5.
move it is. In this case, Botvinnik board because of the presence of an Kd6-e7l and the knight is caught. Why
skillfully makes use of this advantage, "extra" file. In {act, paradoxical situ- r,r,ou1cln't i. Ne7-f5 work? Let's see
ably de{ending his d-pawn while at- ations arise precisely because of this hon, it p1a.vs out: 1. ... Ne5-i7l 2. Ke6-
tacking his opponent's. file. The following examples are all f6 Kc8-f8 3. Be2-f3 KiS-e8 4. Nf5-g7+
11. Nd2-e4! Nf5xe4 problems based on this phenomenon. Ke8-f8 5. Ng7-s(1KfS g8! As it tums

84 ottAtrtrtttll/tltAY rsso
out, the bishop needs one more file to 4. Kc5-d4 Kg4xg3 In this example, the h-file will be
check the king. 5. c4-c5 S5-54 used in a completely di{ferent way.
6. c5-c6 Kg3-f2 1. d5xe6! d6xe5
Let's look.at another problem. 7. c6-c7 g4-SB (AIter 1. ... d6xc5 2. e6xl7 Kd8-e7 3.
8. c7-c8Q S3-52 e5-e5, the moves by biack ultimately
A. Seleznev, l9l7-white king on The queen can easily manage the lead only to a stalemate.)
e5, bishop on a8, and pawn on e6; black pawn. What if the g-pawn is 2. e6xI7 KdB-e7
black king on 94, knight on f4, and advanced first? 1. g3-g4? Ke6-d6! 2. 3. Kd3-e4 Ke7xI7
pawns onb4 andh4. To draw. Ke4-f5 (2. c3-c4 Kd5-e6) 2. ... Kd6-d5 3. 4. Ke4xe5 KI7-e7
K{5x95 Kd5-c4 4,K95-14 Kc4xc3 5. 94- 5. Ke5-f5l Ke7-d7
"Wherds the qrmmetryhere?" yotfte 95 c5-c4 6. 95-96 Kc3-b2 7. 96-97 c4-c3 6. Kf5-e5 Kd7-c7
saying. It tums up after three moves: 8. S7-S8Q c3-c2. The white pawn has 7. Ke5-d4 Kc7-b8
1. Ba8-e4! Nf4xe6 allowed black to get a draw (the stale- 8. Kd4-c4 Kb8-a7
2. Be4-f5+ Kg4-f3 mate idea)l 9. Kc4-b4 Ka7-a6
3. Bf5xe6 Kf3-e3l 10. Kb4-a4
It seems white won't be able to save Draw.
itself (one of the pawns will march on
to become a queen). Andyet...
\NUNN
N t..*.N t
Opening to the other side won't
work, though: 1. d5xc5? d6xc5! 2.
4. Ke5-d6! Ke3-d4 *..N NN.SN
c6xb7 KdB-c7 3. Kd3-c4 Kc7xb7 4.
5. Kd6-c6 Kd4-c3 *oN I \It^ tN* Kc4xc5 Kb7-c7 5. Kc5-b5 Kc7-d7 6.
6. Kc6-d5! b4-b3 N SAW\ o*N
"N Kb5-c5 Kd7-e7 7. Kc4-d4 Ke7-f8 B.
7. Kd5-e4 b3-bz
B. Be6-a2 h4-h3
N **N
.NN **N Kd5-e4 Kf8-g7 9.Ke4-t4 Kg7-h6! Black
has now gained an advantage thanks
9. Ke4-f3 *\N Nffio*N *\-* to the maneuver on the "extta" file on
*N .N **\ N'* o
.N N .N .\.N
Both pawns are held back. U white's the far right.
fourth move had been Ke5-f6?, the
white bishop would not have found a
position at the right corresponding to O. Riildrimaa,1942. To draw
a2. It appears that this is the first chess
problem in history with "symmetry-
asymmetry."
Methods ofMotion
An lntroduction to lvlechanics
BookOne

^$..*N. .$ How do objects move? Isaac Newton really believed that an object moving in
astraight line would continue with constant speed. Do your students? This

iSh manual was created to help teachers introduce the sometimes daunting

NNN "r*\NN
N
subject of Newtonian mechanics to students in the middle grades. The 27
activities presented here use readily available materials to give students
visual, aural, and tactile evidence to combat their misconceptions. And the
teacher-created and tested modules are fun: Marble races, a tractor-pull
Y. Kneppel, 1967. To win. using toy cars, fettucini carpentry, and film container cannons will make
teachers and students look forward to class. Readings for teachers, a guide
At first glance, it's not clear what for workhop leaders, and a master materials list follow the activities,
significance the extreme left file has making this manual useful for inservice workshops.
in this pawn problem. After all, the #P839, 1989, 168 pp. $16.50
pawn isn't in a position to land on it.
1. c3-c4! Ke6-f6 from A/S714 Publicotions
(Altematively: 1. ... Ke6-d6 2.Ke4-
f5 95-g4 3. Kf5xg4 Kd6-e5 4. Kg4-h5 All orders of $25 or less must be prepaid. Orders over $95
Ke5-d4 5. 93-g4 Kd4xc4 6.g4-g5Kc4- must include a purchase order. All orders must include a
d3 7 . 95-96 c5-c4 8. 96-97 c4-c3 9. 97 - postage and handling fee of $2. No credits or refunds for
returns. Send order to: Publications Sales, NSTA, 1142 Con-
gSQ c3-c2 10. Qg8-s5.)
necticut Ave. NW !(ashington, D.C. 90009.
2. Ke4-d5 Kf5-fs
3. Kd5xc5 Kf5-g4

OllAlllTUllll/[llICI(lllATI! 85
W

ffiqffi__--.-

ffi,:E*

ffi
rc
ffi*=:

Dr. Robert Ballard is taking the robot JASON


and 250,0@ students .to the bottom of Lake
Ontario in May 1990 to look at War of 1812
shipwrecks. The NSTA Great I-akes JASON
Curriculum includes 360 pages of expedition
activities, a poster, map, and gameboard. Order
from NSTA Special Publications, 1742 Connec-
ticut Ave., N'S7, Washington, DC 200@. include
check for $14.95.

r:.iitii}-:j-:l,tj.1;tr.ql

Вам также может понравиться